ORTHOPEDIC MCQS ONLINE 20 OB TRAUMA 1D

ORTHOPEDIC MCQS ONLINE 20 OB TRAUMA 1D

 

fractures is associated with decreased shoulder strength and increased nonunion rates. Risk factors for non-union in non-operative management of midshaft clavicle fractures include advanced age, female gender, displacement and comminution. Open reduction and internal fixation of clavicle fractures is associated with improved results when there is > 2 cm of shortening and 100% displacement.

 

Zlowodzki et al. retrospectively to compared nonoperative versus operative management of 2144 clavicle fractures. They found that non-operative treatment of displaced midshaft clavicle fractures had a nonunion rate of 15%. Risk of nonunion was reduced by 86% when plating was compared to nonoperative measures.

 

The Canadian Orthopaedic Trauma Society conducted a multicenter prospective study comparing non-operative management versus plate fixation of midshaft clavicle fractures. They found that Constant/DASH scores were significantly improved in the operatively treated group at all time points up to one year of followup. Additionally, a lower rate of nonunion/malunion was identified in the operative group.

 

Figure A shows an AP radiograph of the clavicle that is significantly shortened and displaced. Illustration A shows a postoperative AP radiograph of the clavicle after open reduction/internal fixation was completed.

 

Incorrect Answers

Answers 1,2: ORIF is associated with decreased rates of nonunion

Answer 3: ORIF is associated with improved cosmetic results compared to non operative measures

Answer 5: Range of motion after ORIF is comparable to that of non operative treatment

 

 

 

 

 

OrthoCash 2020

 

  1. In each of the following scenarios, atrophic fracture nonunion occurred after initial treatment with intramedullary nail fixation. Which scenario has shown to have the highest rate of osseous union if treated with exchange intramedullary nailing?

    1. Oligotrophic nonunion of a comminuted humeral shaft fracture

    2. Oligotrophic nonunion of a transverse humeral shaft fracture

    3. Oligotrophic nonunion of an oblique distal femur fracture

    4. Oligotrophic nonunion of a comminuted tibial shaft fracture

    5. Oligotrophic nonunion of an oblique tibial shaft fracture Corrent answer: 5

    Reamed exchange nailing is recommended for the management of aseptic nonunions of noncomminuted tibial shaft fractures. Union rates have been reported between 76-96% in large studies.

     

    Tibial exchange nailing promotes osseous bone healing of non-unions by providing biological and mechanical support. The biological support is provided by reaming the medullary canal. This increases periosteal blood flow and stimulates periosteal new-bone formation. The mechanical support is provided by a larger-diameter intramedullary nail, which increases the rigidity and strength of the nail.

     

    Brinker et al. reviewed the concept of exchange nailing of nonunited long bone fractures. They showed that exchange nailing is the most successful in the treatment of nonunions following closed or open fractures without substantial bone loss. Aseptic, noncomminuted diaphyseal femoral and tibial shaft fractures showed the highest rates of union with exchange nailing, which were found to be 76-100% and 72-96%, respectively.

     

    Illustration A shows a heterotrophic non-union of the tibia after intramedullary nailing. The patient was treated with exchange nailing with a larger nail. On the right shows a 4 month post-op radiograph after exchange nailing showing osseous union at the fracture site.

     

    Incorrect Answers:

    Answer 1, 2: Exchange nailing has not produced good results following failures of intramedullary nail fixation of humeral shaft fractures. Union rates have been shown between 30-40%,

    Answer 3: There is little evidence to support exchange nailing for distal femoral nonunions. Union rates have been shown at 50-60%.

    Answer 4: Nonunions following comminuted fractures do not appear to respond

    to exchange nailing. Union rates have been reported as 45-80%.

     

     

     

     

     

     

    OrthoCash 2020

     

  2. Figures A to C show radiographs of a 70 year-old male who fell on the ice 5 days ago. He could initially walk after the fall, but he is now unable to weight-bear on the leg due to pain. Physical examination shows diffuse ankle swelling, with no medial sided ankle tenderness. He is neurovascularly intact. The patient does not smoke and has no medical history. A gravity stress radiograph is demonstrated in Figure

    D. After closed reduction, there is 2 mm of fracture displacement, no talar shift and the talocrural angle = 83 degrees. What is the most appropriate treatment?

     

     

     

     

     

     

    1. Total contact casting, non-weightbearing, for 6-8 weeks.

    2. MRI to better evaluate the deep deltoid ligament

    3. Open reduction and internal fixation

    4. CT scan to better identify the fracture pattern

    5. Below knee splint for 2-4 weeks and advance to weight-bearing as tolerated in an immobilizer boot

    Corrent answer: 5

     

    The patient has a 5 day old Weber B fibula fracture with a well reduced mortise and stress radiograph that demonstrates no instability of the mortise. The appropriate treatment would be a period of immobilization followed by advanced weight-bearing as tolerated.

     

    The patient has an isolated lateral malleolus fracture. These fracture are stable if the deltoid ligament is intact, and unstable if the deltoid ligament is disrupted. The competency of the deltoid ligament can be evaluated with a stress radiograph which is completed by placing the ankle in maximum dorsiflexion and 15° of internal rotation while an external rotation force is applied to the foot. A stress radiograph (manual or gravity assisted) is positive if there is subluxation of the talus during the exam. The presence of tenderness or swelling medially at the ankle has been shown to be a poor indicator of medial-sided injury. The clinical utility of MRI scans in ankle fractures is controversial.

     

    Koval et al. reported on 21 patients with 5-8 mm of clear space widening on stress radiographs after an isolated Weber B lateral malleolus fracture. They performed MRI's of all ankles and found 90% had at least a partially intact deep deltoid. The patients with a partially intact deep deltoid were treated with weight-bearing as tolerated in a removable boot. 14/15 patients that were available for one year follow-up had an AOFAS score of 100.

     

    Egol et al. reported on the treatment of 30 patients with an isolated fibula fracture and a stress XR with over 4mm of clear space widening, but no signs of medial injury such as tenderness, ecchymosis, or swelling around the medial malleolus on physical exam. Twenty patients were treated without surgery, and 10 patients were treated with surgery. The average AOFAS score was similar in both groups at final follow up.

     

    Figure A demonstrates an AP radiograph of a Weber B fibula fracture with minimal medial clear space widening.

    Figure B demonstrates a mortise radiograph of a Weber B fibula fracture with a congruent mortise.

    Figure C demonstrates a lateral radiograph of a Weber B fibula fracture with a congruent mortise.

    Figure D demonstrates a stress radiograph with minimal medial clear space widening.

     

    Incorrect Answers:

    Answer 1: Total contact cast (TCC) is an effective treatment in neuropathic, noninfected, and nonischemic foot ulcers. There is no indication for TCC in this

    patient.

    Answer 2: The clinical utility of MRI scans in ankle fractures is controversial. Answer 3: The patient has a stable ankle fracture. The post-reduction parameters are within normal limits - intact mortise, no talar shift, < 3mm displacement, talocrural angle = 83 +/- 4 degrees.

    Answer 4: A CT scan would be indicated for complex intra-articular fractures, loose bodies, joint impaction, etc. There is no indication in this patient.

     

     

     

    OrthoCash 2020

     

  3. A 44-year-old male presents after being struck by a car. He is hypotensive, and has an obvious open tibia and a closed femoral shaft fracture. A pelvic radiograph is obtained and shown in Figure A. A representative CT scan image in shown in Figure B. The patient is placed in a pelvic binder, and his blood pressure normalizes with crystalloid and a blood transfusion. He then undergoes pelvic angiography, and his internal iliac artery is embolized. Which of the following definitive treatment options is most appropriate in this case?

     

     

     

     

     

     

    1. Anterior pelvic external fixation with pins placed in the AIIS and a posterior sacroiliac screw

    2. Open reduction and internal fixation of the pubic symphysis with a two hole plate and posterior fixation with a iliosacral screw

    3. Open reduction and internal fixation of the pubic symphysis with a nonlocking multi-hole plate and posterior fixation with a iliosacral screw

    4. Anterior pelvic external fixation and posterior triangular osteosynthesis

    5. Open reduction and internal fixation of the pubic symphysis with a locked multi-hole plate and posterior sacral plate fixation

    Corrent answer: 3

     

    The radiographs and CT scan show a fracture pattern consistent with an APC-III injury. This is most appropriately treated with pubic symphysis ORIF with a multi-hole plate and posterior iliosacral screw fixation.

     

    APC-III pelvic ring injuries are associated with with disruption of the anterior and posterior SI ligaments (SI dislocation) as well as disruption of

    sacrospinous and sacrotuberous ligaments. They are commonly associated with vascular injury and retroperitoneal bleeding.

     

    Sagi et al studied the radiographic and clinical outcome of symphyseal plating techniques, specifically comparing two hole (THP) vs multi-hole plating (MHP). Retrospective review of charts and radiographs immediately after the index procedure to latest follow-up was performed. When comparing the 2 different methods of anterior fixation, they found that the rate of fixation failure was greater in group THP (33%) than group MHP (12%). When evaluating the presence of a malunion as a result of these 2 treatment methods, there were more present in the THP group (57%) than in the MHP group (15%). On the basis of these findings, the authors recommended multi-hole plating of unstable pubic symphyseal disruptions.

     

    Grimshaw et al performed a biomechanical study to determine the failure risk and potential benefit to use of locked fixation constructs in simulated pelvic ring injuries. The authors used a six-hole 3.5-mm plate specifically designed for the symphysis pubis with the capability of fixation in locked or unlocked mode, six pelves were fixed with locked screws and six pelves were fixed standard unlocked bicortical screws. Biomechanical testing was performed, and no abrupt failures were noted. However, locked plating of the pubic symphysis did not appear to offer any advantage over the standard unlocked technique

     

    Illustrations A, B, and C show the AP, outlet, and inlet postoperative radiographs.

     

    Incorrect Answers:

    1,4-Anterior pelvic fixation is typically not used as definitive treatment of these injuries unless performed in a damage-control setting. The patient in this case is stable.

    2-Multi-hole plate fixation of the pubic symphysis is preferred over 2-hole plate fixation.

    5-Posterior sacral plating is not indicated in this fracture pattern, as there is no significant sacral comminution of vertical shear component. Furthermore, no distinct advantage has been found with locked symphyseal plating.

     

     

     

     

     

     

     

     

     

     

    OrthoCash 2020

     

  4. Figure A is a radiograph of an 80-year-old woman who sustained a closed injury to her left arm 10 months ago. She presents to office today complaining of persistent pain in her arm. What is the most likely metabolic or endocrine abnormality contributing to this patients presentation?

     

     

     

    1. Vitamin D deficiency

    2. Central hypogonadism

    3. Thyroid disorder

    4. Parathyroid hormone disorder

    5. Calcium deficiency

     

    Corrent answer: 1

     

    Figure A is a radiograph showing oligotrophic nonunion of a proximal third humeral shaft fracture. The most likely underlying metabolic or endocrine abnormality with this presentation is vitamin D deficiency.

     

    The etiology of fracture nonunion is multifactorial. Predisposing factors include: mechanical instability, poor bone-to-bone contact, infection and biological/biochemical factors. A variety of other contributing factors, such as cigarette smoking and malnutrition, have also been described. Correctable metabolic or endocrine abnormalities are common in elderly patients and are considered contributory in the etiology of fracture non-union. Correction of these abnormalities should be considered prior to surgical fixation.

     

    Brinker et al. studied 37 patients with fracture nonunions with identifiable metabolic or endocrine abnormalities. The distribution of nonunion types at the 46 sites were: oligotrophic (23), atrophic (12), infected (7), and hypertrophic

    (4). They showed that 85% of patients had a previously undiagnosed metabolic or endocrine abnormality. The most common newly diagnosed abnormality was vitamin D deficiency in 25 of the 37 patients (68%).

     

    Kuo et al. studied 155 consecutive minimal-trauma fracture subjects for metabolic abnormalities with bone mineral density and laboratory testing. At baseline, only 26% had had proper workup for metabolic abnormalities. After diagnostic work-up, 44% of patients were recommended for anti-resorptive therapy and 56% were recommended calcium/vitamin D medication.

     

    Illustrations A and B show open reduction internal fixation of Figure A with a proximal humeral plate.

     

    Incorrect Answers:

    Answer 2: Studies show 15% of fracture nonunions can be attributed to undiagnosed central hypogonadism.

    Answer 3: Studies show 24% of fracture nonunions can be attributed to previously established or undiagnosed thyroid disorders

    Answer 4: Studies show 11% of fracture nonunions can be attributed to previously established or undiagnosed parathyroid disorders

    Answer 5: Studies show 35% of fracture nonunions can be attributed to lack of calcium.

     

     

     

     

     

     

     

     

    OrthoCash 2020

     

  5. Which of the following fracture patterns is classically associated with varus malunion if treated with closed reduction and casting?

     

     

     

     

     

     

     

     

     

     

     

     

     

    1. Figure A

    2. Figure B

    3. Figure C

    4. Figure D

    5. Figure E

     

    Corrent answer: 2

     

    Figure B shows a midshaft tibia fracture with an intact fibula, which places the fracture at increased risk of varus malalignment during healing.

     

    Nonsurgically treated tibial shaft fractures with an intact fibula have an increased risk of varus malunion when treated nonsurgically. The fibula acts as a strut, preventing valgus collapse, but predisposing to varus angulation. Limb-length discrepancies can also occur.

     

    Teitz et al. found that 26 percent of patients over the ago of 20 years old with isolated tibial shaft fractures treated nonsurgically went on to varus malunion. They also found that when the fibula remains intact, a tibiofibular length discrepancy can develop and cause altered strain patterns in the tibia and fibula leading to later ipsilateral joint sequeale.

    Bonnevialle et al performed a retrospective study to assess the outcome after reamed nailing of tibial fractures with an intact fibula. Thirty-eight fractures were evaluated at a minimum of one year from surgery. Healing was achieved after the first intention treatment in 30 patients, after dynamization in 6. A non-union in 2 patients was also successfully managed with new nailing and dynamization. The authors concluded that nailing is a reliable technique for the treatment of tibial fractures with an intact fibula.

     

    Sarmiento et al reviewed 1,000 consecutive closed diaphyseal tibial fractures, treated with prefabricated functional below-knee braces, to determine factors predictive of final fracture outcome. They found that final angulatory deformity in any plane was < or = 6 degrees in 90% of patients. They also noted that the presence of an intact fibula was a relative contraindication for functional fracture bracing because angulatory deformity was more likely to develop.

    Figure B shows a midshaft tibial shaft fracture with an intact fibula. Incorrect Answers:

    Answer 1: Figure A shows a distal tibia and fibula fracture which are not

    specifically prone to varus malunion. Rather, if treated with IM nailing, fracture alignment is dictated by placement of the guide-wire and path of reaming which must be center-center distally.

    Answer 3: Figure C shows a midshaft fibula and tibia fracture at the same level. The fibula, in this case, would not predispose the fracture specifically to varus malalignment.

    Answer 4: Figure D shows a proximal tibial shaft and associated fibula fracture. This fracture pattern is predisposed to valgus and procurvatum.

    Answer 5: Figure E shows a segmental tibial shaft fracture which are prone to shortening, translational, and rotational malalignment.

     

     

     

    OrthoCash 2020

     

  6. A 55-year-old female presents to the emergency room after falling off her balcony. She sustained the isolated, closed injury shown in Figures A and B. She is otherwise healthy, but routinely smokes 30 cigarettes per day. What would be the most appropriate sequence of treatment steps for definitive management of this injury?

     

     

     

     

     

     

    1. Closed reduction and splinting followed by delayed casting

    2. Immediate open reduction internal fixation

    3. Closed reduction and splinting, CT scan, and immediate open reduction internal fixation

    4. Closed reduction and splinting, CT scan, external fixation, delayed open reduction internal fixation

    5. Closed reduction and splinting, external fixation, CT scan, delayed open reduction internal fixation

    Corrent answer: 5

     

    Figures A and B show radiographs of a comminuted AO/OTA C3 fracture of the distal tibia. The most appropriate steps in the management of this patients injury would include: Closed reduction and splinting, external fixation, CT scan, and delayed open reduction internal fixation in this sequence.

     

    Infection and wound healing problems are common with comminuted distal tibia fractures due to an increased incidence of soft-tissue complications associated with the operative management. Patients with a history of diabetes and smoking are at further risk. To decrease the risk of complications in this patient, a two-staged, delayed open reduction internal fixation technique is the best option for the treatment of severe pilon fractures.

     

    Patterson et al. reviewed 23 consecutive patients with comminuted distal tibia fractures. They showed 0% infections or wound-healing problems in their patient population treated with a two-staged protocol. Their protocol involved fibula fixation with an intramedullary implant and application of a medial external fixator to to regain length and restore anatomic alignment. Re-evaluation of the limb occurred ten to fourteen days later for definitive fixation.

     

    Sirkin et al. retrospectively reviewed 40 closed and 82 open pilon fractures (AO types 43A-C) that were treated with staged surgical management (avg. time from ext. fix. to formal reconstruction was 14 days (range 4 to 31) They reported 17% post-operative wound complication in the closed group and 11% post-operative wound complication in the open group (Gustilo Type I-III). They suggest the technique was successful in both closed and open pilon fractures.

    Illustration A shows open reduction internal fixation of the above injury. Incorrect Answers:

    Answer 1: Universally, nonoperative treatment of OTA/AO C3 fractures has led

    to poor outcomes.

    Answer 2,3: Although Ruedi and Allgower (1969) were the first to report successful results with primary open reduction and internal fixation of low-energy fracture patterns, few studies have achieved good results with low soft-tissue complications.

    Answer 4: CT scan is most appropriate after frame application. This will allow for distraction and indirect reduction of the fragments to better characterize the fracture pattern for pre-operative planning.

     

     

     

     

     

    OrthoCash 2020

     

  7. When using a trochanteric femoral nail, placing your start point in which area will result in varus deformity?

     

     

     

    1. A

    2. B

    3. C

    4. A and B

    5. B and C Corrent answer: 1

    Placing your start point lateral to the tip of the greater trochanter (location A) will cause varus deformity.

     

    When using a trochanteric entry femoral nail, the ideal starting point is just medial to the tip of the greater trochanter. Placement lateral to the tip of the trochanter will cause varus malalignment.

     

    Ostrum et al. in a cadaveric study, reported on deformities following medial, standard, and lateral starting points for trochanteric entry femoral nails. Nails inserted too medially yielded excessive valgus, while nails inserted too laterally yielded varus malalignment. Those started at the tip yielded the most neutral alignment, but still with slight varus. The authors recommended a start point just medial to the tip of the greater trochanter.

    Figure A exhibits ideal starting point for a trochanteric entry nail (B), for a piriformis start nail (C) and a start point that is too lateral, causing varus (A). Illustration A depicts 4 different types of trochanteric entry nails that started at the tip of the greater trochanter and although mostly neutral, still exhibit slight varus.

     

    Incorrect answers:

    Answer 2. Ideal starting point for a trochanteric entry nail resulting in neutral alignment.

    Answer 3. Ideal starting point for a piriformis entry nail or valgus in a trochanteric entry nail.

    Answer 4. All of the choices will yield different results. Answer 5. The appropriate answer choice is A.

     

     

     

     

     

     

    OrthoCash 2020

     

  8. A 20-year-old male is taken to the emergency department following a motorcycle collision with the injury seen in Figures A and

    B. He undergoes serial debridements with placement of an antibiotic bead pouch, followed by intramedullary nailing and free tissue transfer. His preoperative examination is notable for absent plantar sensation. Which of the following is predictive of a worse long term outcome in this patient?

     

     

     

     

     

     

    1. Absent plantar sensation at presentation

    2. Depression at 3 months post-injury

    3. Use of an intramedullary nail

    4. Free tissue transfer instead of rotational flap

    5. Need for multiple debridements Corrent answer: 2

    The presence of depression at the 3 months post-injury is a significant predictor of worse long term outcome as well as lower return to work rates in patients that present with limb threatening lower extremity trauma.

     

    The findings of the Lower Extremity Assessment Project (LEAP study) has challenged many of the long-held beliefs of lower extremity trauma surgery, including the concept that patients with absent plantar sensation should undergo immediate amputation . The LEAP study has also brought to light the importance of socioeconomic and psychosocial factors in patient outcomes following these injuries.

    Bosse et al analyzed a subgroup of 55 patients from the LEAP study with absent plantar sensation at presentation. Those patients that underwent limb salvage despite absent sensation (n=26) had no significant difference in functional outcome when compared with either insensate patients who underwent amputation or a matched control group of sensate patients with similar injuries. All but one patient in the salvage group had return of some sensation at 2-year follow up and the authors concluded that an insensate plantar foot at presentation should not be included in the algorithm for amputation.

     

    Mackenzie and Bosse reviewed the results of the LEAP study with particular emphasis on the social and economic variables that influence outcomes in severe lower extremity. The authors note that 19% of patients screened positive for severe depression. The presence of anxiety, pain and depression at 3 months post-injury were significant predictors of poor outcome

     

    Incorrect Answers:

    Answer 1: Absent plantar sensation does not predict worse outcomes and patients may have significant return of sensation at 2-year follow up.

    Answer 3: No difference has been demonstrated with methods of internal fixation for type IIIB tibia shaft fractures.

    Answer 4: No difference has been shown with the use of free tissue versus rotational tissue for type IIIB tibia fractures.

    Answer 5: The number of debridements has not been shown to influence outcome.

     

     

     

    OrthoCash 2020

     

  9. A 45-year-old male is involved in a motorcycle crash. Work-up reveals a closed, right femoral shaft fracture, an open, right tibial shaft fracture, a closed left mid-shaft radius and ulna fractures, and a closed left femoral shaft fracture. His heart rate is 115bpm with a systolic blood pressure of 105mmHg. Initial hematocrit values are reported at

    31.5 g/dL and his base deficit is 10mEq/L. What is the most appropriate next step in management?

    1. Reamed intramedullary nailing bilateral femur fractures, irrigation and debridement followed by reamed intramedullary nailing right tibia, open reduction internal fixation left forearm

    2. Reamed intramedullary nailing bilateral femur fractures, irrigation and debridement followed by external fixation right tibia, splinting left forearm

    3. External fixation bilateral femur fractures, irrigation and debridement followed by external fixation right tibia, open reduction internal fixation left

      forearm

    4. External fixation bilateral femur fractures, irrigation and debridement followed by external fixation right tibia, splinting left forearm

    5. External fixation right femur and right tibia following irrigation and debridement, reamed intramedullary nailing left femur, splinting left forearm

    Corrent answer: 4

     

    With a base deficit of 10mEq/L, the patient is under-resuscitated and unstable. Thus, damage control orthopedics via external fixation of the long bone injuries with irrigation and debridement of the open tibia is the appropriate next step in management.

     

    Of all of the reported values, the most important predictor of morbidity and mortality is the base deficit (normal range -2 to +2mEq/L), which represents overall resuscitation status. Another representative parameter of resuscitation status is lactate (normal <2mg/dL). Heart rate, blood pressure and hematocrit are not reliable predictors of normalized resuscitation status, morbidity or mortality.

     

    Callaway et al. retrospectively reviewed a large cohort of blunt trauma patients over a 6 year period. Only base deficit and lactate levels were directly correlated with and were reliable predictors of mortality.

     

    Paladino et al. retrospectively reviewed a prospective database of over 1400 patients. Base deficit and lactate were significant and useful predictors of triage upon initial presentation to denote severe versus non-severe injury.

     

    Martin et al. retrospectively analyzed over 2000 sets of laboratory data in 427 ICU patients. Base deficit (anion status), even in ICU patients with normal lactate levels, were predictive of decreased survival.

     

    Incorrect Answers:

    Answer 1: Represents ultimate, definitive fixation. However, in an unstable patient, this should not be the next step in management.

    Answer 2, 3 and 5: When employing damage control orthopedics, temporary stabilization should include external fixation of all long-bone injuries and splinting of upper extremity injuries.

     

     

     

    OrthoCash 2020

  10. A 26-year-old male sustains an elbow injury after a fall from a skateboard resulting in valgus and supination forces across the left elbow. A CT scan of the left elbow is shown in Figures A through D. This fracture pattern is most commonly associated with what other traumatic elbow pathology?

     

     

     

     

     

     

     

     

     

     

     

     

    1. Posteromedial rotatory instability

    2. Capitellum fracture

    3. Radial head fracture and posterolateral ulnohumeral dislocation

    4. Trans-olecranon fracture dislocation

    5. Medial (ulnar) collateral ligament rupture Corrent answer: 3

    The clinical presentation is consistent with a coronoid tip fracture. This fracture pattern is associated with a radial head fracture and posterolateral ulnohumeral dislocation - together making up the terrible triad injury.

    A terrible triad injury is the result of a valgus and supination injury and involves posterolateral elbow dislocation or lateral collateral ligament injury, radial head fracture, and fracture of the coronoid process. The elbow may dislocate postero-laterally with the anterior bundle of the MCL intact, but if the MCL is injured it is typically the last structure to fail. The coronoid fracture is typically a small fragment isolated to the tip. This is a result of a posteriorly directed force driving the coronoid into the trochlea prior to posterior elbow dislocation. CT scan is a useful modality when small or comminuted fragments are difficult to visualize on plain radiographs.

     

    Steinmann reviews the anatomy, diagnosis, classification and treatment of coronoid fractures with a focus on surgical exposures and fixation techniques.

     

    Doornberg et al. reviewed 67 coronoid fractures to determine whether type of coronoid fracture correlated with pattern of instability. They found strong associations between (1) large coronoid fractures and trans-olecranon fracture-dislocations, (2) small fractures and terrible-triad injuries, and (3) anteromedial facet fractures and varus posteromedial rotational injury mechanisms.

     

    Doornberg et al. evaluated 18 patients with a fracture of the anteromedial facet of the coronoid. They found that malalignment of the anteromedial facet fragment was associated with arthrosis and a fair or poor result.

     

    Figures A through D show consecutive 2.00 mm sagittal CT reformats demonstrating a small coronoid fracture fragment which was addressed with suture fixation.

     

    Incorrect Answers:

    Answer 1. Coronoid anteromedial facet fracture (not tip) in conjunction with LCL disruption can cause posteromedial rotatory instability. Coronoid tip fractures may be associated with posteroLATERAL rotatory instability Answer 2. Capitellum fractures are not associated with coronoid fractures Answer 4. Trans-olecranon fracture dislocations are associated with larger coronoid fracture fragments.

    Answer 5. MCL injury is associated with valgus instability but not coronoid fracture

     

     

     

    OrthoCash 2020

     

  11. A 62-year-old right-hand-dominant school teacher sustains a mechanical fall at home and presents with right shoulder pain. Plain

    radiographs of the right shoulder are pictured in Figures A and B. The patient asks you what she can expect in terms of recovery following this injury. Which of the following is the most appropriate statement?

     

     

     

     

     

     

     

    1. At 1-year post-injury, the right shoulder range of motion will most likely be equal to the contralateral extremity.

    2. At 1-year post-injury, you will most likely have returned to your baseline functional status.

    3. Early range of motion exercises risk fracture displacement and should be avoided until at least 4 weeks post-injury.

    4. Most patients do not return to work following this injury.

    5. One in 5 patients with this fracture go on to nonunion and you may benefit from surgery in the future to address this.

    Corrent answer: 2

     

    This patient has a minimally displaced (1-part) proximal humerus fracture involving the humeral neck and greater tuberosity. This injury pattern is most commonly managed nonoperatively with the majority of patients returning to their baseline functional status by 1 year.

     

    Proximal humerus fractures (PHF) can be classified by number of parts (Neer classification), with a part defined as a fracture fragment displaced > 1cm (> 5mm for greater tuberosity) or angulated > 45°. One-part PHF comprise ~80% of all PHF and are treated nonoperatively with a sling and early range of motion (ROM).

     

    Tejwani et al performed a prospective study of 67 patients with 1-part PHF. At 1-year follow up the ASES score and functional status was similar to pre-injury status. However, ROM of the affected shoulder was diminished in both external and internal rotation. Forward flexion was preserved.

     

    Hanson et al prospectively analyzed 160 patients with PHF of all types (1-4 parts and head-splitting) managed nonoperatively. At 1-year follow up, 93% showed solid union. Constant and DASH scores improved steadily over time but were still lower compared to the contralateral extremity. Of employed patients, 97.6% returned to work with a median time off of 10 weeks and no difference between manual and nonmanual workers.

     

    Figures A and B are the AP and axillary radiographs of the right shoulder, respectively, demonstrating a 1-part PHF involving the humeral neck and greater tuberosity.

     

    Incorrect Responses:

    Answer 1: At 1-year follow up, most patients have restored forward flexion but lack both internal and external rotation compared to the contralateral extremity.

    Answer 3: Early ROM exercises starting at 2 weeks post-injury have not been shown to result in fracture displacement. Patients managed with early rehabilitation exhibit high union rates and largely return to baseline functional status.

    Answer 4: The vast majority of patients with PHF of all types return to work post-injury.

    Answer 5: One-part PHF have an excellent union rate (>90%) and rarely require surgery.

     

     

     

    OrthoCash 2020

     

  12. A 44-year-old male presents with the isolated injury seen in Figure A after a motor vehicle accident and underwent the operative treatment seen in Figure B within 8 hours from the time of incident. Which of the following complications is this patient at highest risk of developing?

     

     

     

     

     

     

     

    1. Pulmonary embolus

    2. Periprosthetic fracture

    3. Contralateral hip fracture

    4. Osteonecrosis

    5. Infection

     

    Corrent answer: 4

    This young male patient has sustained a displaced femoral neck fracture and underwent open reduction internal fixation with 3 cannulated screws. Based on the available options, the patient is most at risk of developing osteonecrosis of the femoral head.

     

    Femoral neck fractures in young patients typically are the result of a high-energy trauma. Fracture displacement has been shown to disrupt vascular supply to the femoral head by interrupting retinacular vessels and ligament teres vascularization, as well as increasing intracapsular pressure, producing a tamponade effect. The incidence of osteonecrosis in patients younger than 60 years with displaced femoral neck fractures has been shown to be between 15-30%. Quality of reduction is one key factor that has been shown to influence outcomes postoperatively.

     

    Loizou et al. prospectively studied 1,023 patients who sustained an intracapsular hip fracture that was treated with internal fixation using standard fixation modalities. They showed that osteonecrosis was less common for undisplaced (4.0%) than for displaced fractures (9.5%). The population at greatest risk were women younger than the age of 60 with displaced fractures.

     

    Barnes et al. review subcapital hip fractures. They found that late segmental collapse was more common in displaced fractures in women younger than age 75 years than in those older than age 75 years treated with internal fixation.

     

    Figure A shows a displaced, Garden 3/Pauwels III hip fracture. Figure B shows anatomical fixation with 3 cannulated screws.

     

    Incorrect Answers:

    Answer 1: The incidence of pulmonary embolus after hip fracture is <1%. Answer 2: The incidence of periprosthetic fracture is <5%.

    Answer 3: The incidence of contralateral hip fracture is minimal in this patient. Answer 5: The rate of infection after hip fracture is <5%.

     

     

     

    OrthoCash 2020

     

  13. A 58-year-old male is involved in a motor vehicle collision and sustains the injury shown in Figure A in addition to right 5th and 6th rib fractures. Upon evaluation in the emergency department, he is noted to have a 2 centimeter laceration over the anterior aspect of his left leg with visible bone. Vitals and labs are normal. Which of the following statements is most accurate regarding surgical management for this patient?

     

     

     

    1. Reamed intramedullary nailing is favored due to increased rates of union

    2. Unreamed intramedullary nailing is favored due to presence of concomitant rib fractures

    3. Reamed intramedullary nailing is favored due to decreased rates of infection

    4. Unreamed intramedullary nailing is favored due to less local trauma

    5. Both unreamed and reamed intramedullary nailing are equivalent Corrent answer: 5

    Both unreamed and reamed intramedullary nailing are equivalent treatments in patients with open tibia fractures. Intramedullary nailing is the treatment of choice for stable patients with tibial shaft fractures.

     

    Tibial shaft fractures can be the result of low energy twisting injuries or higher energy axial loads. Closed fractures with acceptable alignment can be often be treated with closed reduction and casting. Intramedullary nailing, unreamed or reamed, is the treatment of choice for open fractures except in the setting of damage control orthopaedics when an external fixator may be more appropriate.

     

    Bhandari et al. investigated reamed and unreamed intramedullary nailing for tibial shaft fractures in a randomized trial ("SPRINT" Trial - Study to Prospectively Evaluate Reamed Intramedullary Nails in Patients with Tibial Fractures Investigators). They concluded that reamed nailing was more beneficial (decreased rate of primary outcome event: need for bone grafting, implant exchange or removal for infection, debridement for infection) for closed fractures, but had no benefit in open fractures.

     

    Finkemeier et al. evaluated consecutive patients treated with unreamed and reamed intramedullary nailing and found similar rates of union in both open and closed tibial shaft fractures at six and twelve months.

    Figures A shows AP and lateral xrays of the left tibia showing a tibial shaft fracture.

     

    Incorrect Answers:

    Answer 1-4: Unreamed and reamed intramedullary nailing for open tibia fractures are similar in terms of union rates, infection rates, and rates of reoperation. There is a theoretical risk of pulmonary complications with reaming but it has not been proven.

     

     

     

    OrthoCash 2020

     

  14. A 36-year-old male falls from a 10-ft scaffold and suffers the injuries shown in Figures A and B. The patient is placed in a spanning external fixator and brought back to the operating room once his soft tissues are amenable. Planning to use a dual-incision approach, what is the correct interval to use when approaching the medial side?

     

     

     

     

     

    1. Popliteus and pes anserine

    2. Lateral head of the gastrocnemius and pes anserine

    3. Politeus and lateral head of the gastrocnemius

    4. Iliotibial band and medial head of the gastrocnemius

    5. Pes anserine and medial head of the gastrocnemius Corrent answer: 5

    The posteromedial approach to the tibial plateau is between the the pes anserine tendons and the medial head of the gastrocnemius.

     

    A dual-incision approach is often utilized to optimally place definitive fixation for bicondylar tibial plateau fractures. For fractures that require posterior or posteromedial fixation, the correct interval is between the pes anserine and the medial head of the gastrocnemius.

     

    Higgins et al. in a large cohort morphological review, noted a high incidence of a posteromedial fragment in bicondylar fractures. Occurring at a high frequency, the authors recommended direct visualization and reduction via a dual approach rather than using indirect reduction techniques.

     

    Falker et al. describes a step-by-step approach to utilizing the posteromedial approach for the tibial plateau and placing an anti-glide plate.

     

    Figure A and B exhibit a bicondylar tibial plateau fracture with a posteromedial fragment noted on the lateral x-ray. Illustration A exhibits the surrounding anatomy and interval in between the medial head of the gastrocnemius and the pes anserine.

     

    Incorrect answers:

    Answers 1-4. These intervals are not the correct intervals utilized in a posteromedial approach to the tibial plateau.

     

     

     

     

     

    OrthoCash 2020

     

  15. A 25-year-old male presents to the emergency department with the injury seen in Figure A after a motorcycle collision. The patient has a blood pressure of 70 systolic, elevated lactate and a tense abdomen with positive FAST examination. Trauma surgery will be performing an emergent laparotomy. Orthopaedic surgery is consulted and places a pelvic external fixator intraoperatvely to assist with resuscitation. What is an advantage of supra-acetabular external fixator pins as compared with iliac crest pins?

     

     

     

    1. Less interference with pelvic surgical incisions

    2. Less risk of pin tract infection

    3. Less risk of malreduction

    4. Less control of posterior pelvic ring

    5. No interference with laparotomy Corrent answer: 1

    One advantage of supra-acetabular external fixator pins is that they do not interfere or contaminate future approaches to the pelvis or acetabulum involving the lateral window.

     

    In multiply injured patients with pelvic trauma external fixation of the pelvic ring is a valuable tool to assist with resuscitation. Pelvic external fixation should be applied rapidly and allow full access to the abdomen for general surgery intervention. Regardless of the technique used, a pelvic external fixator should form a stable construct that minimizes motion of fracture surfaces and allows for clot formation.

     

    Haidukewych et al evaluated the safety of supra-acetabular pin placement in a cadaveric study. The authors found that the lateral femoral cutaneous nerve (LFCN) was most at risk during pin placement.

     

    Figure A demonstrates a widely displaced symphyseal dislocation with associated bilateral sacroiliac (SI) dislocations (APC 3). Illustration A demonstrates an outlet radiograph of a supra-acetabular external fixtator in conjunction with posterior pelvic ring fixation for an LC3 pelvic ring injury.

    Illustration B is an illustration of iliac crest external fixation. The video demonstrates techniques for application of both supra-acetabular and iliac

    crest external fixation pins.

     

    Incorrect Answers:

    Answer 2: There has not been a difference demonstrated with pin tract infection between supra-acetabular and iliac crest pins

    Answer 3: Both techniques have a risk of malreduction

    Answer 4: Theoretically supra-acetabular pins may better control the posterior ring as they may be advanced to the posterior ilium (PSIS) if placed appropriately

    Answer 5: Neither type of pin placement or frame should interfere with laparotomy

     

     

     

     

     

     

     

     

     

    OrthoCash 2020

  16. What physical exam finding is most likely to be found in association with the injury shown in Figures A and B?

     

     

     

     

    1. Numbness in the small finger and ulnar side of the ring finger

    2. No elbow instability

    3. Varus posteromedial rotatory instability

    4. Valgus posterolateral rotatory instability

    5. An anterior open wound Corrent answer: 3

    The x-ray shows a fracture of the anteromedial facet of the coronoid with an intact radial head. Large anteromedial facet fractures are associated with varus posteromedial rotatory instability.

     

    The anteromedial facet of the coronoid provides support to the medial elbow against varus stress. Varus and posteromedial force applied to the elbow results in disruption of the lateral collateral ligament (LCL) from its proximal origin. The coronoid is fractured as it is forced against the medial trochlea.

    Coronoid fractures of significant size involving the sublime tubercle (insertion of medial collateral ligament) result in varus instability.

     

    Steinmann reviews the anatomy, diagnosis, classification and treatment of coronoid fractures with a focus on surgical exposures and fixation techniques. He states that when a coronoid fracture is associated with a pattern of varus instability, it requires fixation with either suture, buttress plating or screw fixation. Concomitant LCL repair or reconstruction will also be necessary.

     

    Doornberg et al. reviewed 67 coronoid fractures to determine whether type of coronoid fracture correlated with pattern of instability. They found strong

    associations between (1) large coronoid fractures and trans-olecranon fracture-dislocations, (2) small fractures and terrible-triad injuries, and (3) anteromedial facet fractures and varus posteromedial rotational injury mechanisms.

     

    Doornberg et al. evaluated 18 patients with a fracture of the anteromedial facet of the coronoid. They found that malalignment of the anteromedial facet fragment was associated with arthrosis and a fair or poor result.

     

    Figure A is an AP view of an elbow with an anteromedial facet of the coronoid fractured. The lateral joint space is widened due to injury to the LCL. The medial joint space is narrowed and collapsed. A lateral view is shown in Figure

    B. Illustrations A and B show AP and lateral views of a coronoid fracture fixed with buttress plating. The LCL origin was fixed with a suture anchor. Illustration C shows the O'Driscoll classification of coronoid fractures. Illustration D lists injury patterns that suggest posteromedial versus posterolateral rotatory instability.

     

    Incorrect Answers:

    Answer 1. Although possible, ulnar nerve symptoms are not characteristic of this injury and would not be as likely as elbow instability in the non-dislocated elbow shown in the radiograph

    Answer 2. Although not dislocated, the AP radiograph shows signs of subluxation and instability (lateral joint space widening, medial joint space collapse and large coronoid fragment).

    Answer 4. Valgus posterolateral instability is associated with very small coronoid fragments

    Answer 5. If open, the wound associated with these fractures is usually posterior

     

     

     

     

     

     

     

     

    OrthoCash 2020

     

  17. A 35-year-old man presents to the ED as the restrained driver of a high speed motor vehicle collision complaining of hip, chest, and abdominal pain. He becomes diaphoretic, tachycardic, and hypotensive in the trauma bay and is noted to have diminished lower extremity pulses. He is found on ATLS workup to have mediastinal widening.

    Which of the following injuries is most associated with thoracic aortic injury?

     

     

     

     

     

     

     

     

     

     

     

     

     

     

     

    1. Figure A

    2. Figure B

    3. Figure C

    4. Figure D

    5. Figure E

     

    Corrent answer: 3

     

    Thoracic aortic rupture is associated with posterior hip dislocation in deceleration trauma mechanism of injuries.

     

    Posterior hip dislocations are infrequently associated with local vascular injuries. With bilateral perfusion deficits, more proximal large vessel trauma should be considered, and in this situation, thoracic surgery should be involved emergently. Screening chest x-ray in the trauma bay should be reviewed for widened mediastinum, suggestive of aortic injury, as shown in illustration A. Given the high energy mechanism associated with these injuries, a full ATLS trauma survey must be done for every patient.

     

    Marymont et al. studies the association between posterior hip dislocation and thoracic aortic injury. They performed a retrospective chart review of 89 posterior hip dislocations and found 8% had an aortic injury. Although not statistically significant, they note the importance of evaluation for aortic injury in patients with posterior hip dislocations given its emergent life-threatening nature.

     

    In addition to associated chest injuries, Schmidt et al. highlight the importance of evaluating the ipsilateral knee after high-energy traumatic hip dislocation. In a prospective study, they identified a 93% rate of ipsilateral knee injury on MRI including effusion (37%), bone bruising (33%), and meniscal tear (30%) as the most common. They recommend a thorough exam but also expanded use of knee MRI after hip dislocation.

     

    Illustration A shows an example of chest x-ray with a widened mediastinum, suggestive of thoracic aortic injury.

     

    Answer choices A,B,D,E show other injury patterns: Answer A - Right femoral neck fracture

    Answer B - Right intertrochanteric hip fracture Answer D - Right subtrochanteric hip fracture

    Answer E - Right superior and inferior pubic rami fractures

     

     

     

     

     

    OrthoCash 2020

     

  18. A 31-year-old female smoker was involved in a skiing accident approximately 9 months ago and underwent open reduction internal fixation of the radius and ulna at the time of injury. She now returns to the clinic complaining of increasing pain with range of motion and activity. Radiographs from her most recent follow-up can be seen in Figure A. Laboratory tests show ESR, CRP and WBC count to be within normal limits. Which of the following options is the most appropriate next step in management?

     

     

     

    1. Bone scan

    2. Above elbow cast

    3. Removable splint

    4. Reamed intramedullary nail

    5. Iliac crest bone grafting + compression plating Corrent answer: 5

    This patient is presenting with an atrophic non-union of the ulna after open reduction internal fixation for a both bone forearm fracture 9 months ago. The most appropriate next step in management would be iliac crest bone grafting and compression plating of the ulna.

     

    The primary issue with an atrophic nonunion is biological. The blood supply is poor and therefore incapable of purposeful fracture healing. Smokers, as in this vignette, are at high risk for nonunion. The treatment of an atrophic nonunion involves improving biology at the fracture site through use of autologous bone graft (e.g. iliac crest) and providing mechanical stability by means of compression plating (e.g. 3.5 mm LC-DCP).

     

    dos Reis et al. reports excellent results of 31 cases of diaphyseal forearm fracture non-unions treated with autologous bone grafting and compression

    plating. Thirty of thirty-one patients went on to bony union within 3.5 months of revision surgery.

     

    Nadkarni et al. presented a case series of 11 patients with non-unions of various long bones initially managed with intradmedullary (IM) nail fixation. The authors successfully used locking compression plates while retaining the IM nails in the treatment of the nonunion in all cases.

     

    Figure A shows an AP radiograph of a both bone forearm fracture. Figure B shows an AP and lateral radiograph of an atrophic non-union of the ulnar shaft. Illustration A shows a lateral x-ray of a fully healed radius and ulna after hardware removal 1 year after revision surgery.

     

    Incorrect Answers:

    Answer 1: With normal WBC, ESR and CRP levels, there is little reason to suspect an infectious non-union, thus, there is no indication for a bone scan at this time.

    Answer 2 & 3: Immobilization will not treat the underlying issue of biology. Autologous bone grafting is necessary in the treatment of atrophic non-unions. Answer 4: A reamed IM nail does not provide adequate mechanical stability and is not used in the treatment of radius or ulna non-unions.

     

     

     

     

     

    OrthoCash 2020

     

  19. A 27 year-old patient sustains a fracture-dislocation of the acetabulum. Pelvic radiographs (Figures A and B) are taken at initial presentation and a CT scan (Figures C and D) is performed after reduction of the hip in the emergency room. What is the importance of the finding highlighted in the CT scan cuts?

     

     

     

     

     

     

     

     

     

     

     

    1. Comminution indicates a better result with non-operative management

    2. Significant marginal impaction could compromise the results of the surgical reduction if the joint surface is not properly restored

    3. The impacted fracture segment will heal without fixation because it is not gapped or translated

    4. The CT scan finding highlighted indicates osteochondral defects to the femoral head, which can be addressed arthroscopically

    5. Intraarticular fracture fragments should be discarded from the surgical field, as incorporation of the fragments into the fixation construct leads to a high rate of avascular necrosis

    Corrent answer: 2

     

    The CT images shown in Figures C and D display significant marginal impaction of the joint surface.

    Marginal impaction is common in posterior wall fractures and fracture-dislocations. Critical review of CT imaging of posterior wall fractures can help with preoperative planning for identifying impaction of the articular surface of the acetabulum. Restoration of the sphericity of the acetabulum to match that of the femoral head is important for successful outcome following ORIF of posterior wall fractures. A common surgical technique to accomplish joint surface restoration includes freeing the impacted articular segments, bone grafting of the void created to support the articular segments, and buttress plating of the posterior wall fracture fragments.

     

    Patel et al. discuss the challenge of interpreting imaging of the acetabulum for assessing fracture characteristics that may significantly impact success or surgical intervention. These characteristics include: articular displacement, marginal impaction, incongruity of the joint surface, intra-articular fragments, and osteochondral injury to the femoral head. Based on expert review of images, determination of significant marginal impaction had a poor intraobserver reliability, as did each of the other modifiers listed.

     

    Figures A and B are radiographs of the posterior wall fracture and hip dislocation. They do not show the large amount of marginal impaction of the acetabular surface. Figure C (coronal reconstruction) and Figure D (sagittal reconstruction) point out a large a amount of marginal impaction of the acetabular. Note the disruption of the joint surface on the intact portion of the acetabulum.

     

    Incorrect answers:

    1. Comminuted posterior wall fractures still should be surgically stabilized if the joint is unstable

      1. This impacted fragment on the margin of the main fracture line will likely heal regardless of restoration of the articular surface; however, this malreduction will lead to a incongruent joint surface

      2. These CT cuts do not show any osteochondral defects of the femoral head; however if found in other CT cuts or intraoperatively, they should be appropriately addressed

      3. Intraarticular fracture fragments should be removed from the joint, but if they make up a substantial portion of the joint surface, they should be incorporated in the fixation construct to obtain the goal of anatomic reduction of the joint surface

       

       

       

      OrthoCash 2020

       

  20. A 32-year-old female is involved in a motor vehicle collision and suffers a right hip dislocation. She is in the twelfth week of pregnancy.

    Evaluation in the emergency department reveals no other injuries and ultrasound reveals a strong fetal heart rate and no abnormalities. She undergoes emergent closed reduction but the hip remains unstable and a traction pin is placed. Post-reduction films are shown in Figure

    1. What is the most appropriate next step in management?

       

       

       

       

       

       

      1. Acute open reduction internal fixation

      2. Exam under anesthesia

      3. Skeletal traction for 6-8 weeks

      4. Fetal monitoring until 15 weeks followed by open reduction internal fixation

      5. Percutaneous pinning

         

        Corrent answer: 1

         

        This patient has a large posterior wall fracture of the right acetabulum with an unstable hip. The most appropriate next step in treatment is open reduction and internal fixation.

         

        Fixation of acetabular fractures during pregnancy is not contraindicated in the setting of stable fetal heart rate and no abnormalities on pelvic ultrasound.

        There is, however, an increased risk of complications for the mother and fetus. Injury severity and mechanism are most closely associated with increased rate of fetal complications. The trimester of pregnancy is not associated with increased risk of complications.

         

        Leggon et al. reviewed 101 cases of pelvic and acetabular fractures in pregnant patients and found mechanism of injury and injury severity were associated with higher mortality for both mother and fetus. Trimester of pregnancy was not associated with increased mortality.

         

        Flik et al. reviewed orthopaedic trauma in a pregnant patients and recommended fetal ultrasound for assessment of fetal well-being in all pregnant patients.

         

        Desai et al. investigated orthopaedic trauma during pregnancy and reported minimal radiation risk to the fetus when obtaining x-rays. They also advocate for LMWH as one of the safest choices for anticoagulation.

         

        Figure A is an x-ray showing a right posterior wall acetabular fracture. Figures B and C are Judet views of the pelvis focusing on the right hip. A large posterior wall fragment is visible in Figure B.

         

        Incorrect Answers:

        Answer 2,3,5: ORIF is the most appropriate treatment for this posterior wall fracture. There is no contraindication to ORIF.

        Answer 4: There is no indication to delay treatment as the fetal heart rate is normal and ultrasound shows no abnormalities.

         

         

        OrthoCash 2020

         

  21. Figure A is radiograph of a 50-year-old male science teacher that was involved in a motor vehicle accident. He underwent closed reduction as seen in Figure B and C. What would be the most appropriate treatment?

     

     

     

     

     

     

     

     

     

    1. Open reduction and internal fixation with medial bridge plate and lateral screw in non-lagging mode

    2. Tibiotalocalcaneal arthrodesis

    3. Open reduction and internal fixation with lateral and medial screw in lagging mode

    4. Closed reduction and internal fixation with medial and lateral screw in non-lagging mode

    5. Closed reduction with percutaneous pins Corrent answer: 1

    This patient is presenting with a Hawkins II talar neck fracture with medial wall comminution. The most appropriate treatment of this patient would be open reduction internal fixation with medial plate and lateral screw in non-lagging mode.

     

    The treatment of talar fractures is based on the severity of the fracture, soft-tissues, and patient factors. The fracture and subluxation of the subtalar joint should be reduced and stable anatomical fixation should be obtained. When there is comminution of either the superior, lateral or medial aspects of the talus, one should avoid shortening the medial wall as this will cause a varus malunion. The use of a medial or lateral plate can help to re-establish column length, which can often prevent this potential complication.

     

    Sanders et al. showed significant complications after fixation of talar neck fractures. They showed the incidence of secondary reconstructive procedures following talar neck fractures increased from 24% +/- 5% at 1 year to 48%

    +/- 10% at 10 years post-injury.

    Vallier et al. retrospectively reviewed the records of 39 fractures of the talar neck treated with open reduction and internal fixation. Twenty-one (54%) of thirty-nine patients had development of posttraumatic arthritis, which was more common after comminuted fractures (p < 0.07) and open fractures (p = 0.09).

     

    Vallier et al. reviewed 81 talar neck fractures to revisit the rate of osteonecrosis and post-traumatic arthritis based on the Hawkins Classification. They found that delaying definitive internal fixation does not increase the risk of developing osteonecrosis. Thirty-five patients (54%) developed posttraumatic arthritis, including 83% of those with an associated talar body fracture (p < 0.0001) and 59% of those with Hawkins type-III injuries (p < 0.01).

     

    Figure A shows a Hawkins II talar neck fracture. Figures B and C are saggital and coronal CT images, respectively, of the foot. There is significant comminution of the medial wall of the talus with extension into the subtalar joint.

     

    Incorrect Answers:

    Answers 2: Tibiotalocalcaneal arthrodesis would not be indicated primarily in a patient with no underlying arthritis of the tibiotalar joint and subtler joint.

    There is some research to suggest primarily subtalar arthrodesis with these injuries. However, to date, there is no high level evidence that has conclusively shown subtalar arthrodesis to be better than ORIF.

    Answers 3,4,5: Medial wall comminution will cause varus malreduction with lag screws or wires. A medial or lateral wall plate can be used buttress or briging mode to maintain length and prevent varus collapse. Lagging of the screws should be avoided as this may over compress the medial talar neck. Positional screws may also be used in non-lagging mode, as another way to avoid this complication.

     

     

     

    OrthoCash 2020

     

  22. A 28-year-old male college student sustains a severe foot injury from gunshot-related violence, and subsequently undergoes a lower-extremity amputation as shown in Figure A. At long-term follow-up, which of the following is the strongest predictor of patient satisfaction as related to his injury?

     

     

     

    1. Age less than 30

    2. Marijuana use

    3. Use of negative pressure wound therapy

    4. Male gender

    5. Ability to return to work Corrent answer: 5

    The strongest factor to predict patient-reported outcomes after trauma-related lower extremity amputations is the patient's ability to return to work. This is likely due to the effect of the return to work on the physical, emotional, and financial aspects of the patient's life.

     

    The LEAP study is a multicenter, prospective study evaluating multiple aspects of reconstruction versus amputation in the treatment of mangled extremity injuries. With regard to patient satisfaction, treatment variables such as decision for reconstruction versus amputation, or initial presence or absence of plantar sensation have little impact. In addition, demographic factors such as age, gender, socioeconomic status, and education level do not predict patient satisfaction. Instead, the most important predictors of patient satisfaction at 2 years after injury include the ability to return to work, absence of depression, faster walking speed, and decreased pain.

     

    O'Toole et al reviewed 463 patients treated for limb-threatening lower-extremity injuries and identified factors associated with patient reported outcomes two years after surgery. They found that return to work was the most associated with outcomes, but that physical functioning, walking speed, pain levels, and presence of depression were also associated to a lesser extent with outcomes.

    Bosse et al performed a multicenter, prospective study to assess outcomes of 569 patients with severe lower extremity limb injuries that resulted in either amputation or limb salvage procedures. They found that at two years postoperatively, no significant differences were seen between groups in patient-reported outcome. Worse outcomes were associated with rehospitalization for a major complication, a low educational level, nonwhite race, poverty, lack of private health insurance, poor social-support network, low self-efficacy (the patient's confidence in being able to resume life activities), and involvement in disability-compensation litigation.

     

    Figure A shows a clinical photograph of a Pirigoff amputation at early follow-up. This amputation is an end-bearing amputation that utilizes the plantar heel pad for weightbearing, and relies on a tibiocalcaneal arthrodesis.

     

    Incorrect Answers:

    1-4: These options are not as strong of a factor of patient satisfaction in longterm follow up after trauma-induced lower extremity amputation.

     

     

     

    OrthoCash 2020

     

  23. A 34 year-old male falls off of motorcycle on an outstretched hand suffering the injuries shown in Figures A and B. He is brought to the operating room and undergoes radial head replacement and fixation and repair of the coronoid and the lateral collateral ligament (LCL). Prior to closing, the elbow is still unstable upon testing range of motion. What is the next best step in management?

     

     

     

     

     

     

    1. Exchange radial head for larger implant

    2. Complete resection of radial head

    3. Cast at 90 degrees of flexion for 6-8 weeks

    4. Reinforce LCL repair with non-absorbable suture

    5. Repair the ulnar collateral ligament Corrent answer: 5

    Following complete fixation and repair of a terrible triad, a final range of motion test should be performed prior to closure. If still unstable, the next step should be to assess and repair the ulnar collateral ligament. Another option

    would be to placed a hinged external fixator.

     

    Operative reconstruction of a terrible triad injury should be performed in a systematic fashion, working from deep to superficial. Working through a lateral incision and through the radial head fracture, the coronoid should be fixed first, followed by radial head fixation or replacement and then repair/reconstruction of the LCL. If still unstable, the medial side should be addressed, or the patient placed in a hinged external fixator.

     

    Mathew et al review the anatomic, biomechanic, and operative principles (why the above step-by-step method works) to achieving appropriate stability in order to obtain early range of motion to maximize clinical outcome.

     

    Pugh et al. in this retrospective, multi-center study report outcomes on 36 terrible triad injuries fixed with the standard protocol described above. The authors recommend following this systematic approach to achieve the best results.

     

    Figures A and B are AP and lateral radiographs exhibiting a terrible triad elbow fracture-dislocation.

     

    Incorrect answers:

    Answer 1. Overstuffing the radial head should be avoided.

    Answer 2. Resection of the radial head is contraindicated in an unstable elbow. Answer 3. While casting the extremity may provide initial immobilization, you are unable to monitor the wound and truly assess stability, especially when swelling subsides.

    Answer 4. Reinforcing the LCL will not make increase any stability not already achieved.

     

     

     

    OrthoCash 2020

     

  24. When treating the pathology depicted in Figures A through D, which of the following is necessary to preserve the blood supply to the femoral head?

     

     

     

     

     

     

     

     

     

     

     

    1. Dissection of the gluteal musculature off the iliac crest

    2. Ligation of the ascending branches of the lateral femoral circumflex artery

    3. Greater trochanteric osteotomy

    4. Identification and detachment of the piriformis tendon

    5. Supine positioning

     

    Corrent answer: 3

     

    Figures A-D show a femoral head with associated acetabular fracture (Pipkin IV). Both the posterior wall fracture and the femoral head fracture can be addressed through a surgical dislocation via greater trochanteric osteotomy.

     

    Pipkin IV femoral head fracture (with associated acetabular fractures) are somewhat problematic in that the femoral head fracture is usually anterior, while the acetabular fracture usually involves the posterior wall. A Kocher-Langenbeck approach gives good access to the posterior wall but limited access to the articular surface and femoral head avascular necrosis (AVN) is a concern. A Smith-Peterson approach provides good access to the femoral head

    but not to the posterior wall. Combined approaches significantly increase the amount of surgical dissection. Surgical dislocation with trochanteric flip osteotomy provides access to the femoral head and posterior wall while preserving blood supply to the femoral head.

     

    Solberg et al. retrospectively reviewed 12 patients with Pipkin IV injuries treated via a trochanteric flip osteotomy. All patients healed their acetabular fractures. Eleven of 12 patients healed their femoral head fractures and one patient (8.3%) developed osteonecrosis.

     

    Henle et al. likewise treated 12 patients with Pipkin IV injuries through a trochanteric flip osteotomy. Two of 12 patients (16.7%) developed osteonecrosis. The remaining 10 patients (83.3%) had good or excellent results. Heterotopic ossification occurred in five patients, causing significant range of motion loss in four of these.

     

    Figure A is a pre-reduction AP pelvis in which the posterior wall fracture is apparent. Figure B is a post-reduction AP pelvis in which an infra-foveal femoral head fracture is apparent (Pipkin IV). Figure C is an axial CT cut which further characterizes the posterior wall fracture. Figure D is an obturator oblique showing femoral head dislocation and posterior wall fracture. The video shows a surgical hip dislocation technique.

     

    Incorrect Answers:

    Answer 1: The extended iliofemoral approach exposes the entire innominate bone, which is not necessary to treat this injury.

    Answer 2: The Smith-Peterson anterior approach provides access to the anterior femoral head but not the posterior wall of the acetabulum.

    Answer 4: Detachment of the short external rotators is not necessary for surgical dislocation.

    Answer 5: Surgical dislocation with greater trochanteric osteotomy is described in the lateral position. Supine positioning is not necessary.

     

     

     

    OrthoCash 2020

     

  25. A 42-year-old male presents to your clinic for the first time with the radiographs seen in Figure A. He sustained the injury 4 weeks ago while skiing overseas and treatment was provided by the local orthopaedic surgeon. The operative note states that he sustained an Gustilo Type I open fracture. After surgical fixation of this type of injury, what is the most common complication requiring reoperation?

     

     

     

    1. Chronic elbow instability

    2. Post-traumatic arthritis

    3. Infection

    4. Heterotopic ossification

    5. Loss of elbow range of motion Corrent answer: 5

    This patient sustained a terrible triad elbow fracture-dislocation. Reduced range of motion of the elbow joint is the most common complication REQUIRING reoperation with these injuries.

     

    Terrible triad elbow fracture-dislocations are characterized by posterolateral dislocation/lateral collateral ligament (LCL) injury, radial head fracture and coronoid fracture. Displaced fractures result in elbow instability. Acute radial head stabilization, coronoid open reduction and internal fixation, and LCL +/-medial collateral ligament (MCL) repair/reconstruction is considered the most appropriate treatment for displaced fractures. Operative complications include elbow stiffness, recurrent instability, arthritis, failure of hardware, heterotopic ossification, posterior interosseous nerve palsy and infection.

     

    Egol et al. looked at the functional outcomes of 27 patients that underwent fixation of terrible triad injuries. At one year follow-up, the average flexion-extension arc of elbow motion was 109 degrees +/- 27 degrees, and the average pronation-supination arc was 128 degrees +/- 44 degrees. Grip strength averaged 72% of the contralateral extremity. Although operative fixation led to functional elbow stability, results were poor.

     

    They included a reference to McKee et al. to highlight that intra-articular fractures of the elbow have high rates of stiffness. While not specific to terrible

    triads, they looked at the effectiveness of the posterior elbow approach in 25 patients that underwent internal fixation of intra-articular distal humerus fractures. They showed poor outcomes at a mean follow-up of 36 months with reduced range-of-motion, decreased strength and high re-operation rates.

     

    Figure A shows AP fluoroscopic image of a terrible triad injury that has undergone operative fixation. The radial head and coronoid have undergone open reduction internal fixation, and the MCL bony avulsion has been repaired.

     

    Incorrect Answers:

    Answer 1: Chronic elbow instability is more common following type I or II coronoid fracture when not operatively managed.

    Answer 2: Post-traumatic arthritis results from chondral damage at time of injury and/or residual instability.

    Answer 3: Infection is more prevalent with open fractures, however Type I injuries are usually not associated with increase in infection rates.

    Answer 4: Heterotopic ossification is a common complication after fixation of these injuries. However, it does not always necessitate reoperation.

     

     

     

    OrthoCash 2020

     

  26. Figure A is a radiograph from a 59-year-old male that was transferred to a Level I trauma center five hours after a motor vehicle accident. Closed reduction and skeletal traction was successfully performed in the trauma bay. Which of the following factors has been shown to increase the risk of unsatisfactory clinical outcome for this patient?

     

     

     

     

    1. Need for skeletal traction

    2. Mechanism of injury

    3. Gender

    4. Age

    5. Time to reduction Corrent answer: 4

    Age greater than 55-years-old has been found to be an independent risk factor for inferior clinical outcome in patients with combined acetabular fractures and hip dislocations.

     

    The most important initial step in management following resuscitation involves urgent reduction of the dislocated hip. This should be followed by a preoperative CT scan and ultimately surgical fixation of the combined acetabular fracture. Hip dislocations should be reduced within 6-12 hours for optimal outcome, although different critical times have been cited, particularly for dislocations with concomitant acetabular fractures. Skeletal traction may be required to maintain hip reduction.

     

    Moed et. al. present a Level 3 retrospective review of 100 patients who had been treated with open reduction internal fixation of an acetabular fracture. The authors found that factors associated with unsatisfactory clinical outcomes included age greater than 55, intra-articular comminution, osteonecrosis, and delay of greater than 12 hours for reduction of an associated hip dislocation.

    Additionally, they showed that there was a strong association of clinical outcome and final radiographic grade.

    Figure A demonstrates an acetabular fracture with concomitant hip dislocation. Incorrect Answers:

    Answer 1-3, and 5: Placement and need for skeletal traction, mechanism of

    injury, male gender, and time to reduction <6 hours have not been shown to be related to unsatisfactory outcomes.

     

     

     

    OrthoCash 2020

     

  27. A 37-year-old male cashier is shot in the leg. He sustains the injury shown in Figures A and B, and is subsequently taken to the operating room for intramedullary nailing. Figure C shows a radiograph of the nail starting point (*). What complication is most likely to result?

     

     

     

     

     

    1. Varus malunion

    2. Nonunion

    3. Valgus malunion

    4. Malrotation

    5. Superficial peroneal nerve injury Corrent answer: 3

    This patient is presenting with a comminuted fracture of the proximal third of the tibia. He is appropriately undergoing intramedullary nail fixation, however, the start point illustrated in Figure C is too medial and often leads to a valgus malunion.

     

    Intramedullary nail fixation is more technically demanding in proximal tibial fractures than diaphyseal fractures. The valgus deformity is due to imbalanced muscle forces on the proximal fragment, which are then accentuated by a start point that is too medial. An apex anterior (procurvatum) deformity can also occur and results from the pull of the patellar tendon or a posteriorly directed nail that deflects off the posterior tibial cortex and rotates the proximal fragment. The ideal starting point for proximal tibial fractures is slightly lateral to the medial aspect of the lateral tibial spine on a true AP x-ray and very proximal and just anterior to the anterior margin of the articular surface.

     

    Nork et al. reported the results of intramedullary nailing of proximal tibial fractures with emphasis on techniques of reduction. Various techniques were found to be successful including attention to the proper starting point, the use of unicortical plates, and the use of a femoral distractor applied to the tibia.

     

    Lowe et al. describe surgical techniques for complex proximal tibial fractures. They describe the extended leg position, use of a femoral distractor, temporary plate fixation, blocking (Poller) screws, and use of percutaneous clamps as means to achieve reduction during fixation.

     

    Figure A and B show an AP and lateral radiograph of a comminuted extra-articular fracture through the proximal third of the tibia. Figure C demonstrates a start point that is too medial (represented by the asterisk) for intramedullary nail fixation. Illustration A and B show the ideal start point for intramedullary nail fixation of the tibia on AP and lateral radiographs.

     

    Incorrect Answers:

    1. Varus malunion is more likely to occur in midshaft tibia fractures with an intact fibula.

    2. Nonunion after a proximal tibial fracture treated with intramedullary nailing is less common than malunion.

      1. Malrotation occurs most commonly after IM nailing of fractures through the distal third of the tibia.

      2. The superficial peroneal nerve is at risk during distal screw fixation using a LISS plating technique for fracture fixation.

       

       

       

       

       

       

       

      OrthoCash 2020

       

  28. A 24-year-old motorcyclist is brought in as a polytrauma after striking a tree at 65 mph. He is found to have injuries involving the chest, abdomen, pelvis, as well as a left open femoral shaft fracture. He undergoes resuscitation in the trauma bay. Which of the following parameters best supports proceeding with irrigation, debridement and external fixation as opposed to immediate reamed intramedullary nailing?

    1. Temperature = 35.5°C (95.9°F)

    2. Fractures of ribs 2-3 with left apical pneumothorax

    3. Grade IV liver laceration with SBP = 85 mmHg

    4. Left superior and inferior pubic ramus fractures

    5. Lactate = 2.3 mg/dL

     

    Corrent answer: 3

     

    Significant abdominal trauma with evidence of hemorrhagic shock (SBP < 90 mmHg) following resuscitation is an unstable parameter and therefore is an indication to proceed with damage control orthopaedics (irrigation and debridement of open fractures and temporizing external fixation) in a polytraumatized patient.

     

    The management of orthopaedic injuries in a polytrauma patient depends on the physiological stability of the patient. In an unstable patient, damage control orthopaedics (DCO) is preferred over early total care (ETC) to avoid an iatrogenic second hit with development of adult respiratory distress syndrome (ARDS) and/or multiple organ failure. Clinical parameters indicative of instability include shock (BP < 90 mmHg, refractory to blood products, lactate

    > 2.5 mg/dL), coagulopathy (platelet count < 90,000 mm3, fibrinogen < 1 g/L), hypothermia (< 35°C), and significant chest, abdomen or pelvis injuries (pulmonary contusions, severe liver/spleen lacerations, pelvic ring disruption).

     

    Pape et al. (2009) authored a review article detailing the management of a multitrauma patient. Polytrauma patients can be classified as stable, borderline, unstable or in extremis using a variety of criteria pertaining to hemodynamic stability, coagulation, temperature and soft tissue injury.

    Patients who are stable or borderline can undergo ETC, while patients who are unstable or in extremis should be managed with DCO.

     

    Pape et al. (2008) concluded that all patients who underwent early femoral nailing demonstrated increased systemic inflammatory response compared to external fixation, regardless of clinical stability. However, unstable patients

    with a preexisting elevation of inflammatory status are likely more impacted by this additional increase. Improved postoperative clinical status coincided with a less vigorous inflammatory response.

     

    Illustration A is a table from Pape et al (2009) depicting the criteria used to determine clinical condition of a polytraumatized patient. Illustration B is an algorithm from Pape et al (2009) detailing management of the multitrauma patient.

     

    Incorrect Responses:

    Answer 1: Temperature = 35.5°C (95.9°F) is a borderline parameter. Temperature < 35°C (95°F) would be an unstable parameter.

    Answer 2: Fractures of 2-3 ribs with a pneumothorax is a borderline parameter. Pulmonary contusions would be an unstable parameter.

    Answer 4: Left superior and inferior pubic ramus fractures (AO pelvis type A) is a stable parameter. An AO pelvic type C such as a sacroiliac dislocation (rotationally and vertically unstable) would be an unstable parameter.

    Answer 5: Lactate = 2.3 mg/dL is a stable parameter. Lactate > 2.5 mg/dL would be an unstable parameter.

     

     

     

     

     

     

     

     

     

    OrthoCash 2020

     

  29. A 92-year-old female sustains the injury shown in Figure A to her nondominant extremity as the result of a non-syncopal ground-level fall. She denies any previous injury or pain of the elbow, and her medical history is significant only for osteoporosis and hypothyroidism. What is the most appropriate treatment for her injury?

     

     

     

    1. Immediate range of motion as tolerated with a sling for comfort

    2. Long arm cast for 3 weeks, then physical therapy for motion

    3. Open reduction and internal fixation

    4. Radiocapitellar arthroplasty

    5. Total elbow arthroplasty Corrent answer: 5

    Use of total elbow arthroplasty (TEA) in the elderly is a well-recognized method of treatment of complex distal humerus fractures. This procedure allows for improved ROM, improved patient-reported outcomes, and decreased revision rates as compared to fixation.

     

    TEA is a preferred alternative for ORIF in elderly patients with complex distal humeral fractures that are not amenable to stable fixation. Elderly patients appear to accommodate to objective limitations in function with time, which is important, as most recommendations list restrictions of lifting no more than 5-10 pounds postoperatively.

     

    McKee et al conducted a prospective, randomized, controlled trial to compare functional outcomes, complications, and reoperation rates in elderly patients with displaced intra-articular, distal humeral fractures treated with open reduction-internal fixation (ORIF) or primary semiconstrained total elbow arthroplasty (TEA). Patients who underwent TEA had a quicker procedure, improved DASH scores at 6 months, improved elbow ROM, and decreased revision rates.

    Athwal et al review TEA and the options available at the time of publication. They also report on the techniques and purported advantages of arthroplasty as compared to fixation of complex distal humerus fractures.

     

    Frankle et al reviewed patients >65 years old with distal humerus fractures at a minimum of 2 years follow-up. Outcomes were excellent in 33% of cases undergoing ORIF and 92% excellent with TEA. They recommend TEA in instances of arthritis, osteoporosis, or other diagnoses requiring steroids.

     

    Figure A shows a significantly comminuted distal humerus fracture in an osteoporotic patient. Illustration A shows the same patient after undergoing total elbow arthroplasty.

     

    Incorrect Answers:

    1:Immediate range of motion is not recommended for this injury, even with the "bag of bones" treatment method. A brief period of immobilization is generally recommended for this technique.

    2: Casting is not indicated for this injury.

    3: ORIF of this injury will lead to worse outcomes as compared to arthroplasty. 4: Isolated radiocapitellar replacement is not indicated for this injury.

     

     

     

     

     

     

    OrthoCash 2020

     

  30. A 56-year-old right hand dominant attorney falls from standing and sustains the closed injury shown in Figure A. The treating surgeon elects to fix her fracture using a plate and screw construct. Based on

    the available imaging, which of the following fracture characteristics best justifies this fixation choice?

     

     

     

     

    1. Fracture displacement

    2. Intra-articular fracture extension

    3. The fracture extends distal to the coronoid

    4. Oblique fracture line

    5. Fracture comminution

     

    Corrent answer: 5

     

    This patient has a displaced, intra-articular, comminuted olecranon fracture. Comminution is an indication for plate fixation.

     

    Most displaced olecranon fractures are treated operatively. Options include tension band constructs, intramedullary screws, plate and screw fixation or fragment excision with triceps advancement. Any construct relying on interfragmentary compression (tension band, intramedullary screws) requires a non-comminuted fracture pattern. Plate fixation is indicated in the setting of comminution, extension past the coronoid, or in the setting of associated instability.

     

    Bailey et al. retrospectively reviewed 25 patients who underwent plate fixation of displaced olecranon fractures. Twenty-two of 25 patients had good or excellent outcomes. Five of 25 patients (20%) of patients required plate removal for symptomatic hardware. The authors concluded that plate fixation

    was an effective treatment for displaced olecranon fractures, with good functional outcomes.

     

    Figure A shows a displaced, comminuted olecranon fracture without evidence of propagation past the coronoid.

     

    Incorrect answers:

    Answer 1. All displaced olecranon fractures should be considered for internal fixation. Displacement alone does not direct choice of implant.

    Answer 2. Intra-articular extension is an indication for surgery to re-establish articular congruity but does not dictate implant selection.

    Answer 3. Extension distal to the coronoid is an indication for plate fixation but there is no evidence of such extension on the radiograph shown

    Answer 4. This fracture is comminuted, without a distinct fracture line.

     

     

     

     

    OrthoCash 2020

     

  31. A 35-year-old male was involved in a high speed motorcycle accident. He has a closed head injury, bilateral pulmonary contusions and splenic rupture. His orthopaedic injuries are shown in Figure A. He has a blood pressure of 90/50 mm Hg and a heart rate of 115, despite aggressive resuscitation. An arterial blood gas reveals that his blood lactate is 3.5 and base deficit is -6 mmol/L. Following successful closed reduction of the right hip in the operating room with a percutaneous inserted Schantz pin, what is the next most appropriate treatment for his orthopaedic injuries?

     

     

     

    1. Bilateral open reduction and internal fixation

    2. Open reduction internal fixation on the right, reamed intramedullary nailing on the left

    3. Temporizing external fixation on the right, open reduction and internal fixation on the left

    4. Bilateral reamed intramedullary nailing

    5. Bilateral temporizing external fixation Corrent answer: 5

    This patient presents with features of hemodynamic instability and a high injury severity score. The next most appropriate treatment would be temporizing external fixation bilaterally. This patient meets the criteria for damage control orthopaedics.

     

    Damage control orthopaedics is an approach that contains and stabilizes orthopaedic injuries so that the patient's overall physiology does not undergo further inflammatory insult. As a result, external fixation of femoral shaft fracture and pelvic stabilization is an effective treatment under this strategy. Other indications include vascular injury and severe open fracture.

     

    Pallister et al. reviewed the effects of surgical fracture fixation on the systemic inflammatory response to major trauma. They show that early stabilization of major long bone fractures is beneficial in reducing the incidence of acute respiratory distress syndrome and multiple organ failure. However, early fracture surgery increases the post-traumatic inflammatory response, which

    carries a higher complication rate compared to temporary fixation.

     

    Tisherman et al. created clinical guidelines for the endpoints of resuscitation. Level I data found that standard hemodynamic parameters do not adequately quantify the degree of physiologic derangement in trauma patients. The initial base deficit, lactate level, or gastric pH should be used to stratify patients with regard to the need for ongoing fluid resuscitation.

     

    Pape et al. retrospectively reviewed the impact of early total care vs. damage control orthopaedics in the treatment of femoral shaft fractures in polytrauma patients. They found a significantly higher incidence of acute respiratory distress syndrome (ARDS) with intramedullary nailing (15.1%) compared to external fixation (9.1%) when DCO subgroups were compared.

     

    Figure A is a pelvic AP radiograph showing a right hip fracture-dislocation with an ipsilateral femoral shaft fracture. On the left side there is a displaced pertrochanteric hip fracture.

     

    Incorrect Answers:

    Answer 1-3: Open reduction internal fixation would not be indicated in damage control orthopaedics.

    Answer 4: Bilateral reamed intramedullary nailing would be indicated in early total care of these injuries. As this patient is hemodynamically unstable, with a high injury severity score and bilateral lung contusions, damage control orthopaedics would be indicated.

     

     

     

    OrthoCash 2020

     

  32. Which of the following has been shown to be the greatest risk factor for refracture after implant removal from a radial shaft?

    1. Removal of locking screws

    2. Removal of small fragment plates

    3. Removal of metaphyseal implants

    4. Removal of implants less than 1 year after insertion

    5. Removal of protective splinting from limb earlier than 10 weeks postoperatively

    Corrent answer: 4

     

    Removal of implants earlier than 1 year after insertion is a risk factor for refracture of the bone after implant removal.

     

    The risk of refracture after hardware removal is multifactorial. Multiple

    variables have been studied such as protective splinting for 6 weeks after hardware removal, waiting 12 months or more prior to hardware removal, and the location of the fracture. The variable that seems to correlate most with the risk of refracture is a diaphyseal location of the initial fracture. Large fragment plates (4.5 mm), when removed, are also at higher risk for refracture in the forearm.

     

    Deluca et. al reported on a case series of patients who sustained a refracture of a forearm after implant removal. They noted that radiolucency at the site of the original fracture was seen in most refractured patients when the plate was removed. They also recommend delaying implant removal to two years after insertion to minimize risk.

     

    Rumball et. al reported that the incidence of refracture after forearm implant removal is 6% in their series. They found that early removal, lack of postoperative immobilization, and plate size are the most critical risk factors for refracture.

     

    Illustration A shows a forearm with evidence of refracture after implant removal.

     

    Incorrect Answers:

    Answer 1: Removal of locking screws does not increase the risk of refracture. Answer 2: Removal of large fragment plates (4.5mm) are at increased risk of refracture as compared to small fragment plates (3.5mm).

    Answer 3: Metaphyseal implant removal has a decreased risk of refracture compared to diaphyseal implant removal.

    Answer 5: Protective splinting for 6 weeks is all that is generally recommended for these patients.

     

     

     

     

     

    OrthoCash 2020

     

  33. A 23-year-old male arrives to the trauma bay after a motorcycle crash caused by a drive-by shooting. The patient is awake and alert and following commands. Vital signs include a blood pressure of 145/90 and a heart rate of 117bpm. Initial lactate is reported as 2.4 mmol/L. The patient has 2 rib fractures on the right with a clear chest radiograph. The patient is neurovascularly intact with a 4cm transverse wound over the medial ankle. Figures A, B and C exhibit his orthopaedic injuries. What is the most appropriate management?

     

     

     

     

     

    1. Irrigation, debridement and placement external fixator right ankle, external fixation femur and intramedullary fixation tibia

    2. Irrigation, debridement and placement external fixator right ankle, intramedullary fixation femur and tibia

    3. Irrigation, debridement and placement external fixator right ankle, intramedullary fixation femur and external fixation tibia

    4. Irrigation, debridement and placement external fixator right ankle, femur and tibia

    5. Irrigation, debridement and external fixation right ankle and skeletal traction

    Corrent answer: 2

     

    The patient is relatively hemodynamically stable. In this case the femur and tibia should be definitively fixed while the open ankle fracture can be irrigated and debrided and placed in a spanning external fixator, temporizing for later definitive fixation.

     

    Aside from an elevated heart rate and mildly elevated lactate (normal < 2.5 mmol/L), the patient is relatively stable making him a good candidate for long bone stabilization and temporizing external fixation of the right ankle. Gross contamination of the open injury also supports temporizing fixation, which can be brought back for repeat I&D and possible fixation.

     

    Pape et al. compared outcomes for intramedullary nailing (IMN) versus staged fixation for femur fractures in stable versus borderline patients. Borderline patients were defined as those with multi-system injury (especially to lungs) and exhibited higher lung complications following acute IMN when compared to stable patients with isolated orthopaedic injuries.

     

    O'Brien reviewed the literature regarding early total care in regards to IMN stabilization of femur fractures. Summarized data noted isolated injuries treated with early IMN had good outcomes, whereas those with head or lung injury had worse outcomes and pulmonary complications.

     

    Figure A exhibits a right open ankle fracture dislocation. Figure B exhibits a mid-shaft tibia fracture. Figure C exhibits a ballistic mid-shaft femur fracture.

     

    Incorrect Answers:

    Answer 1: Early total care for the femur and tibia is appropriate in this stable patient.

    Answer 3: While temporizing the ankle fracture is appropriate in this clinical scenario, definitive intramedullary fixation should be performed for both the tibial and femur fractures.

    Answers 4,5: This course of action may be appropriate for someone in extremis.

     

     

     

    OrthoCash 2020

     

  34. Figure A is an anterior-posterior (AP) radiograph of a 27-year-old male who was a bicyclist struck by a motor vehicle. He was intubated in the field and unresponsive in the trauma slot. Ultrasound of his abdomen is positive for blood and he is brought to the operating room emergently for an exploratory laparotomy. He is found to have ischemic bowel and a grade 4 liver laceration. His lactate is 9.0 mg/dL. Which figure represents the next appropriate step in regard to his pelvic ring injury?

     

     

     

     

     

     

     

     

     

     

     

     

     

     

    1. Figure B

    2. Figure C

    3. Figure D

    4. Figure E

    5. Figure F

     

    Corrent answer: 4

     

    The radiograph exhibits an elevated left hemipelvis with complete sacroiliac disruption, which can be temporized with placement in skeletal traction.

     

    The patient is unstable, as indicated by an elevated lactate level. The most appropriate next step is temporizing skeletal traction to reduce the left hemipelvis.

     

    Langford et al. review the initial diagnosis, evaluation and resuscitation in the management of pelvic fractures. Reduction of pelvic volume can be achieved with pelvic binders and temporizing external fixation for anterior posterior compression (APC) and/or lateral compression (LC) fracture patterns, while skeletal traction can help do the same in vertical shear patterns.

     

    Matullo et al. review the uses of skeletal traction in orthopaedic trauma, where lower extremity skeletal traction can be an efficient, fast, easy way to help reduce pelvic volume in vertical shear injuries, especially when the patient is unstable and not cleared for definitive fixation.

     

    Figure A exhibits an elevated left hemipelvis indicative of a vertical shear injury and complete SI disruption. Figure B is an example of a pelvic binder. Figure C is a pelvic reconstruction plate. Figure D is a schematic of an anterior pelvic external fixator. Figure E is a schematic drawing of a patient in lower extremity

    skeletal traction. Figure F is a radiograph exhibiting S1 and S2 sacroiliac (SI) screws.

     

    Incorrect answers:

    Answer 1: A pelvic binder would not be helpful in this scenario and actually may worsen the deformity.

    Answer 2: Definitive plate fixation of the anterior pelvis will be needed, but only when the patient is adequately resuscitated.

    Answer 3: An anterior external fixator may actually do more harm than good as it may require more time than placing skeletal traction, and also have difficulty controlling the posterior pelvis from the front.

    Answer 5: Percutaneous SI screws may be placed later, once the patient is resuscitated and stable.

     

     

     

    OrthoCash 2020

     

  35. A 38-year-old man is involved in a motor vehicle collision and suffers the grossly open injury shown in Figure A. He subsequently undergoes irrigation and debridement and placement of an external fixator. In Figure B, if the proximal pin is placed at the red circle as compared to the black circle, the patient is at increased risk for which of the following?

     

     

     

    1. Foot drop

    2. Injury to the anterior tibial artery

    3. Septic arthritis

    4. Flexion contracture of the knee

    5. Patellar tendon rupture Corrent answer: 3

    The patient is at increased risk of septic arthritis when placing the proximal tibial pin too proximal due to penetration of the joint capsule. Pin site flora can track into the joint and lead to a septic knee.

     

    Tibial external fixators can be used to temporize tibial shaft, pilon, and ankle fractures not ready for definitive management due to soft tissue concerns and/or practice of damage control orthopaedics. Intracapsular placement of fixator pins can lead to septic arthritis. The capsular reflection typically extends 14 mm distal to the subchondral line.

     

    DeCoster et al. reported a cadaveric dissection study for safe placement of proximal tibia pins and determined that the capsule inserts 14 mm below the articular surface along the posteromedial and posterolateral surfaces. For fractures requiring extremely proximal pin placement, they recommend

    anterior cortex penetration only at least 6 mm distal to articular surface.

     

    Reid et al. investigated safe transtibial pin placement using MRI and cadaveric and volunteer knees. They found that pin placement 14 mm distal to subchondral bone will result in low likelihood of capsular penetration.

     

    Figure A is an AP radiograph showing a segmental middle third tibia/fibula fracture. Figure B is a lateral diagram of the tibia showing potential sites of proximal pin placement.

     

    Incorrect Answers:

    Answer 1: Foot drop would occur secondary to common peroneal nerve injury, which is not at risk with very proximal pin placement.

    Answer 2: The anterior tibial artery is not at risk with very proximal pin placement.

    Answer 4: The knee is free to move and therefore should not develop a flexion contracture secondary to the external fixator.

    Answer 5: Patellar tendon rupture is not complication of external fixator pin placement in the tibia.

     

     

     

    OrthoCash 2020

     

  36. Figures A and B are radiographs of a 43-year-old, right-hand dominant, male that injured his arm in a motor vehicle accident. What would be an absolute indication for surgical fixation of his injury?

     

     

     

    1. Radial nerve palsy

    2. Intra-articular extension

    3. 2mm fracture distraction, 5 degrees of rotational malignment

    4. Ipsilateral proximal both bone forearm fracture

    5. Bilateral fracture

     

    Corrent answer: 4

     

    This patient has a humeral shaft fracture. An absolute indication for surgery would include a floating elbow, i.e. ipsilateral both bone forearm fracture.

     

    The primary causes of humeral fractures include motor vehicle accidents, falls, or violent injury. Almost all cases are treated non-operatively with functional bracing. The absolute indications for surgical management include: ipsilateral vascular injury, severe soft-tissue injury, open fracture, compartment syndrome, and associated ipsilateral forearm fracture, ie, floating elbow. The relative indications for surgical management include: segmental fracture, intraarticular extension, significant fracture distraction, bilateral humeral fracture, inability to maintain acceptable alignment, and polytrauma.

     

    Klenerman et al. reviewed non-operative treatment of humeral shaft fractures. They showed that acceptable results could be achieved even after 20° of

    anterior bowing, 30° of varus angulation, 15° of malrotation, and 3 cm of shortening.

     

    Carroll et al. reviewed the management of humeral shaft fractures. They state the indications for operative fixation to be polytraumatic injuries, open fractures, vascular injury, ipsilateral articular fractures, floating elbow injuries, and fractures that fail nonsurgical management. Surgical techniques include external fixation, open reduction and internal fixation, minimally invasive percutaneous osteosynthesis, and antegrade or retrograde intramedullary nailing

     

    Figure A and B shows a comminuted mid-shaft humeral fracture with intraarticular extension.

     

    Incorrect Answers:

    Answer 1,2,3,5: These associated injuries are NOT an absolute indication for surgery. An area of controversy is the occurrence of new-onset radial nerve injuries after closed manipulation. However, the current algorithm does not support early open nerve exploration and fixation.

     

     

     

    OrthoCash 2020

     

  37. Which of the following findings is a contraindication in retrograde nailing of a periprosthetic distal femur fracture around a total knee arthroplasty?

    1. Posterior-stabilized total knee implant

    2. Cruciate retaining total knee implant

    3. Spiral fracture pattern

    4. Distal femoral replacement

    5. Knee flexion contracture of 15 degrees Corrent answer: 4

    A distal femoral replacement (TKA) implant will generally preclude placement of a retrograde nail due to the long stem on the femoral component.

     

    Supracondylar femur fractures above a well-fixed TKA component are increasingly common. These fractures are often treated with a lateral locking plate, but can also be treated with a retrograde nail in certain circumstances. An important factor in determining if nailing is a viable option are knowing the TKA implant and it's design. In addition, if the TKA component is known, the maximum size of reamer head and nail can be determined preoperatively from the size of the femoral 'box'.

    Schutz et al report on a prospective multicenter study of 112 patients who underwent fixation of a distal femur fracture with the LISS system. They report that 90% of fractures went on to union and they attribute all of the failures to either the high-energy nature of particular fractures or a lack of experience in applying the plate in an appropriate pattern. They also note that primary grafting of these fractures is not necessary.

     

    Illustration A shows a periprosthetic femur fracture treated with a retrograde nail.

     

    Incorrect Answers:

    1: A posterior-stabilized implant can be treated with an intramedullary nail in many circumstances but can be technically challenging, depending on the components.

    2: A cruciate retaining TKA is not a contraindication to use of a retrograde nail. 3: A spiral pattern periprosthetic supracondylar femur fracture can be treated with a femoral nail.

    5: A knee flexion contracture will often provide the flexion necessary for access to the box of the femoral component. A knee extension contracture, however, can preclude access to this box for placement of a nail.

     

     

     

     

     

     

    OrthoCash 2020

  38. A patient falls and sustains the isolated injury seen in Figures A and B. The surgical plan includes open reduction and internal fixation with a small mini-fragment plate using a direct lateral approach. During the approach, the forearm was placed in a fully pronated position. What would be the correct position of the forearm during plate application?

     

     

     

     

    1. Full pronation

    2. 25 degrees pronation

    3. Neutral

    4. 25 degrees supination

    5. Full supination

     

    Corrent answer: 3

     

    Using the lateral approach (Kocher or Kaplan), the correct placement of the arm should be in a neutral position so that the plate can be placed on the bare area of the proximal radius.

     

    Displaced radial head fractures with less than 3 fragments can be amendable to open reduction internal fixation. The methods of fixation include buried or headless screws, if placed at the articular surface, or posterolateral plating, if placed in the bare area. The safe zone for plating is located at a 90-110 arc from the radial styloid to Lister's tubercle with the arm in neutral rotation. This position helps to avoid impingement of ulna against the plate with forearm rotation. It should be noted that during the approach, that the forearm should be fully pronated to avoid injury to the posterior interosseous nerve.

     

    Mathew et al. reviewed the concepts of terrible triad injuries of the elbow. Radial head fractures are treated conservatively when there is an isolated minimally displaced (less than 2mm) fracture with no mechanical block to motion. Open reduction internal fixation is used for Mason II or III fractures with < 3 fragments. Radial head replacement is considered for comminuted

    fractures (Mason Type III) with 3 or more fragments.

     

    Cheung et al. reviewed the surgical approaches to the elbow. The lateral approach (Kocher or Kaplan) is most commonly used with these injuries. The Kocher approach utilizes the intramuscular plane between anconeus and extensor carpi ulnaris. Kaplan utilizes the plane between extensor digitorum commons and extensor carpi radialis brevis.

     

    Figure A and B show AP and lateral radiographs of the left elbow. There is a displaced radial head fracture. Illustration A shows a schematic diagram of the radial head "safe zone" between the radial styloid to Lister's tubercle.

     

    Incorrect Answers:

    Answers 1,2,4,5: Plating the arm in these positions would not adequately expose the bare area of the radial head using the lateral approach to the elbow.

     

     

     

     

     

     

    OrthoCash 2020

     

  39. A 38-year-old male is involved in a high speed motor vehicle collision. He has a Glasgow Coma Scale of 13 and receives 2 liters of fluid en route to the emergency department. Upon evaluation in the emergency department, he is found to have a bilateral femoral shaft fractures, a right ankle fracture, and a left both bone forearm fracture. He also has 2 left sided rib fracture and a grade II liver laceration. His heart rate is 130 and blood pressure is 85/50. All of the following

    would be indications to practice damage control orthopaedics in this patient except:

    1. Bilateral femur fractures

    2. Rib fractures

    3. Lactate of 5.2

    4. Urine output of 20 cc/hr

    5. Heart rate and blood pressure Corrent answer: 2

    Rib fractures without evidence of further thoracic trauma would not be an indication to practice damage control orthopaedics. This patient is underresuscitated based on his lactate level, urine output, and vital signs and definitive management should be delayed.

     

    Damage control orthopaedics is the practice of delaying definitive management of fractures and utilizing temporary stabilization (such as an external fixator) until a patient has recovered from the initial physiologic insult of trauma.

    Patients are at increased risk for perioperative complications such as ARDS and multi-system organ failure during the acute period after polytrauma. In addition to underresuscitation, other indications to practice damage control orthopaedics include: injury severity score>40 (or >20 with thoracic trauma), bilateral femoral fractures, hypothermia below 35 degrees Celsius, and pulmonary contusions.

     

    Pape et al. (2007) studied the incidence of acute lung injuries in polytrauma patients undergoing either intramedullary nailing or external fixation and later definitive fixation of femoral shaft fractures. They found that patients undergoing immediate intramedullary nailing were nearly 6.7 times more likely to have acute lung injury

     

    The Canadian Orthopedic Trauma Society studied the effect of reamed versus unreamed femoral nailing on incidence of ARDS for femoral shaft fractures in trauma patients using a randomized controlled study. They found no difference between the groups.

     

    Pape et al. also examined the pathophysiological cascades that accompany soft tissue injuries of the extremities, abdomen, and pelvis and recommend a more comprehensive for evaluation of patients with these injuries.

     

    Incorrect Answers:

    Answer 1: Bilateral femur fractures are an indication to practice damage control orthopaedics and delay definitive fixation

    Answers 3,4,5: All of these answers suggest that the patient is underresuscitated. Definitive fixation should be delayed

     

     

     

    OrthoCash 2020

     

  40. The anterior intrapelvic (modified Stoppa) approach is most appropriate for which of the following fractures?

     

     

     

     

     

     

     

     

     

     

     

     

     

     

     

    1. Figure A

    2. Figure B

    3. Figure C

    4. Figure D

    5. Figure E

     

    Corrent answer: 4

     

    The anterior intrapelvic (AIP) or modified Stoppa approach provides access to the quadrilateral plate, which is a common location for fracture displacement in associated both column acetabulum fractures as seen in Figure D.

     

    Compared to the traditional ilioinguinal approach, the modified Stoppa with a lateral window can offer comparable access to the quadrilateral plate, which can allow for its use in associated both column fracture patterns.

     

    de Peretti et al. prospectively followed 25 patients with both column fractures

    treated via an iliofemoral approach. Results led the authors to not recommend the extensile approach for both column fractures due to lack of efficiency and high complication rates.

     

    Alonso et al. compared the extensile iliofemoral and triradiate approaches, and both reported acceptable results. However, concerning were the relatively high rates of heterotopic ossification, despite prophylaxis.

     

    Bible al. performed a cadaver study to quantify the amount of access provided by the modified Stoppa approach. This approach provides access to approximately 80% of both the inner pelvis, and the quadrilateral plate, however, comparison to the ilioinguinal approach was not performed.

     

    Shazar et al., in a cohort comparison between the ilioguinal and Stoppa approaches, noted better visualization and potential improve fracture reduction via the Stoppa approach for both column fractures. However, this study was limited in its retrospective and relative observer bias.

     

    Figure A depicts a posterior wall fracture dislocation with concomitant femoral neck fracture. Figure B is an iliac oblique view which depicts a posterior column fracture. Figure C exhibits a posterior column + posterior wall fracture. Figure D depicts acetabular fracture with protrusio. Figure E exhibits a posterior wall fracture.

     

    Incorrect answers:

    Answers 1,2,3, and 5: Posterior sided injuries, especially for these injuries, would probably be best approached via the Kocher-Langenback approach. Surgical dislocation with a trochanteric slide may help visualization in answer 1.

     

     

     

    OrthoCash 2020

     

  41. Figure A is a radiograph of a 75-year-old woman that fell onto her non-dominant shoulder from a standing height. She was treated nonoperatively for 9 months but continues to complain of pain when she elevates her arm. In patients with this type of fracture pattern, what factor has the greatest impact on fracture healing?

     

     

     

    1. Hand dominance

    2. Angulation of fracture

    3. Smoking

    4. Early physical therapy

    5. Diet

     

    Corrent answer: 3

     

    This patient has an impacted varus proximal humerus fracture. Smoking has been shown to increase the nonunion risk up to 5.5 times with these fractures.

     

    Impacted varus proximal humerus fractures can be managed effectively with non-operative care. The major factors that influence non-union are age and smoking. Solid bony union can be seen in 93-98% of patients at 1 year, with more than 97% of people returning to pre-injury level of function. The angulation of fracture, hand dominance and physical therapy does not seem to influence bone union or functional outcomes with this fracture pattern.

     

    Court-Brown et al. looked at the outcomes of impacted varus fractures. They determined that the age of the patient was the major factor in overall outcome. They showed that the best results occurred in younger patients, but results deteriorate with advancing age. Physical therapy was not found to

    impact outcome.

     

    Hanson et al. showed that impacted varus fractures can be successfully managed with non-operative care. They found that overall fracture displacement had a minor impact of fracture healing and functional outcome. The predicted risk of delayed union and nonunion was 7% with patients that smoke. This was 5.5 times greater than non-smokers.

     

    Figure A shows an AP radiograph of a varus angulated proximal humerus fracture. This radiograph shows delayed atrophic union.

     

    Incorrect Answers:

    Answers 1,2,4: These factors do not have a significant influence on bone healing.

    Answer 5: Diet has shown to improve bone healing but this does not have the greatest impact.

     

     

     

    OrthoCash 2020

     

  42. A 26-year-old male epileptic patients presents with right shoulder pain and deformity after a grand mal seizure. After medical stabilization, he denies previous injury to his shoulder. Pre-reduction and post-reduction radiographs of the shoulder are shown in Figures A-C, respectively; physical examination reveals a normal upper extremity neurovascular examination. After shoulder immobilization, what would be the next most appropriate step in management of this patient?

     

     

     

     

     

     

     

     

    1. Abduction brace for three weeks, followed by therapy

    2. Right shoulder MR arthrogram

    3. Open reduction and internal fixation

    4. Hemiarthroplasty

    5. Early range of motion Corrent answer: 3

    This patient has presented with a fracture dislocation of the right shoulder. After urgent closed reduction, this patient requires open reduction internal fixation of the proximal humerus, and greater tuberosity fracture fragment in particular.

     

    Isolated greater tuberosity fractures may be associated with shoulder dislocations. Careful review of imaging is critical to identify fracture lines that may extend into the humeral neck and head. If these extensions go undetected, catastrophic propagating fractures may occur during closed reduction maneuvers. Treatment is usually with open reduction internal fixation (ORIF). Young patients with proximal humerus fractures should be treated more aggressively with ORIF as compared to elderly patients. Another example would be a severely impacted valgus proximal humeral fracture in a young patient.

     

    Erasmo et al. examined of 82 cases of humerus fracture dislocations treated with the lateral locking plates. Overall outcomes were excellent to good based on standard scoring systems. Complications included avascular necrosis (12%), varus positioning of the head (4.8%), impingement syndrome (3.6%), secondary screw perforation (3.6%), non-union (2.4%) and infection (1.2%).

    Robinson et al. looked at severely impacted valgus proximal humeral fractures treated with open reduction internal fixation in young patients. Anatomic reduction is required with lateral plating to re-establish the normal head/neck angle. Good to excellent results can be achieved with fixation methods.

     

    Figure A shows an anterior fracture-dislocation of the right shoulder. Figure B and C show post-reduction radiographs with a congruent glenohumeral joint. Displacement of the greater tuberosity (GT) fragment is greater than 5mm.

     

    Incorrect Answers:

    Answer 1: The GT fragment is displaced more than 5-10mm which would warrant surgical reduction and stabilization of the fracture.

    Answer 2: MR arthrogram may be indicated to rule out an intra-capsular soft-tissue injury. However, this would not be warranted prior to fixation of the fracture.

    Answer 4: Hemiarthroplasty would be indicated in some 4-part proximal humerus fractures or elderly patients. Considering this patient's age and valgus impaction of the fracture, ORIF would be most appropriate.

    Answer 5: Early range of motion would be warranted AFTER fixation of the fracture.

     

     

     

    OrthoCash 2020

     

  43. Pelvic packing can be performed to temporarily treat a hemodynamically unstable patient with a pelvic ring fracture. Which of the following is the preferred location of the skin incision to perform pelvic packing?

    1. Right anterior superior iliac spine (ASIS) to mid-symphysis, left lateral window incision

    2. Left ASIS to mid-symphysis, right lateral window incision

    3. Subumbilical incision

    4. ASIS to ASIS bilaterally

    5. Pararectus incision

     

    Corrent answer: 3

     

    The preferred skin incision location is a subumbilical incision, 6-8cm extending upwards from the pubic symphysis towards the umbilicus; this allows access to all of the appropriate areas for pelvic packing.

     

    Following skin incision, the rectus fascia is then divided in the midline which allows for access to both sides of the bladder for packing deep in the pelvic

    brim. On each side, 3 lap pads are placed from sacroiliac joint to the retropubic space, all placed below the level of the pelvic brim.

     

    Hak et al. review the options for emergent treatment in life threatening hemorrhage secondary to pelvic fractures. The authors offer several options for emergent treatment, which includes the use of pelvic binders, the placement of external fixators, pelvic packing and interventional angiography. Goals include reduction of pelvic volume and stopping rapid hemorrhage to save a patient's life. Pelvic packing, properly performed, is done through a subumbilical incision, as described above.

     

    Osborn et al. retrospectively reviewed and compared emergent pelvic packing to angiography in hemorrhagic pelvic fracture clinical scenarios. The authors noted comparable results in mortality with a noted decrease in need for post-procedure transfusions in the pelvic packing group.

     

    Cothren et al. reported their outcomes following an institutional algorithmic change from pelvic ex-fix/angiography to pelvic packing and ex-fix. Since their institutional change, the authors noted a significant decrease in transfusions, need for angiography and mortality.

     

    Incorrect answers:

    Answers 1,2: While these approaches may give access to the pelvic brim, each only give access to one side; one needs to circumferentially access and pack the pelvis.

    Answer 4: An ASIS to ASIS skin incision is an unnecessary dissection with too many important structures that lie very close to the skin, including the femoral vessels.

    Answer 5: A pararectus incision is an alternative approach to access the quadrilateral plate for acetabular fractures and does not play a role in pelvic packing.

     

     

     

    OrthoCash 2020

     

  44. A 28-year-old man is brought by ambulance to the emergency department after falling from the roof of his home four hours ago. Upon initial evaluation, he has visible deformities of his bilateral lower extremities and a positive FAST exam. Heart rate is 135, blood pressure 85/58, and urine output is 40 cc over 3 hours. According to ATLS guidelines, what percentage of his blood volume has this patient likely lost?

    1. <15%

    2. 15-30%

    1. 30-40%

    2. 40-50%

    3. >50%

     

    Corrent answer: 3

     

    This patient has likely lost 30-40% of his blood volume and is likely in stage III hemorrhagic shock on the basis of his heart rate, blood pressure, and urine output.

     

    Initial assessment of a trauma patient should involve evaluation of airway, breathing and circulation. An average adult has approximately 5 L of circulating blood volume. Class III and IV hemorrhagic shock, approximated by loss of greater than 30% of blood volume, typically requires resuscitation with fluids and blood products. Lactate level (normal <2.5) is typically the most sensitive indicator of adequate perfusion.

     

    According to ATLS guidelines, the emphasis of the initial assessment is to assume the worst injury and begin treatment before diagnosis. In cases of hemorrhagic shock, treatment involves aggressive resuscitation with crytalloid and/or blood products.

     

    Illustration A is a chart showing the physiological changes associated with different levels of hemorrhagic shock

     

    Incorrect Answers:

    1: Blood loss of <15% typically leads to no changes in vital signs

    2: Blood loss of 15-30% typically leads to heart rate >100 and diminished urine output

    4, 5: Blood loss of greater than >40% typically leads to heart rate >140, decreased blood pressure and negligible urine output.

     

     

     

     

     

    OrthoCash 2020

     

  45. Which of the following fracture patterns would be most appropriately treated with open reduction and internal fixation with posteromedial and lateral plates via dual incisions?

     

     

     

     

     

     

     

     

     

     

     

     

     

     

    1. Figure A

    2. Figure B

    3. Figure C

    4. Figure D

    5. Figure E

     

    Corrent answer: 4

     

    The patient in Figure D has a bicondylar tibial plateau fracture with a posteromedial fracture fragment, which is an indication for lateral and posteromedial plating via dual incisions.

     

    Bicondylar tibial plateau fractures are typically treated with open reduction internal fixation. Studies have shown that the posteromedial fragment is common in bicondylar tibial plateau fractures. Standard lateral implants do not gain adequate screw purchase in posteromedial bone. Posteromedial incisions using the interval between the semimembranosus and medial head of the gastrocnemius can be used to gain access to the fragment and provide exposure to place a posteromedial plate.

     

    Barei et al. reviewed 57 OTA C-type bicondylar tibial plateau fractures, and found the presence of a posteromedial fragment in 74% of cases. They suggest

    the use of alternate methods of fixation for the posteromedial fragment rather than lateral fixed-angle plates alone.

     

    Higgins et al. reviewed 111 patients with bicondylar tibial plateaus that underwent CT scan evaluation. They found the incidence of a posteromedial fragment in 59% of the patients and on average accounted for 25% of the articular surface.

     

    Barei et al. reviewed 83 patients treated with dual incisions for fixation of bicondylar tibial plateau fractures. They found deep infections in 8.4% of patients. Satisfactory articular reduction was found in 62% of patients, satisfactory coronal alignment in 91% of patients, satisfactory sagittal alignment in 72%, and satisfactory condylar width in 98%.

     

    Figure A shows an AP and lateral radiograph of a split/depressed lateral tibial plateau fracture. Figure B shows an AP and lateral radiograph of a proximal third tibial shaft fracture with a proximal fibula fracture. Figure C shows an AP and lateral radiograph of a medial plateau fracture with articular depression. Figure D shows an AP and lateral radiograph of a bicondylar tibial plateau fracture. Figure E shows an AP and lateral radiograph of a tibial tubercle fracture in a tibia with open physes.

     

    Incorrect Answers:

    Answer 1: Schatzker II tibial plateau fractures can be treated with a single lateral plate.

    Answer 2: Proximal third tibial shaft fractures can be treated with an intramedullary nail or percutaneous plating

    Answer 3: Schatzker IV tibial plateau fractures can be treated with a single medial or posteromedial plate.

    Answer 5: Tibial tubercle fractures in children can be treated with anterior to posterior screws.

     

     

     

    OrthoCash 2020

     

  46. Figure A shows the radiographs of a 87-year-old patient after a fall from standing. He lives in a nursing home and uses a walker to transfer from bed to chair. His past medical history includes recurrent urinary tract infections, congestive heart failure, angina and diabetes. Which of the following factors is associated with the best postoperative outcome in this patient?

     

     

     

    1. Immediate surgical intervention

    2. Postoperative epidural analgesia

    3. Postoperative antibiotics

    4. Pre-operative medical optimization

    5. Choosing total hip arthroplasty instead of hemiarthroplasty Corrent answer: 4

    Geriatric patient with hip fractures tends to have a number of coexisting medical conditions that impact surgical risk. A successful pre-operative medical evaluation has the greatest impact on surgical outcomes in this patient population.

     

    Patients with complex past medical histories are at great risk of complications with surgery. This helped to drive the formation of the ASA classification system as a way to score patients out of 5 based on their anesthetic and surgical risks. Patients with significant systemic disease (ASA III and IV) have shown to demonstrate poorer outcomes as compared to patients with less severe medical comorbidities (ASA I and II).

     

    Egol et al. looked at factors that impact the outcomes of hip fractures in geriatric patients. They showed that cardiac and pulmonary complications were most frequent complications post-operatively. It was stated that early mobilization and pre-operative evaluations have the greatest impact on outcomes.

     

    Parvizi et al. looked at the thirty-day mortality following hip arthroplasty for acute fracture. They reviewed a database of 7774 consecutive patients that underwent hip arthroplasty for the treatment of an acute fractures. The overall mortality was 2.4%. Risk factors were found to be cemented implants, female patients, elderly patients, and patients with cardiorespiratory comorbidities.

    Roberts et al reviewed the 2015 AAOS Clinical Practice Guideline: Management of Hip Fractures in the Elderly. Although pre-operative medial optimization was not mentioned in this review, there is strong evidence to support an interdisciplinary care program for patients with hip fractures.

    Figure A shows a displaced left femoral neck fracture. Incorrect Answers:

    Answer 1: Most studies have shown an association between operative delay of

    more than 24-48 hours and a higher 1-year mortality rate. However, In patients with with more than 3 medical comorbidities, medical optimization has a greater impact on post-operative outcomes than time to surgery.

    Answer 2: Post-operative epidural anaesthesia has shown to decrease postoperative delirium but does not affect 30-day mortality.

    Answer 3: Postoperative antibiotics have not shown to reduce the 30-day mortality.

    Answer 5: Total hip arthroplasty has shown to improved functional outcomes at 1 year compared with hemiarthroplasty. However, there has been no change in mortality.

     

     

     

    OrthoCash 2020

     

  47. A 36-year-old male sustains severe injuries as a result of a motor vehicle collision. After 5 days, the patient is stabilized and transferred to your facility for continued management. After obtaining appropriate imaging, including the images shown in Figures A, B and C the patient should be given which of the following mobility restrictions?

     

     

     

     

     

     

     

     

    1. Touch-down weight bearing on the left lower extremity, non-weightbearing on the right lower extremity

    2. Weight bearing as tolerated on the left lower extremity, non-weightbearing on the right lower extremity

    3. Non-weight bearing on the left lower extremity, non-weightbearing on the right lower extremity

    4. Touch-down weight bearing on the left lower extremity, weight bearing as tolerated on the right lower extremity

    5. Weight bearing as tolerated on the left lower extremity, weight bearing as tolerated on the right lower extremity

    Corrent answer: 2

     

    Figures B and C shows a stable, impacted lateral compression (LC) type I injury, which can be treated with immediate mobilization without restrictions. The right lower extremity has a pilon fracture, which should be treated with non-weight bearing.

     

    LC-I injuries are stable patterns, as they involve an impaction injury to the anterior sacrum, which has some inherent stability. In addition, the posterior sacro-iliac (SI) soft tissue structures remain intact, which provides critical stability during immediate mobilization.

     

    The reference by Tile is a review article on the principles of management of these injuries, and he reviews how the Tile classification system is important to help determine surgical need and where the stabilization is required. For the stable LC-1 type injury, no fixation is required.

     

    Incorrect Answers:

    Answer 1: A patient cannot do touch-down weight bearing on one side and non-weightbearing on the other side simultaneously.

    Answer 3: The LC-I injury pattern is stable and can undergo immediate weightbearing as tolerated.

    Answer 4: The LC-I pattern is stable and would allow immediate weightbearing, and the right pilon fracture would require non-weightbearing. Answer 5: The right pilon fracture would require non-weightbearing at this time.

     

     

     

    OrthoCash 2020

     

  48. A 24-year-old male sustains the fracture dislocation shown in Figure A. How is this fracture pattern best classified?

     

     

     

     

    1. Moore I

    2. Moore II

    3. Schatzker III

    4. Schatzker V

    5. OTA type 41B3 Corrent answer: 2

    Figure A shows a Moore Type II fracture. Moore type II fractures consist of fractures involving the entire tibial condyle, where the fracture line begins in the opposite compartment and extends across the tibial eminence.

     

    A number of classification systems are used to describe tibial plateau fractures. The main classification systems that are widely used include Schatzker, AO/OTA, Hohl and Moore. The Moore classification describes high energy fracture-subluxation injuries of the knee. This is thought to have clinical implications as the type of fracture pattern can indicate an associated soft-

    tissue injury.

     

    Moore retrospectively reviewed over a 1000 proximal tibia fractures to devise a classification systems based on the characteristic of five types of tibia plateau fracture patterns. He believed this system helped to better understand knee instability and concomitant soft-tissue injury. For example, Moore Type III fractures (rim avulsion) are associated with a high rate of neurovascular injury.

     

    Figure A shows a fracture involving the entire tibial condyle. Illustration A shows a schematic diagram of the Moore classification (I - V). Illustration B shows a table that describes the Moore classification (I - V).

     

    Incorrect Answers:

    Answer 1: Moore Type I is a coronal split fracture of the medial tibia plateau which displaces distally. It does not cross over to the contralateral side of the eminence.

    Answer 3: Schatzker Type III is a pure lateral plateau depression.

    Answer 4: Schatzker Type V is a bicondylar fracture where the metaphysis and diaphysis remain intact.

    Answer 5: OTA type 41B3 is a partial articular fracture with a split-depression of the unilateral plateau.

     

     

     

     

     

     

     

     

    OrthoCash 2020

     

  49. A 19-year-old male football player plants and twists his right lower extremity sustaining a spiral fracture of his distal third tibial shaft. Of the following images, which is most commonly associated with distal third spiral tibial shaft fractures.

     

     

     

     

     

     

     

     

     

     

     

     

     

    1. Figure A

    2. Figure B

    3. Figure C

    4. Figure D

    5. Figure E

     

    Corrent answer: 1

     

    The patient has a spiral distal tibial shaft fracture. Spiral distal tibial shaft fractures are commonly associated with intraarticular fracture extension, usually in the form of a posterior malleolus fracture.

     

    Spiral distal third tibial shaft fractures need to be evaluated for intraarticular extension prior to operative management. As this can be missed on x-rays, a CT scan of the ankle is recommended to identify this associated injury. This is important when intramedullary fixation is used for definitive management, as nail insertion can displace a previously non displaced intraarticular fracture.

    Anteroposterior screw fixation prior to nailing may be useful in these cases.

     

    Boraiah et al. found that in 62 patients with spiral distal tibial fractures, 39% (24 patients) had a posterior malleolus fracture. They recommended CT evaluation of the ankle to prevent missed intraarticular fractures.

     

    Hou et al. found a posterior malleolus fracture in 9.7% (28 out of 288 cases) of patients with tibial shaft fractures. They recommended CT or MRI evaluation of the ankle prior to surgery.

     

    Figure A shows an axial CT scan of a right ankle with a posterior malleolus

    fracture. Figure B shows an AP radiograph of a right ankle with a vertical medial malleolus fracture. Figure C shows a coronal CT scan of a right ankle with a Tillaux fracture. Figure D shows a sagittal CT scan of a right ankle with a comminuted talus fracture. Figure E shows a lateral radiograph of a right knee showing a knee dislocation. Illustration A shows an AP radiograph of a left distal third spiral/oblique tibial shaft fracture. Illustration B shows a lateral radiograph of a left distal third spiral/oblique tibial shaft fracture. Illustration C shows a sagittal CT of a distal third spiral/oblique tibial shaft fracture with a posterior malleolus fracture.

     

    Incorrect Answers:

    Answer 2: Vertical medial malleolar fractures are not typically associated with distal third tibial shaft fractures.

    Answer 3: Juvenile Tillaux fractures occur typically in patients with open physes. They do not occur in patients with closed physes, and are not associated with tibial shaft fractures.

    Answer 4: Spiral/oblique tibial shaft fractures are not associated with talus fractures.

    Answer 5: Knee dislocations are considered high energy injuries, and are not associated with tibial shaft fractures.

     

     

     

     

     

     

     

     

     

     

    OrthoCash 2020

     

  50. A 67-year-old female patient presents with increasing right hip/thigh pain over the past three months, which is now recalcitrant to anti-inflammatories. There is no history of trauma or constitutional symptoms. Her past medical history consists of hypertension, coronary artery disease, osteoporosis and gastric reflux. Physical examination reveals mild pain at the extremes of range of motion of the hip and a painful right sided limp. A radiograph of the right hip is seen in Figure

    1. What would be the most appropriate treatment for this patient at this time?

       

       

       

      1. Observation only

      2. Referral to physiotherapy

      3. MRI spine and hip

      4. Total hip arthroplasty

      5. Intramedullary femoral nail Corrent answer: 5

        This osteoporotic female patient is presenting with subtrochanteric lateral cortical thickening and hip pain. This is consistent with an insufficiency fracture of the femur secondary to use of bisphosphonate medication for treatment of osteoporosis. The most appropriate treatment would be intramedullary femoral nail fixation.

         

        Bisphosphonate medications have been shown to be associated with atypical (subtrochanteric) femur fractures. These patients often have prodromal hip pain and lateral cortical thickening on radiographs prior to fracture. In addition, there has shown to be a significantly increased risk of fracture in the presence of the “dreaded black line” that occurs at the site of thickening.

         

        Lenart et al. examined a case series of patients using bisphosphonates for the treatment of osteoporosis. They identified 15 postmenopausal women who had

        been receiving alendronate for a mean (±SD) of 5.4±2.7 years and who presented with atypical low-energy fractures. Cortical thickening was present in the contralateral femur in all the patients with this pattern.

         

        Goh et al. retrospectively reviewed patients who had presented with a low-energy subtrochanteric fractures. They identified 13 women of whom nine were on long-term alendronate therapy. Five of these nine patients had prodromal pain in the affected hip in the months preceding the fall, and three demonstrated a stress reaction in the cortex in the contralateral femur.

         

        Figure A shows a right hip radiograph with subtrochanteric lateral cortical thickening. There is mild arthritic changes in the hip. Illustration A shows a bone scan and radiographs of subtrochanteric lateral cortical thickening that resulted in fracture.

         

        Incorrect Answers

        Answer 1: Symptomatic lateral femoral cortical stress fractures, known as "dreaded black lines", should be treated with early stabilization to prevent subtrochanteric fracture.

        Answer 2: Referral to physiotherapy would not be indicated.

        Answer 3: Symptoms are related to a subtrochanteric stress fracture, which has been identified on radiograph. MRI would not be indicated.

        Answer 4: This patient has mild arthritis. Treatment with a long bridging femoral stem may stabilize the fracture, However, the gold standard treatment would be intramedullary nailing.

         

         

         

         

         

         

        OrthoCash 2020

         

  51. A right-hand dominant female sustains a right proximal humerus fracture. The patient is provided a sling, and is recommended pendulum exercises with elbow range of motion to begin in 1 to 2

    weeks. Which of the following would be an indication for surgical management?

     

     

     

     

     

     

     

     

     

     

    1. Age greater than 70 years.

    2. Fracture pattern in Figure A

    3. Significant medical comorbidities.

    4. Fracture pattern in Figure B

    5. Fracture pattern in Figure C

    Corrent answer: 4

     

    The patient has been treated with non-operative management for her proximal humerus fracture. Operative management should be considered in patients with head splitting proximal humerus fractures and in those with dislocations that cannot be reduced.

     

    Head splitting proximal humerus fractures should be treated with operative management. Open reduction internal fixation versus hemiarthroplasty are used to treat this type of fracture. Surgical management is also considered in proximal humerus fractures in young patients, in fractures where the greater tuberosity is displaced >5 mm, and in proximal humerus fractures associated with humeral shaft fractures.

     

    Koval et al. studied 104 patients with one-part proximal humerus fractures treated non-operatively, and found 80% with good or excellent results. They also found that 90% of patients treated non-operatively had either no or mild pain about the shoulder at follow-up.

     

    Lefevre-Colau et al. performed a randomized prospective study on 74 patients with an impacted proximal humerus fracture. One group was treated with early mobilization of the shoulder (within 3 days after the fracture) while the other group was immobilized for 3 weeks followed by physiotherapy. They concluded that early mobilization was safe and allowed for quicker return to functional use of the affected limb.

     

    Figure A shows an AP radiograph of a right minimally displaced greater tuberosity proximal humerus fracture. Figure B shows AP and axillary radiographs of a right head split proximal humerus fracture that is posteriorly dislocated. Figure C shows an AP radiograph of a right minimally displaced Salter Harris II proximal humerus fracture. Illustration A shows an AP radiograph of a left valgus impacted proximal humerus fracture with a greater tuberosity fragment displaced >5mm treated with ORIF.

     

    Incorrect Answers:

    Answers 1, 2, 3, and 5 are all factors that would favor non-operative management for a proximal humerus fracture.

     

     

     

     

     

    OrthoCash 2020

     

  52. A 37-year-old male arrives to the trauma slot following a high-speed motorcycle crash. His Glasgow Coma score is 14 and his only orthopaedic injury is exhibited in Figure A. His current vital signs are a BP of 90/60, HR 120, and a lactate of 2.5 mMol/L. Other findings include a grade 1 splenic laceration and bilateral pulmonary contusions seen on chest radiograph. Which of the following has been suggested as an indication to perform damage control orthopedic care?

     

     

     

    1. HR >110

    2. Bilateral pulmonary contusions seen on chest radiograph

    3. SBP = 90mmHg

    4. Unilateral femur fracture

    5. Lactate = 2.5 mMol/L

     

    Corrent answer: 2

     

    Pulmonary contusion severe enough to be diagnosed on chest radiograph alone is an indicator that the patient may benefit from damage control orthopaedics (DCO).

     

    Despite the patient's overall stable nature, suffering pulmonary injury severe enough to be seen on x-ray alone suggests that temporary stabilization with staged definitive fixation may avoid potential morbidity.

     

    Pape et al. review the evolution and balance of early total care (ETC) and DCO. Summarizing the literature, the authors report several standalone indicators that would justify DCO regardless of stable status. This includes: Injury Severity Score of greater than 40, Injury Severity Score of greater than 20 with chest trauma, multiple injuries with severe pelvic/abdominal trauma/ hemorrhagic shock, bilateral femoral fractures, pulmonary contusion noted on radiographs alone, hypothermia of less than 35 degrees C), and a head injury with an Abbreviated Injury Score of 3 or greater.

     

    Figure A exhibits a right femoral shaft fracture. Illustration A exhibits a summarized table stating the criterion used to determine the condition of a polytrauma patient. (Table from Pape et al, PMID: 19726738)

     

    Incorrect answers:

    Answer 1: Heart rate alone (even elevated severely) is not an indicator to

    implement DCO.

    Answer 3: A systolic BP of 90 mmHg is not an indicator to implement DCO. A SBP of 70mmHg or lower may indicate a patient in extremis (along with other parameters) indicating the benefits of DCO.

    Answer 4: A unilateral femur fracture alone in a stable patient is not an indicator of DCO; a patient with bilateral femur fractures, however, may benefit from DCO.

    Answer 5:A lactate of 2.5 mMol/L, while slightly elevated, does not warrant DCO. A lactate value of greater than 6 alone would indicate the need for DCO.

     

     

     

     

     

     

    OrthoCash 2020

     

  53. A 24-year-old patient presents after a fall from the balcony of a third story building in which he landed on his feet. He reports lumbar back pain and numbness in his perineum region. Radiographs of his hips and pelvis are seen in Figure A, while CT images are shown in Figures B and C. How is this fracture pattern best classified?

     

     

     

     

     

     

    1. Young-Burgess APC Type II

    2. Young-Burgess LC Type I

    3. "U" Type Spinopelvic Dissociation

    4. Denis Zone-I

    5. Denis Zone-II

     

    Corrent answer: 3

     

    This patient has a Denis zone-III "U" Type Spinopelvic Dissociation of the sacrum.

     

    The Denis classification system for sacral fractures is based on anatomical fracture zones. Zone-III fractures involve the spinal canal and have a high rate of associated neurologic injury. It is important to recognize bilateral sacral

    fractures with a transverse component, as this often causes spinopelvic discontinuity and possible cauda equina. The lack of mechanical continuity between the spine and pelvis will most likely require reduction and fixation for initial stability. There may also be a need for sacral decompression with fixation mechanisms given the onset of neurologic symptoms.

     

    Mehta et al. reviewed sacral fractures. They report that patients with a transverse sacral fracture involving more proximal sacral segments (S1, S2, or S3) tend to have a higher prevalence of bladder dysfunction than do those involving distal sacral segments (S4 or S5).

     

    Schildhauer et al. report the best way to visualize Denis zone-III "U" type fractures is to obtain a lateral view of the sacrum or sagittal reformatted images with a CT scan. Standard pelvic radiographs often miss this injury.

     

    Figure A shows an AP pelvis with suggestion of sacral irregularities. No fracture pattern can be indentified. Figures B and C show CT scan images showing a "U" Type fracture pattern with angulation and translational displacement of the cephalad and caudad parts of the sacrum. Illustration A shows examples of complex sacral Denis zone-III fractures.

     

    Incorrect Answers:

    Answer 1: APC Type II fractures do not classify fracture extensions into the sacral canal. The posterior SI ligaments are considered intact. There is usually disruption of sacrospinous and sacrotuberous ligaments.

    Answer 2: LC Type I is an oblique or transverse ramus fracture and ipsilateral anterior sacral ala compression fracture.

    Answer 4,5: Figures A-C demonstrate a "U" type fracture of the sacrum.

     

     

     

     

     

     

    OrthoCash 2020

  54. A 55-year-old male presents with the radiographs seen in Figures A and B after falling off his bike. Physical examination reveals an isolated, closed elbow injury. His limb is neurovascularly intact. Which complication would be most likely with surgical fixation of this injury?

     

     

     

     

     

     

     

    1. Ulnar neuropathy

    2. Posterior interosseous nerve injury

    3. Early loss of fixation

    4. Elbow flexion contracture

    5. Avascular necrosis

     

    Corrent answer: 4

    This patient is presenting with a comminuted capitellar and trochlear fracture. If treated with open reduction internal fixation, the most likely post-operative complication would be elbow stiffness or contracture.

     

    An axial compression force transmitted by the radial head to the capitellum with the elbow in a semiflexed position can result in a shear fracture of the anterior portion of the capitellum. AP, lateral, and radiocapitellar radiographs are recommended to identify these injuries. The lateral X-ray may reveal the “double arc” sign, which represents extension of the capitellum fracture into the trochlea. Extension of the fracture into the trochlea has important implications into the surgical approach to these injuries.

     

    Ruchelsman et al. retrospectively reviewed the outcomes of sixteen patients with capitellar fractures treated with open reduction internal fixation. They found the presence of greater flexion contractures at the time of follow-up in elbows with Type-IV capitellar fractures. All fractures healed, and no elbows had instability or weakness with fixation.

     

    Ring et al. retrospectively reviewed the outcomes of twenty one distal humerus articular fractures that were reduced and stabilized with implants buried beneath the articular surface. Ten patients required a second operation: (6) release of an elbow contracture; (2) treatment of ulnar neuropathy; (1) removal of hardware; (1) early loss of fixation.

     

    Figures A and B show AP and lateral radiographs of the elbow with a comminuted fracture of the capitellum and trochlea. Note the “double arc” sign on the lateral view. Illustrations A and B show open reduction internal fixation of the fracture. Note fixation with multiple interfragmentary screws.

     

    Incorrect Answers:

    Answers 1,2,3,5: All of these may be complications with this surgery. These complications are not as common as elbow contracture or stiffness.

     

     

     

     

     

    OrthoCash 2020

     

  55. Figure A shows a radiograph of a 30-year-old male who underwent fixation of a left leg injury just over two years ago. He presents with persistent pain and drainage from the distal wound despite 4 months of oral antibiotics. He has no systemic symptoms. He has a past medical history of Grave's disease and Irritable Bowel Syndrome. What would be the best management at this stage?

     

     

     

    1. Chronic suppressive, culture-directed, antibiotic therapy

    2. Above knee amputation

    3. Endocrine consultation, irrigation and debridement, removal of hardware and negative-pressure wound therapy

    4. Irrigation and debridement, removal of hardware, over-reaming medullary canal, external fixation and culture-directed antibiotics

    5. Irrigation and debridement, retention of hardware, acute bone grafting and culture-directed antibiotics

    Corrent answer: 4

     

    This is a case of fracture nonunion in the setting of chronic osteomyelitis and infected hardware. The best treatment option available would be irrigation and debridement, removal of hardware, ring external fixator and culture directed antibiotics.

     

    The management of infected nonunion in the setting of chronic osteomyelitis is technically demanding. The aims of treatment are to eradicate the infection and obtain bone union. Non-surgical options are largely unsuccessful in patients with draining chronic osteomyeltis in the setting of infected hardware and nonunion. Surgical options involve incision and debridement of necrotic tissue followed by reconstruction of bone and possible soft tissue (to provide

    healthy viable coverage). The most common techniques are ringed fixator/circular frames, staged intramedullary device with or without external fixator, free tissue transfer, or radical debridement, bone grafting, and fixation.

     

    Motsitsi et al. reviewed the management of infected nonunion of long bones. They suggest that the Ilizarov technique is regarded as a standard treatment in infected nonunion of the tibia. When there is bone defect after debridement, the bone can be shortened or treated with bone transport.

     

    Egol et al look at a series of patients with chronic osteomyelitis. Limb salvage should be attempted in all patients. The presence of a chronic draining sinus requires surgical debridement and culture directed antibiotics. Infected hardware should be removed. A two-stage strategy is the best and well-proven treatment option.

     

    Figure A shows a intramedullary nail in the left tibia. There is a moderate amount of bone loss at the fracture site with mixed sclerotic bone suggestive of osteomyelitis.

     

    Incorrect Answers:

    Answer 1: Chronic suppressive therapy would be considered in a patient that is unable to undergo surgical intervention

    Answer 2: Amputation is a primary option for patients unable to undergo lengthy or multiple surgical interventions.

    Answer 3: After removing the hardware, the fracture non-union will need to be stabilized.

    Answer 5: This patients has a chronic infection that requires radical bone debridement and local infection control. The use of acute bone grafting in a single stage procedure in the distal 1/3 tibia (poor blood supply) is less ideal, especially in the setting of infection.

     

     

     

    OrthoCash 2020

     

  56. A 32-year-old male sustained the injury seen in Figure A after a motor vehicle accident. Which of the following factors is most predictive of mortality with this type of injury?

     

     

     

    1. Fracture classification

    2. Number of blood transfusions in the first 24 hours

    3. Gender

    4. Time to operative fixation

    5. Use of pelvic binders Corrent answer: 2

    The best predictors of mortality with pelvic ring fractures include older age and hemodynamic shock at presentation. The amount of blood transfused indicates the severity of hemodynamic instability.

     

    Pelvic ring fractures are typically high energy, blunt injuries. The leading cause of mortality with these injuries is hemorrhage and hemodynamic instability.

    The most common source of hemorrhage include venous injury (80%), which is usually caused by a shearing injury of posterior thin walled venous plexus and bleeding cancellous bone. Other sources of hemorrhage include arterial injury (10-20%) from the superior gluteal artery (posterior ring injury, anterior posterior compression [APC] pattern), internal pudendal artery (anterior ring injury, lateral compression [LC] pattern) and obturator artery.

     

    Smith et al. found fracture pattern and angiography/embolization were not predictive of mortality in patients with unstable pelvic injuries. The three factors they found to be predictive were: increased blood transfusions in the first 24 hours, age >60 years, and increased Injury Severity Scores (ISS) scores. Deaths were most commonly from exsanguination (<24 hours) or multiorgan failure (>24 hours).

     

    Starr et al. demonstrated that age and shock on presentation were most predictive of mortality after pelvic ring injury.

    Figure A shows a APC III pelvic fracture. Illustration A shows a table describing the Young-Burgress classification of pelvic ring fractures.

     

    Incorrect Answers:

    Answers 1,3-5: Gender, fracture pattern, time to operative fixation or use of pelvic blinders do not correlate directly with mortality.

     

     

     

     

     

     

    OrthoCash 2020

     

  57. Figure A shows intraoperative radiographs of a 45-year-old patient with a left elbow injury. What would be the next most appropriate step in this patients care?

     

     

     

     

    1. Early range of motion

    2. Hinged elbow brace for 4 weeks

    3. Repair lateral collateral ligament

    4. Remove and upsize implant

    5. Remove and downsize implant Corrent answer: 5

    The intraoperative images are consistent with overstuffing of the ulnohumeral joint during a radial head replacement. The most appropriate next step would be removing and downsizing the implant.

     

    Overstuffing the radiohumeral joint by >2.5 mm can significantly alter elbow kinematics. It has also shown to lead to pain and early joint disease. The most sensitive method to assess for overstuffing of the joint is by direct visualisation intra-operatively. This can be performed by visualising the lateral aspect of the ulnohumeral joint when the radial head is resected and comparing this to when the trial radial head is reduced in place. In comparison, radiographic asymmetry of the medial ulnohumeral joint has been shown to be less sensitive. Radiographic findings of incongruity of ulnohumeral joint only occurs when significant overlengthening of the radius occurs.

     

    Frank et al. examined the effect of radial head thickness in seven cadaver specimens. They found that incongruity of the medial ulnohumeral joint would only become apparent radiographically after overlengthening of the radius by

    >or=6 mm.

     

    Doornberg et al. examined seventeen computed tomography scans of the elbow to investigate the height of the radial head relative to the lateral edge and central ridge of the coronoid process. They found that the radial head was on average only 0.9 mm more proximal than the lateral edge of the coronoid process.

     

    Figure A shows intraoperative radiographs of a patient that has undergone a radial head arthroplasty. There is significant widening of the medial ulnohumeral joint on an AP radiograph as well as widening of the ulnohumeral joint on the lateral radiograph,

     

    Incorrect Answers:

    Answer 1: Early range of motion is supported after radial head arthroplasty. However, the next most appropriate step in this scenario would be the intraoperative assessment for radiohumeral joint overstuffing.

    Answer 2: Hinged braces may be used in the post operative period. However, overstuffing the radiohumeral joint is of first priority.

    Answer 3: There is no indication in this question for lateral collateral ligament repair

    Answer 4: Removing the implant and upsizing the radial head would be the opposite of what is needed in this scenario.

     

     

     

    OrthoCash 2020

  58. Aside from improving the intramedullary nail starting point in Figure A, the use of blocking screws could have been used at the time of fixation to prevent this post-operative deformity. What would have been the correct orientation of these screw(s) in the proximal fragment?

     

     

     

     

    1. Medial only

    2. Lateral only

    3. Anterior and medial

    4. Posterior and medial

    5. Posterior and lateral Corrent answer: 5

    The clinical presentation is consistent with a malunion of a proximal one-third tibia fracture with a valgus and procurvatum deformity. The correct orientation of Poller blocking screws to help prevent this malalignment would be in the posterior and lateral aspects of the metaphyseal fragment.

     

    Insertion of the blocking screw lateral to the nail prevents valgus deformity, and insertion of the blocking screw posterior to the nail prevents apex anterior deformity. A more lateral starting point for nail insertion can also help to

    prevent valgus deformity. The semiextended position of the knee for nail insertion also helps to eliminate the tendency for the fracture to flex, due to the avoidance of excessive knee flexion during the reduction.

     

    Stedtfeld et al. describes the mechanical model for blocking screws. They state that two or more blocking screws can be placed in each plane (AP and lateral) alongside the nail to create a narrow canal for the nail. This allows for multiple points of fixation and realignment of the bone as the nail is passed.

     

    Lindvall et al looked at a series of 56 extra-articular proximal tibial fractures treated with intramedullary nailing or percutaneous locked plating. Neither nailing or plating these fractures showed a distinct advantage in the overall outcomes. Apex anterior malreduction however was the most prevalent form of malreduction in both groups.

     

    Lang et al. looked at a series of proximal third tibia fracture treated with intramedullary nailing. They reported that posterior tibial comminution will also contribute to apex anterior angulation. They states this occurs when the fracture hinges on the intact cortex anteriorly during nail insertion.

     

    Figure A shows a proximal one third tibia fracture treated with an intramedullary nail. There is malreduction of the fracture with valgus and procurvatum deformity.

     

    Incorrect Answers:

    Answer 1: would prevent varus malalignment Answer 2: would prevent valgus malalignment alone

    Answer 3: would prevent varus and recurvatum malalignment Answer 4: would prevent varus and procurvatum malalignment

     

     

     

    OrthoCash 2020

     

  59. When evaluating a fracture dislocation of the elbow, a varus and posteromedial rotation mechanism of injury typically results in what injury pattern?

    1. A fracture of the radial head requiring ORIF

    2. A highly comminuted radial head fracture requiring radial head arthroplasty or resection

    3. An MCL injury requiring repair

    4. A type I avulsion fracture of the coronoid

    5. An anteromedial coronoid fracture

    Corrent answer: 5

     

    A varus and posteromedial rotation mechanism of injury typically results in a fracture of the anteromedial facet of the coronoid which frequently requires reduction and fixation to restore stability.

     

    A varus and posteromedial mechanism of injury about the elbow presents with an injury pattern distinctly different from other injury patterns. A key part of treating this injury pattern is recognizing a fracture of the anteromedial facet of the coronoid, which often requires reduction and fixation to restore stability about the elbow. It is important to recognize this during preoperative planning since this injury typically requires a medial approach.

     

    Steinman presents a review article describing coronoid fracture patterns and their mechanisms of injury.

     

    Doornberg and Ring present a level 4 review showing that coronoid fracture patterns and their required treatments are predictable based on mechanism of injury. Varus and posteromedial mechanisms were found to reliably create a fracture of the anteromedial facet of the coronoid, and were associated with sparing of the MCL and radial head.

     

    Doornberg and Ring also presented a Level 3 review of anteromedial facet cornoid fractures. They found that they could not be adequately visualized and treated from a lateral approach, and that they typically required reduction and fixation to restore adequate stability to the elbow. This stresses the importance of recognizing this injury pattern during preoperative planning.

     

    Illustrations A and B are AP and lateral radiographs of an elbow following a varus/posteromedial injury with an anteromedial coronoid facet fracture.

    Illustration C is a diagram demonstrating fracture lines that create an anteromedial facet fracture fragment. This fracture can be subclassified into three subtypes [anteromedial rim (a), rim plus tip (b), and rim and tip plus the sublime tubercle (c)]

     

    Incorrect answers:

     

    Answers 1, 2, 3: Radial head fractures, and MCL injuries are not seen in varus/posteromedial mechanisms. They are seen in terrible triad elbow injuries which are due to valgus and posterolateral rotatory forces.

    Answer 4: The coronoid fracture seen in these injuries is an anteromedial facet fracture, and not an avulsion fracture of the tip of the coronoid.

     

     

     

     

     

     

     

     

     

     

    OrthoCash 2020

     

  60. A 26-year-old female presented to the emergency department with the injury seen in Figure A after an awkward fall while intoxicated. She undergoes closed reduction and repeat radiographs are seen in Figure B. After a normal physical examination, including ranging the hip from 0-90 degrees, which of the following options would be the next most appropriate step in management?

     

     

     

     

     

     

    1. Femoral skeletal traction

    2. CT scan

    3. Hip spica cast application

    4. Observation only

    5. Serial neurovascular examinations Corrent answer: 2

    This patient has presented with a posterior left hip dislocation. The next most appropriate step would be performing a post-reduction CT scan to assess for joint reduction and congruity, associated fractures, or loose bodies.

    Hip dislocations occur most commonly in young patients with high energy trauma. They are usually classified as simple or complex, with complex dislocations being those with associated fractures of the acetabulum or proximal femur. Urgent close reduction should occur within 6 hours from the time of injury. Post reduction CT scans must be performed for all traumatic hip dislocations to look for fractures or impacted areas of the femoral head or acetabulum, as well as incongruent reductions and free intra-articular joint fragments.

     

    Calkins et al. looked at measurements of the posterior acetabulum on CT scans (the Acetabular Fracture Index) after hip dislocations to evaluate for hip stability. Hips were found to be unstable if less than 34% of the remaining posterior acetabulum was present after dislocation. Hips with greater than 55% were stable. In between 34 and 55% were indeterminate.

     

    Moed et al. found that posterior wall fractures involving less than 20% of the posterior wall were considered stable. Fractures involving more than 40%-50% were unstable, leaving a wide range of posterior wall fractures classified as indeterminate.

     

    Figure A shows a left posterior hip dislocation. Figure B shows a reduced left hip with no obvious fracture. Illustration A shows an axial CT scan image of the left hip. There is no acetabular fracture identified.

     

    Incorrect Answers:

    Answer 1: Femoral skeletal traction would be considered if there was an associated acetabular fracture or loose body.

    Answer 3: Hip spica casts are not used with these injuries.

    Answer 4: Observation would be considered after the CT scan is performed. Answer 5: There was no mention of neurovascular injury. This patient has a normal neurovascular examination. There is no fracture or risk of hematoma formation. This would not be necessary.

     

     

     

     

     

    OrthoCash 2020

     

  61. A 40-year-old male sustained the injury seen in Figure A, and subsequently underwent the procedure shown in Figure B. One hour post-operatively he starts to complain of pain in the operative leg, and the pain is unchanged with active or passive stretch. The external dressing is released with little resolution of symptoms. His blood pressure is 115/78 mm Hg with compartment pressures in the leg measuring 31 to 35 mm Hg. His ABI index is 1.1 in the leg. What would be the next step in management?

     

     

     

     

     

     

    1. MRI angiography of leg

    2. Four-compartment fasciotomy

    3. Follow-up examination the following day

    4. Continued monitoring and serial examinations

    5. EMG study

     

    Corrent answer: 4

     

    The patient is at risk for developing compartment syndrome of the leg. The next most appropriate step would be to support his systemic blood pressure and monitor compartment pressures.

     

    A clinical assessment is the diagnostic cornerstone of acute compartment syndrome. However, the intracompartmental pressure measurement has been advocated to help confirm the diagnosis in patients where there remains uncertainty after clinical exam. An absolute compartment pressure >30 mm Hg or a difference in diastolic pressure and compartments pressure (delta p) <30 mm Hg may help to confirm the necessity for fasciotomy. However, the treatment of early compartment syndrome should be to initially improve the limbs perfusion pressure gradient. This can be done by treating underlying factors such as hypotension, coagulopathy, or vascular compromise due to either a true vascular injury or artificially by external compression. Frequent reassessment is then critical to effectively manage these patients. If clinical

    diagnosis persists despite these efforts, urgent fasciotomy would be considered.

     

    McQueen looked at 116 patients with tibial diaphyseal fractures who had continuous monitoring of anterior compartment pressure for 24 hours. They found that using an absolute pressure of 30 mmHg would have resulted in 50 patients (43%) treated with unnecessary fasciotomies. They conclude using a differential pressure of 30 mmHg is a more reliable indicator of compartment syndrome.

     

    White et al. looked at 101 patients with tibial fractures with satisfactory Delta P measurements. THey found that patients with elevated intramuscular pressures >30 mm Hg after tibial fracture do not have a greater incidence of complications than those with low pressures, so long that Delta P <30 mm Hg.

     

    Figure A shows a Shatzker V tibial plateau fracture. Figure B shows fixation of fracture seen in Figure A.

     

    Incorrect Answers:

    Answer 1: MRI scan of the leg is not needed as the ABI=1.1

    Answer 2: Fasciotomy would be considered after serial examinations with delta p < 30 mm Hg or persistently elevated absolute compartment pressures..

    Answer 3: It would not be appropriate to leave this patient with impending compartment syndrome.

    Answer 5: EMG studies help to assess neurological injury. This would not be necessary in the acute setting.

     

     

     

    OrthoCash 2020

     

  62. A 10-year-old girl suffers a displaced tibia fracture. Initial numbness over the dorsum of the the foot resolved following an anatomic closed reduction and placement in a long leg cast performed in the emergency room. The cast was placed with the the ankle dorsiflexed just above neutral to prevent equinus contracture and then the cast and padding was adequately bivalved. Overnight, the patient began experiencing recurrent numbness and paresthesias in her exposed toes and a slight increase in her pain at the fracture site. Your next best step would be:

    1. Repeat closed reduction under conscious sedation

    2. Selective compartment fasciotomies

    3. External fixation and compartment monitoring

    4. 4-compartment fasciotomies with fracture fixation done emergently

    5. Modify the cast to reposition the ankle into slight plantarflexion Corrent answer: 5

    Circumferential casting with the ankle dorsiflexed can cause increased intracompartmental pressures in the leg. However, this patient's cast has been adequately bivalved during initial cast placement. Therefore, the next best step is cast modification to allow the ankle to assume an angle between neutral and 30 degrees of plantar flexion and further reducing the compartment pressure.

     

    Tibia fractures are one of many common underlying etiologies for the development of compartment syndrome in the leg. Fracture reduction as well as eliminating circumferential dressings are important early preventative steps to take. Bivalving casts - including splitting the cast padding - is often indicated in fractures of long bones that are treated with initial casting. While casting patients in a plantigrade or dorsiflexed position reduces the risk of equinus contractures and holds the ankle mortise reduced, dorsiflexing the ankle has been shown to increase the intracompartmental pressures throughout the leg compartments.

     

    Weiner et al. placed pressure monitors into the anterior and deep posterior compartments in healthy volunteers to measure the effects of casting on compartment pressures. They found that the intramuscular pressures were lowest with the ankle positioned between plantigrade and a resting plantar flexion position (0-37 degrees), and that bivalving the cast reduced the pressures 33-47%.

     

    Illustration A shows the needle trajectories for compartment pressure monitoring in the leg. This should be performed within 5 cm of the fracture to get accurate peak pressures.

     

    Incorrect Answers:

    Answers 1, 3: The stem does not suggest that the reduction was lost, so neither repeating a closed reduction or placement of an external fixator is indicated at this time.

    Answers 2, 4: While suspicion should remain elevated for the development of compartment syndrome, simple conservative measures such as cast modification are ideal first steps to avoid onset of compartment syndrome. The clinical vignette given does not portray clinical compartment syndrome requiring emergent fasciotomy. It would be reasonable to maintain a high level of suspicion given the injury, but non-invasive cast modification is a simple, safe and quick first step.

     

     

     

     

     

    OrthoCash 2020

     

  63. A 35-year-old male horseback rider was bucked into the air and then landed forcefully with his perineum on the saddlehorn of the saddle. At a one year follow-up, the only long term sequela of his injuries is erectile dysfunction. Which radiographic injury seen in Figures A-E is most commonly associated with this complication?

     

     

     

     

     

     

     

     

     

     

     

     

     

     

    1. Figure A

    2. Figure B

    3. Figure C

    4. Figure D

    5. Figure E

     

    Corrent answer: 1

     

    This patient has sustained a saddle-horn injury to the pelvis. An anterior posterior compression (APC) injury is the most common pelvic injury associated with this mechanism.

     

    Saddle-horn injury to the pelvis occurs when a horseback rider is bucked into the air and then lands forcefully with his or her perineum on the saddlehorn of the saddle. The bony injury to the pelvis usually consists of a diastasis of the pubic symphysis with/without subsequent widening of the sacroiliac joints. The

    majority of patients are able to return close to their pre-injury level of employment and activity after these injuries. However, sexual dysfunction is a common complication with this injury. Sexual dysfunction can be due to urethral, vascular, neurologic, and psychogenic injuries.

     

    Collinge et al. looked at a series of male patients with injuries to the perineum after coming into contact with the saddle or saddle horn on a horse. Eighteen of the twenty patients were found to have sexual dysfunction at the time of the latest follow-up. A multi-disciplinary approach to these patients is needed when treating pelvic injuries.

     

    Cannada et al. surveyed 71 women of childbearing age who had a pelvic fracture. She found that half the patients reported physical genitourinary complaints with 38 percent of women noting new onset pain with intercourse. In terms of psychological impact, they showed that 45 percent of patients answered affirmatively to decreased interest in intercourse and decreased orgasm frequency after the injury.

     

    van Nieuwenhoven et al. describe three equestrians who suffered from straddle injuries with symphysis diastasis and rupture of sacroiliac ligaments after falls or kicks from horses. They show that these presumed low-energy injuries can cause extremely severe injuries to the pelvis itself and to the adjacent organs.

     

    Figure A shows a Burgess APC Type II pelvic fracture. Figure B shows left ASIS avulsion fracture. Figure C shows a left hip dislocation. No pelvic fracture is identified. Figure D shows a subacute superior/inferior pubic rami fracture.

    Figure E shows a right femoral neck fracture.

     

    Incorrect Answers:

    Answers 2,3,4,5: Any pelvic or lower limb fracture can cause a disruption in a patients activities of daily living. This may include sexual dysfunction, for both men and women. The injury pattern that has the greatest potential for sexual dysfunction are APC type pelvic injuries.

     

     

     

    OrthoCash 2020

     

  64. A 31-year-old male driver was involved in a high-speed motor vehicle accident. His injuries include a left subdural hematoma (Abbreviated Injury Score [AIS]=4), left segmental femur fracture (AIS=3), ruptured spleen (AIS=4), nasal fracture (AIS=2), fractured left ribs 4 to 7 (AIS=2), and a closed pelvic ring fracture (AIS=3). What is his Injury Severity Score (ISS)?

    1. 18

    1. 25

    2. 33

    3. 41

    4. 48

     

    Corrent answer: 4

     

    This patients Injury Severity Score (ISS) is 41. In this patient, the highest AIS scores were from the head (AIS=4), abdomen (AIS=4) and pelvis (AIS=3).

    Therefore, the calculation for the ISS score was 4^2 + 4^2 + 3^2 = 41.

     

    The ISS score is calculated as the sum of the square values from the highest Abbreviated Injury Score [AIS] scores in the three most severely injured ISS body regions. ISS scores range from 1 to 75. A single score of 6 on any AIS region results in automatic score of 75. ISS scores > 15 are associated with mortality of 10%.

     

    Poole et al. found that the AIS and ISS appropriately reflect the impact of extraskeletal injuries in patients with femur fractures, but they do not adequately reflect the increased morbidity associated with multiple lower extremity fractures.

     

    Balogh et al. aimed to compare the scoring efficacy of the injury severity score (ISS) and the new injury severity score (NISS) in predicting post-injury outcomes. They found that multiple orthopaedic injuries, as used in the NISS system, had a significantly greater impact on reported poor post-injury outcomes. They suggest that NISS should replace the traditional ISS when grading trauma related outcomes.

     

    Illustration A shows a table of the various ways to score injury severity in trauma patients.

     

    Incorrect Answers:

    Answers 1,2,3,5: ISS = A^2 + B^2 + C^2, where A, B, C are the AIS scores of the three most severely injured ISS body regions. The only correct answer is ISS=41.

     

     

     

     

     

    OrthoCash 2020

     

  65. A 60-year-old otherwise healthy female sustains the fracture in Figure A. Immediate surgical treatment would most likely prevent which of the following complications?

     

     

     

     

    1. Malunion.

    2. Regional osteopenia.

    3. Skin necrosis.

    4. Posttraumatic arthritis.

    5. Refracture.

     

    Corrent answer: 3

     

    The patient has a calcaneal tuberosity fracture. The Achilles tendon is attached to the displaced fragment, which can cause soft tissue compromise and skin necrosis if not treated promptly.

     

    Displaced calcaneal tuberosity and tongue-type calcaneus fractures have a high incidence of wound complications if not treated urgently. Recommended management includes surgical fixation often performed percutaneously. Lag screws are typically directed from the posterior superior tuberosity inferiorly and distal. In cases of skin sloughing and skin necrosis, flaps may be needed.

     

    Gardner et al. reviewed 139 tongue-type calcaneus fractures, and found that 21% had some degree of posterior skin compromise. Additionally, there was a statistically significant increase in posterior skin compromise with delayed presentations.

     

    Hess et al. reviewed 3 cases of calcaneal tuberosity avulsion fractures that let to skin necrosis because of a delay in treatment.

     

    Tornetta reviewed the indications for percutenaous treatment of calcaneus fractures, and found it was successful and most useful in tongue-type calcaneus fractures.

     

    Figure A shows a lateral radiograph of a R ankle showing a calcaneal tuberosity fracture. Illustration A shows an AP radiograph of a R ankle following percutaneous reduction and internal fixation of the calcaneus fracture with two cannulated screws. Illustration B shows a lateral radiograph of a R ankle showing interval reduction and fixation of the calcaneus fracture. Illustration C shows a clinical photograph of posterior skin compromise associated with a calcaneal tuberosity fracture.

     

    Incorrect Answers:

    Malunion, regional osteopenia, posttraumatic arthritis, and refracture risk would not be significantly altered with immediate surgical intervention as opposed to operative management within a reasonable time frame.

     

     

     

     

     

     

     

     

     

     

    OrthoCash 2020

     

  66. When elevating the joint surface in the injury pattern seen in Figure A, what material has the highest compressive strength when filling the metaphyseal void?

     

     

     

    1. Calcium phosphate

    2. Tricalcium phosphate

    3. Cancellous autograft

    4. Cancellous allograft

    5. rhBMP-7

     

    Corrent answer: 1

     

    Figure A illustrates a depressed lateral tibial plateau. One of the key components of fixing a depressed articular segment is maintaining the reduction.

     

    Trenholm et al reviewed an experimental study where a split depression (Schatzker II) fracture was created in a cadaveric model. The stiffness of the elevated fragment in cadavers using calcium phosphate cement versus cancellous bone graft as support showed no difference, but calcium phosphate cement was found to have greater compressive strength than cancellous bone alone.

     

    The review article by Hak reviews the composition, advantages, and disadvantages of commerically available bone graft substitutes.

     

    Incorrect answers:

    2: Tricalcium phosphate is a bone graft substitute that is osteoconductive, but has less compressive strength.

    3-4: Cancellous bone has less compressive strength than calcium phosphate. 5: rhBMP-7 is not used as a bone filling agent in this instance, as it is typically provided on a easily compressible collagen sponge.

     

     

     

    OrthoCash 2020

     

  67. A 42-year-old male sustains multiple abdominal injuries along with the fractures shown in Figures A and B. Which of the following factors is most useful to determine the patient's resuscitation and ability to undergo early definitive fixation of these two fractures?

     

     

     

     

     

     

    1. Pulse pressure

    2. Respiratory rate

    3. Urine output

    4. Platelet count

    5. Base deficit

     

    Corrent answer: 5

     

    The base deficit, or lactate level, has been shown to best reflect the resuscitation status and survival after trauma.

     

    Normalization of hemodynamic parameters does not accurately reflect the resuscitation status and a patient can be in compensated shock (occult tissue hypoperfusion) despite normalization of the heart rate and blood pressure. The use of temporizing measures with delayed definitive fracture treatment has been shown to decrease systemic complications in these patients with occult hypoperfusion.

     

    Crowl et al. performed a retrospective study of 127 patients with femoral shaft fractures undergoing intramedullary nailing, assessing for occult hypoperfusion. Patients with occult hypoperfusion preoperatively had a twofold increase in postoperative complications and significant increase in postoperative infections.

     

    Tisherman et al. present a review of hemodynamic parameters and their use in assessment of resuscitation. They report that hyperlactatemia is common among critically ill patients, and lactate levels and their trend may be reliable markers of illness severity and mortality. Therefore, measurement of blood lactate is recommended to stratify patients based on the need for fluid resuscitation and the risks of multiple organ dysfunction syndrome and death. They conclude that lactate is a reliable indicator of sepsis severity and a marker of resuscitation; however, it is an unreliable quantitative marker of tissue hypoxia/hypoperfusion.

     

    Pallister et al. present a review of the effect of early stabilization of long bone fractures. They note that early stabilization of major long bone fractures is beneficial in reducing the incidence of acute respiratory distress syndrome and multiple organ failure, both of which are caused by activation of the systemic inflammatory response. This activation results in tissue recruitment of and injury by circulating polymorphonuclear leukocytes.

     

    Figure A is a radiograph showing a femoral shaft fracture, while Figure B shows an open tibial shaft fracture.

     

    Incorrect Answers:

    Answers 1-4: Normalization of these factors does not always predict adequate resuscitation.

     

     

    OrthoCash 2020

     

  68. Interleukin-6 levels have been shown to be a reliable measure of which of the following?

    1. Osteoporosis

    2. Severity of injury

    3. Sarcomatous tumor burden

    4. Bone turnover

    5. Hyperparathyroidism

     

    Corrent answer: 2

     

    Interleukin-6 (IL-6) is a proinflammatory cytokine that is increased according to the level of injury sustained and acts to activate the host immune system. In addition, new literature exists to indicate its high sensitivity and specificity for detecting infection in total joint arthroplasty.

     

    Overactivation of the immune system and cytokines can lead to systemic inflammatory response syndrome (SIRS), which results in end-organ damage, including small-vessel vascular damage; this would lead to parenchymal cell death from hypoxic insult.

     

    Keel et al. report that immediate and early trauma deaths are determined by brain injuries or significant blood loss, while late mortality is caused by secondary brain injuries and host defense failure. The secondary effects are characterized by local and systemic release of pro-inflammatory cytokines, arachidonic acid metabolites, proteins of the contact phase and coagulation systems, complement factors and acute phase proteins, as well as hormonal mediators.

     

    Pape et al. investigated the effect of surgeries as a "second-hit" phenomenon. They found that surgery on days 2-4 was associated with a greater amount of postoperative organ dysfunction than if the secondary surgery was done on days 5-8. They also found a significant association between IL-6 values above 500pg/dL at the time of surgery and development of multiple organ failure.

     

    Sears et al. review the effect of the inflammatory response to trauma and the development of complications (death, multiple organ failure). They report that IL-6 and the HLA-DR2 molecules currently appear to have the most potential for use in predicting outcomes in trauma patients.

     

    Illustration A is a diagram that shows some of the effects of IL-6 production.

    Incorrect Answers:

    1,3-5: IL-6 levels are not known to be reliably altered by these factors.

     

     

     

     

     

     

    OrthoCash 2020

     

  69. A 31-year-old male sustained a displaced proximal humerus fracture after a motor vehicle accident. Which of the following factors has the lowest association with humeral head ischemia in these injuries?

    1. Four-part fracture

    2. Head-splitting fracture

    3. Neck fracture with a long calcar segment

    4. Disrupted medial periosteal hinge

    5. AO type C3 fracture Corrent answer: 3

    A long calcar fracture segment, also known as the posteromedial metaphyseal head extension, is a good prognostic sign against the development of osteonecrosis in displaced proximal humerus fractures.

     

    The humeral head is directly supplied by the anterior and posterior humeral circumflex arteries. Disruption of these vessels can result in humeral head osteonecrosis. Fracture patterns with long medial metaphyseal segments (greater than 8 mm) have not shown to critically disrupt the vascular supply to the humeral head causing osteonecrosis. Other factors shown to be protective against osteonecrosis after trauma include an intact medial hinge and simple fracture patterns.

     

    Hertel et al. examined the factors predictive of osteonecrosis of the humeral

    head after fracture. These include four-part fragments, angular displacement

    >45 degrees of the head, displacement of the tuberosities greater than 10 mm, glenohumeral fracture-dislocation, and head-splitting fractures.

     

    In Hertel's second paper and follow up study they noted that the initial post-fracture humeral head ischemia does not predict development of necrosis.

     

    Illustration A shows an example of a long and short calcar fracture segment. Illustration B shows the AO classification of proximal humerus fractures.

     

    Incorrect Answers:

    Answers 1,2,4,5: All these factors are predictive of osteonecrosis.

     

     

     

     

     

     

     

     

     

     

    OrthoCash 2020

     

  70. A 50-year-old female has been diagnosed with osteomyelitis of her left tibia. The infection is isolated to the meduallary canal of the bone, and her past medical history is significant for heavy smoking and chronic venous stasis. Based on the clinical staging of osteomyelitis, what would be her Cierney-Mader classification?

    1. Stage 2A

    2. Stage 1B

    3. Stage 1C

    4. Stage 3B

    5. Stage 4C

     

    Corrent answer: 2

     

    The Cierny-Mader classification is a clinical classification based on clinical and anatomic features. This patient's Cierny-Mader classification is Stage 1B.

     

    The Cierney-Mader classification system characterizes osteomyelitis in one of four anatomic stages. Stage 1 is confined to the medullary cavity of the bone. Stage 2 involves only cortical bone. Stage 3 is a localized infection to cortical and medullary bone. However, the bone remains stable. Stage 4 involves the entire thickness of the bone, with loss of bone stability. The Cierny-Mader system also sub-characterizes patients as A, B, or C. The [A} patients have no systemic or local immune compromising factors. The [B] patients have one or more systemic or local compromising factors. The [C] patients are severely immunocompromised and treatment will lead to greater morbidity than the infection.

     

    Mader et al. described the treatment algorithm of osteomyelitis using the Cierny-Mader classification. Treatment involves both antibiotic and surgical debridement. For type 1 infection, thorough intramedullary reaming and unroofing is usually performed.

     

    Illustration A shows a table for the Cierny-Mader classification. Illustration B shows a schematic illustration of the anatomical involvement of osteomyelitis using the Cierny-Mader classification.

     

    Incorrect Answers:

    Answer 1: This would be a cortical infection only in a healthy patient. Answer 3: This would be a medullary cavity infection only in a severely compromised patient.

    Answer 4: This would be a localized infection to cortical and medullary bone in a patient with one or more systemic or local compromising factors.

    Answer 5: This would be a diffuse infection involving the entire bone in a severely compromised patient.

     

     

     

     

     

     

     

     

    OrthoCash 2020

     

  71. A 68-year-old female sustains a fall to her dominant arm and sustains the fracture seen in Figure A. She undergoes uneventful open reduction internal fixation with a locked plate and screw construct. Which of the following radiographs (Figures B, C, D, E, or F) show the most common complication associated with this treatment modality?

     

     

     

     

     

     

     

     

     

     

     

     

     

    1. Figure B

    2. Figure C

    3. Figure D

    4. Figure E

    5. Figure F

     

    Corrent answer: 1

     

    Figure B shows screw penetration of the articular surface, which is the most common complication following locked plate and screw fixation.

     

    Inadequate fixation, typically in the humeral head, subsequently fails leading to varus deformity and screw penetration. This most commonly occurs in fixation for 3 and 4 part fractures, but can also occur following fixation for 2 part fractures.

     

    Egol et al. retrospectively reviewed 51 patients following locked plating of 3-and 4-part proximal humerus fractures, they found 16% had screw penetration. The authors recommended ensuring appropriate number and length of the screws placed to prevent cut-out.

     

    Brunner et al. reviewed 158 proximal humerus fractures. They found at 1 year follow-up noted 14% screw penetration rate. They recommended close followup to monitor for cut-out.

     

    Figure A shows a 3-part proximal humerus fracture. Figure B exhibits varus cut-out and screw penetration. Figure C exhibits non-union and fixation failure. Figure D exhibits avascular necrosis and complete head collapse. Figure E (arrow) exhibits changes consistent with heterotopic ossification.

    Figure F exhibits non-union and fixation failure at the level of the shaft.

     

    Incorrect answers:

    Answer choice 2/5 (figure C/F): Non-union and fixation failure can also occur, but at a relatively low rate.

    Answer choice 3 (Figure D): Avascular necrosis also occurs, but at a lower rate than screw penetration.

    Answer choice 4 (Figure E): Heterotopic ossification (arrow) can occur but is rare.

     

     

     

    OrthoCash 2020

     

  72. Figure A shows an acute, isolated and closed, left knee injury in a 40-year-old male struck by a motor vehicle. What would be the most

    appropriate surgical fixation for this injury?

     

     

     

     

    1. Definitive external fixation

    2. Temporary external fixation then lateral percutaneous screws

    3. Lateral nonlocking plate +/- bone graft substitutes

    4. Medial and lateral locking plate +/- bone graft substitutes

    5. Lateral percutaneous screws with assisted arthroscopy Corrent answer: 3

    This patient has sustained a Schatzker 2 (AO/OTA Type B) fracture of the lateral tibial plateau. The most appropriate treatment of this fracture would be fixation with a lateral nonlocking plate +/- bone graft substitute.

     

    Displaced, widened and depressed tibia plateau fractures are an indication for operative intervention. The aim of surgery is to restore anatomic articular congruity, length, alignment and rotation of the proximal tibia. This is best achieved by (1) direct reduction of the articular surface, and (2) fixation of the interfragmentary piece with a plate and screw construct, which provides subarticular support, interfragmentary compression, and prevents shearing of the fracture. In patients with good bone quality, this is best achieved with an undercontoured lateral nonlocking plate in buttress mode.

     

    Karunakar et al. showed that there was no significant difference between split depression tibial plateau fractures (Shatzker II) fixed with either (1) buttress plate with rafting screws versus (2) periarticular plate with built in rafting screw hole options.

     

    Gardner et al. examined 62 consecutive Schatzker type II fractures with radiographs and MRIs preoperatively. They found that joint depression greater than 6mm and widening of greater than 5mm was associated with a lateral meniscal injury over 80% of the time.

     

    Figure A provided a sequence of coronal CT scan images of a left knee. There is a lateral split depression tibial plateau fracture with significant joint

    depression, articular impaction and condylar widening.

     

    Incorrect Answers:

    Answer 1: Definitive external fixation of tibial plateau fractures may be considered in patients who are unable to tolerate definitive surgery, severe open fractures, infections, or significant bone loss.

    Answer 2: Temporary external fixation would be considered in patients with significant soft tissue injury.

    Answer 4: Locking plates do not provide buttress effect when used in pure locking mode. In addition, locking plates add significant incremental cost to the procedure. Medial plating would support a bicondylar fracture.

    Answer 5: Screws alone are unlikely to be adequately stable in this patient. Although not commonly employed, knee arthroscopy is considered by some surgeons to be a valuable method to assist in obtaining the best articular surface reduction during surgery.

     

     

     

    OrthoCash 2020

     

  73. An otherwise healthy young adult male sustains a transverse radial shaft and ulna fracture. He undergoes definitive surgical fixation with two non-locking compression plates (LCPs) as shown in Figure A. What is the principle of this fixation technique on bone healing?

     

     

     

     

    1. Absolute stability with direct healing by callus formation

    2. Relative stability with indirect healing by callus formation

    3. Absolute stability with direct healing by internal remodeling

    4. Relative stability with indirect healing by internal remodeling

    5. Absolute stability with endochondral bone formation Corrent answer: 3

    Definitive surgical fixation for a simple transverse both bone forearm fracture would include open reduction and internal fixation with absolute stability with direct healing by internal remodelling (i.e., primary bone healing). The radius is fixed with a 7-hole 3.5mm LC-DCP plate in compression mode. The ulna was fixed with a 8-hole 3.5mm LC-DCP neutralization plate with (2) lag screws.

     

    Restoration of the radial bow, length, and alignment in conjunction with primary bone healing would optimize the restoration of pronation/supination in the forearm. Compression plating and anatomic reduction of articular fractures are examples of absolute stability. Casting, bridge plating, external fixation, and intramedullary nailing provide relative stability, with secondary bone healing resulting.

     

    Moed et al. reviewed 56 patients with open diaphyseal forearm fractures treated with immediate internal fixation. The complications included deep infection in 2 patients and non-union in 6. This study indicated immediate stable plate fixation was a beneficial method of treatment for open diaphyseal forearm fractures.

     

    Jones et al. discussed current literature on treating adult diaphyseal forearm fractures with open reduction internal fixation versus intramedullary nailing, and concluded a randomized prospective study is needed. They stated current literature is inconclusive.

     

    Figure A shows a both bone forearm fracture fixed with absolute stability. The radius is fixed with a 7-hole 3.5mm LC-DCP plate in compression mode. The ulna was fixed with a 8-hole 3.5mm LC-DCP neutralization plate with (2) lag screws.

     

    Incorrect Answers:

    Answers 1-2, 4-5: Absolute stability is a biological process of osteonal bone remodelling. This is predominantly intramembranous ossification. Enchondral bone formation occurs in non-rigid fracture healing (secondary bone healing).

     

     

     

    OrthoCash 2020

     

  74. A 27-year-old man sustains a tibial plateau fracture as pictured in Figure A-D. Which combination of fixation constructs and stability

    would optimize his outcome?

     

     

     

     

     

     

     

     

     

     

     

    1. Intramedullary nailing for a relative stability construct

    2. Posteromedial buttress plating for an absolute stability construct

    3. Locked lateral plating an absolute stability construct

    4. Medial antiglide plating for a relative stability construct

    5. Spanning external fixation for an absolute stability construct Corrent answer: 2

    Intraarticular tibial plateau fractures require absolute stability constructs to maintain anatomic joint reductions without callus formation. This condylar split fracture is ideally treated with buttress fixation at the apex of the fracture to avoid the high likelihood of shear failure with a laterally placed fixed-angle construct.

     

    Tibial plateau fractures in young adults are high energy injuries, and the condylar split components of the fractures occur due to shear failure. The mechanism of failure - axial loading combined with a varus or valgus moment with the knee extended or flexed - dictates the orientation of the condylar split fracture line(s). There are multiple fracture classification schemes that assist with surgical decision making, particularly with placement of buttress plate fixation to resist shear failure. Articular impaction and joint depression must also be addressed.

     

    Bai et al. performed a biomechanical study on fresh cadavers to simulate residual articular step-offs after fixation. With a >5 mm step-off, contact pressures increased over 200%.

     

    AO guidelines of tibial plateau fracture management are discussed in depth in AO Principles of Fracture Management, and up-to-date principles can be found on the AO surgery reference. Anatomic reduction and absolute stability are important elements of all intraarticular proximal tibia fractures.

     

    Figures A and B are injury films of a tibial plateau fracture with a large posterior split of the medial condyle. Figure C shows an axial CT cut displaying the same fracture. Illustrations A-C show how the injury was treated. Through a posterior incision, an under-contoured anti-glide plate was used to reduce the fracture at the apex. Subsequently, a pre-contoured posteromedial buttress plate was placed to maintain the reduction. The articular reduction was finalized with rafting screws outside of the plating construct to optimize the angle of subchondral support.

     

    Incorrect Answers:

    Answers 1, 4: Relative stability is inappropriate for intraarticular fractures that require anatomic reductions

    Answer 3: Laterally-based locked plating for a medial condyle fracture creates large moments about the screw-plate interface, and the construct is apt to fail in shear

    Answer 5: Spanning external fixation is most commonly used for temporizing tibial plateau fractures when significant soft tissue is present - in the absence of additional fixation for a hybrid construct, it would provide relative stability

     

     

     

     

     

     

     

     

     

     

    OrthoCash 2020

     

  75. Surgical fixation with absolute stability would be most appropriate for which of the following fracture patterns?

     

     

     

     

     

     

     

     

     

     

     

     

     

    1. Figure A

    2. Figure B

    3. Figure C

    4. Figure D

    5. Figure E

     

    Corrent answer: 2

     

    Anatomic reduction and fixation with absolute stability is appropriate for articular fractures, such as posterior malleolar ankle fractures with significant joint involvement.

     

    Compression plating and anatomic reduction of periarticular fractures yield absolute stability. Fractures treated with absolute stability achieve primary bone healing. Posterior malleolar ankle fractures comprising greater than 25% of the joint and non-comminuted lateral malleolar ankle fractures are typically treated with internal fixation to achieve absolute stability. Casting, bridge plating, external fixation, and intramedullary nailing provide relative stability, with secondary bone healing resulting. In fractures with significant comminution and in lower extremity diaphyseal long bone fractures, relative stability is employed.

     

    Gardner et al. surveyed members of the Orthopaedic Trauma Association (OTA) and American Orthopaedic Foot and Ankle Society (AOFAS) regarding treatment of posterior malleolus ankle fractures. They found that size influenced decision making, with 97% electing to treat those posterior malleolus fractures that comprised 50% of the joint. The most common used

    approach was the posterolateral approach, which utilizes the interval between the flexor hallucis longus and peroneals.

     

    Forsberger et al. reviewed 45 consecutive patients treated with surgical fixation of the posterior malleolus through the posterolateral approach. This approach allowed excellent exposure with few soft tissue complications.

     

    De Vries et al. reviewed 45 patients with ankle fractures that had a posterior malleolar component. Those patients that had the posterior malleolar fragment fixed did not have statistically better functional scores than those who did not have the fragment fixed. They concluded that those patients with fragment sizes less than 25% did not require fixation and had good functional outcomes at 13 year follow-up.

     

    Figure A shows an AP and lateral radiograph of a right comminuted distal third radial shaft fracture. Figure B shows an AP and lateral radiograph of a lateral and posterior malleolar ankle fracture in addition to a deltoid rupture. Figure C shows an AP and lateral radiograph of a comminuted middle third tibial shaft fracture. Figure D shows an AP radiograph of a minimally displaced greater trochanter fracture. Figure E shows an AP and lateral radiograph of a pediatric distal radial metaphyseal fracture. Illustration A shows an AP, mortise, and lateral radiograph of a posterior and lateral malleolus ankle fracture treated with open reduction and internal fixation.

     

    Incorrect answers:

    Answer 1: Significant comminuted radial shaft fractures are treated with bridge plating, which would achieve relative stability.

    Answer 3: Diaphyseal tibial fractures are treated with intramedullary nailing, which achieves relative stability.

    Answer 4: Minimally displaced greater trochanter fractures are treated nonoperatively.

    Answer 5: Pediatric distal radius fractures are typically treated with closed reduction and casting, which achieves relative stability.

     

     

     

     

     

    OrthoCash 2020

     

  76. Figures A and B are radiographs of a 72-year-old male who presented to hospital 3 days after an unwitnessed fall. He presents with pain and the inability to mobilize since the fall. The patient lives independently and has no significant past medical history. Physical examination reveals an alert and oriented patient. This is an isolated injury with no other focal or systemic deficits. What would be the most important investigation to be obtained before taking this patient to the operating room?

     

     

     

     

     

     

    1. Pelvic inlet and outlet views

    2. CT head

    3. Duplex ultrasound of both lower extremities

    4. CT hip and pelvis

    5. MRI hip and pelvis Corrent answer: 3

    The most appropriate investigation to order in this patient would be a duplex ultrasound of both lower extremities.

     

    Immobilized elderly patients with a delay of more than two days from the fracture to presentation at the hospital are at a higher risk of deep-vein thrombosis (DVT). The risk can be higher in patients with factors that predispose to clot formation, such as malignancy. Patients with a delayed presentation to hospital should be investigated for DVT. The most common screening modality is duplex ultrasound of both lower extremities.

     

    Hefley et al. looked at the effect of delayed admission to the hospital on the preoperative prevalence of DVT associated with hip fractures. They found that 6 out of 11 patients with a delay of more than two days between the fracture and admission to the hospital had evidence of thrombosis. They conclude all patients with delayed admission, after a hip fracture, should be evaluated for DVT preoperatively.

     

    Figure A and B are AP and lateral radiographs that show a displaced left subcapital hip fracture.

     

    Incorrect Answers:

    Answer 1: There is no indication of pelvic inlet and out views. Generally speaking, pelvic inlet and outlet views assess the pelvic ring and pelvic bone stock, which are normal in this patient.

    Answer 2: CT head would be indicated after an unwitnessed fall if the patient admitted to head injury, had neurological symptoms or acute focal/systemic deficits on exam. None of these are present in this patient.

    Answer 4 and 5: A CT or MRI of the hip and pelvis are not indicated in a displaced fracture, but may be useful in diagnosing nondisplaced fractures.

     

     

     

    OrthoCash 2020

     

  77. A 67-year-old female sustains the injury shown in Figure A after a trip and fall. When discussing the outcomes of surgery with the patient, which of the following statements is true?

     

     

     

    1. Post-surgical mortality rates are significantly lower after total hip arthroplasty compared to hemiarthroplasty

    2. Internal fixation shows better outcomes (reoperation rate, functional status, and/or complication rates) compared to arthroplasty

    3. Bipolar hemiarthroplasty shows better outcomes (reoperation rate, functional status, and/or complication rates) compared to unipolar hemiarthroplasty

    4. A delay in surgery greater than 48 hours is acceptable if the patient has multiple medical comorbiditiesm which are not fully optimized

    5. Dislocation rates are equivalent between total hip arthroplasty and hemiarthroplasty

    Corrent answer: 4

     

    Moderate evidence supports that hip fracture surgery within 48 hours of admission is associated with better outcomes. However, patients with significant medical comorbidity should be fully optimized before surgery.

     

    Although several studies have shown a benefit to surgery within 48 hours, no definitive time frame has been elucidated. The majority of literature has shown improved outcomes in regards to pain, complications, and length of stay with early surgery. Patients with significant medical comorbidities have been shown to have the highest mortality rates.

     

    Moran et al. aimed to determine whether a delay in surgery for hip fractures had an affect on postoperative mortality among elderly patients. In an observational study of 2660 patients, they showed that mortality following hip fracture surgery was 9% at 30-days, 19% at 90-days, and 30% at 12-months.

    Patients with medical comorbidities had 2.5 times the risk of death within 30-days of surgery. In addition, individuals who had surgery delayed beyond 4 days had increased mortality at 90-days and 12-months.

     

    Papakostidis et al. examined the timing of internal fixation of intracapsular fractures of the neck of femur on the development of late complications, particularly osteonecrosis of femoral head (ONFH) and non-union. They showed no benefit of early surgery on incidence of AVN. However, delay of internal fixation of more than 24 hours showed increased rates of non-union.

    Figure A shows a displaced right femoral neck fracture. Incorrect Answers:

    Answer 1: Patients treated with hemiarthroplasty or total hip arthroplasty

    demonstrated no statistically significant difference in mortality.

    Answer 2: Arthroplasty consistently has better outcomes compared to internal fixation for the treatment of unstable/displaced (Garden III and IV) femoral neck fractures in elderly patients.

    Answer 3: Outcomes of unipolar and bipolar hemiarthroplasty for unstable femoral neck fractures are similar.

    Answer 5: Dislocation rates are higher with total hip arthroplasty compared to hemiarthroplasty.

     

     

     

    OrthoCash 2020

     

  78. An 87-year-old female sustains the injury shown in Figure A after a fall from standing. At baseline, she ambulates with a walker in her home and lives with a 24-hour home health aide. She has a past medical history of stroke and mild dementia. Following medical optimization, what is the most appropriate treatment modality?

     

     

     

    1. Intramedullary nail

    2. Hemiarthroplasty

    3. Closed reduction percutaneous pinning

    4. Total hip arthroplasty

    5. Sliding hip screw Corrent answer: 2

    At baseline, with minimal ambulatory status and dependence on a 24-hour home health aide, the best treatment choice is hemiarthroplasty.

     

    Arthroplasty is the gold standard for displaced femoral neck fractures in the elderly. In community ambulators with relative independent lives, total hip arthroplasty (THA) is recommended. For minimal ambulator, hemiarthroplasty is recommended.

     

    van den Bekerom et al. randomized 252 patients over 70-years-old to either THA or hemiarthroplasty for displaced femoral neck fractures. With increased blood loss, operative time and dislocation risk, they did not recommend THA unless there was good preoperative ambulatory status, and/or pre-existing osteoarthritis or rheumatoid arthritis.

    Figure A exhibits a displaced femoral neck fracture

     

    Incorrect answers:

    Answers 1, 3, 5 are not the treatment standards for displaced femoral neck fractures in the elderly.

    Answer 4 is incorrect because the patient is minimally ambulatory at baseline with little independence.

     

     

     

    OrthoCash 2020

     

  79. A 20-year-old female arrives to the trauma bay following a motor vehicle accident. She was a restrained driver and has bilateral chest tubes placed in the trauma slot. Initial laboratory values are significant for a hematocrit of 45.5 g/dL and a lactate of 7 mmol/L. Both lower extremities exhibit grossly open wounds. Radiographs are shown in Figure A and B. What is the best initial management of her orthopaedic injuries?

     

     

     

     

     

     

    1. Irrigation and debridement of left femur and right tibia with bilateral external fixator placement

    2. Irrigation and debridement of left femur and right tibia with left external fixator placement and right tibial intramedullary nail placement

    3. Irrigation and debridement of left femur and right tibia with left femoral and right tibial intramedullary nail placement

    4. Irrigation and debridement of left femur and right tibia with left femur open reduction and internal fixation and right tibial intramedullary nail placement

    5. Irrigation and debridement of left femur and right tibia with bilateral long leg splint placement

    Corrent answer: 1

     

    Presenting as an unstable patient, damage control orthopaedics (DCO) should be performed by placing bilateral external fixators following irrigation and debridement.

     

    Signs of clinical instability include an elevated lactate level as well as injuries to both lungs, necessitating bilateral chest tubes. Furthermore, while the patient is young and her hematocrit reflects a 'normal' number, more likely is a hypovolemic state due to bilateral open long bone injuries. Resuscitation is required and early total care (ETC) should be avoided.

     

    Pape et al. review the early data regarding DCO and ETC. Identification of patients in extremis, borderline, unstable or stable patients is critical to determining appropriate management. Adequate resuscitation, by tracking lactate, as well as full evaluation of the multiply injured patient is crucial in determining application of DCO versus ETC. DCO is necessary as a temporizing stage to improve mortality in critically injured patients.

     

    Renaldo and Egol revisit the evolution of trauma care where early, acute treatment evolved to ETC, and again to the development and implementation of DCO and staged fixation. Reviewing the initial post-traumatic inflammatory response, the authors review literature that supports the implementation of DCO in the unstable patient, especially the role of the 'second hit' phenomenon and the spike in inflammatory cascade that occurs between days 2-5 following injury.

     

    Figure A is a lateral x-ray of a comminuted, displaced, open distal femur fracture. Figure B is an anteroposterior radiograph of a spiral, oblique, open tibia fracture.

     

    Incorrect Answers:

    Answers 2-4: Due to the elevated lactate levels and unstable nature of the patient, DCO should be applied via placement of external fixators. Definitive fixation at this time could worsen the patient's acute condition and subsequent

    morbidity and mortality.

    Answer 5: This patient, while unstable, is not in extremis, therefore temporary skeletal stabilization should be placed with external fixators.

     

     

     

    OrthoCash 2020

     

  80. Figure A is a radiograph of a 31-year-old male cyclist who was struck by a vehicle. Physical examination revealed a 2 cm opening in the skin over the anteromedial leg. He was treated operatively with an unreamed intramedullary nail. What outcome can be expected using this operative modality when compared with reamed intramedullary nailing?

     

     

     

     

    1. Higher rates of nonunion

    2. Higher rates of malunion

    3. Higher rates of infection

    4. Lower rates of infection

    5. No difference

     

    Corrent answer: 5

     

    The best definitive surgical fixation option for an open fracture of the tibia would be either a reamed or unreamed intrameduallary nail.

     

    Treatment of open tibia fractures require immediate antibiotics and early

    irrigation and debridement. Surgical fixation options are vast, but largely based on the severity of injuries both locally or systemically. External fixation devices are mainly used for provisional fixation in polytrauma patients or definitively in proximal or distal metaphyseal fractures. Plating may be used in a staged fashion after a period of external fixation. The most common fixation method is intramedullary nailing. The use of a reamed or unreamed nail technique does not affect union rates, infection rates, or need for additional surgeries in open tibia fractures.

     

    Bhandari et al conducted a multicenter, blinded randomized trial of 1319 adults in whom a tibial shaft fracture was treated with either reamed or unreamed intramedullary nailing. When comparing outcomes in open and closed injuries at twelve months, they found a benefit for reamed intramedullary nailing in patients with closed fractures, but found no difference between approaches in patients with open fractures.

     

    Finkenmeier et al conducted a randomized controlled study of reamed vs. unreamed nails in open and closed tibia fractures (excluding Grades IIIB and IIIC). They found that the use of reamed insertion of IM nails for the treatment of closed tibia fractures lead to an earlier time to union without increased complications. The authors reported no differences in infection rate, compartment syndrome rate, or percent needing additional surgeries to obtain union. More secondary procedures were needed with unreamed nails in closed fractures only.

    Figure A shows a mid-shaft tibia fracture. Incorrect Answers:

    Answers 1, 2, 3, 4: All prospective studies show no difference in the rates of

    non-union, malunion, or infection when using a reamed or unreamed nails to treat open tibia fractures.

     

     

     

    OrthoCash 2020

     

  81. A 27-year-old male presented to the trauma bay following a motor vehicle crash and was diagnosed with a comminuted open tibia fracture. He was subsequently treated with an irrigation and debridement, and un-reamed intramedullary nail. At 4 months followup, despite some signs of healing, the fracture is not fully united. Which of the following is true?

    1. Patient should be scheduled for exchange nailing.

    2. Use of an un-reamed nail increased this patients risk of infection.

    3. Use of an un-reamed nail increased this patient's risk of non-union.

    4. Patient should continue to be observed without intervention.

    5. Use of an un-reamed nail decreased this patient's risk of infection.

     

    Corrent answer: 4

     

    Tibia fractures, open or closed, when treated with an intramedullary nail can take 6 months or longer to achieve clinical and radiographic healing, and should be observed for at least 6 months before secondary intervention is considered.

     

    Open tibia fractures should be managed with debridement and irrigation initially. The choice of definitive fixation between reamed and unreamed nailing remains controversial. Recent randomized controlled studies have examined the outcomes of reamed and unreamed nailing for both closed and open tibia shaft fractures.

     

    Bhandari et al. present a prospective randomized study of patients with tibia fractures randomized to reamed or un-reamed tibial nails. Surgeons participating in the study were mandated to delay intervention for delayed union/nonunion until 6 months after the initial procedure. The authors found that many tibia fractures in both the reamed and unreamed nailing groups progressed to union without secondary intervention with this 6 month delay.

     

    Finkemeier et al. present a prospective randomized trial of tibia fractures treated with reamed or un-reamed intramedullary nails. For closed fractures they found a higher rate of union at 4 months in the reamed group, but no difference at 6 or 12 months. There was no difference in union rates for open fractures at any time point. They found no differences in other variables such as infection or compartment syndrome.

     

    Incorrect answers:

    Answer 1: This fracture has signs of healing at 4 months, and should continue to be observed until at least 6 months before a secondary operation is considered.

    Answers 2 and 5: There is no difference in infection risk between reamed and un-reamed nails for open tibia fractures

    Answer 3: There is no difference in rate of eventual union between reamed and un-reamed nails for open tibia fractures.

     

     

     

    OrthoCash 2020

  82. A 27-year-old male sustains a type I open both bone forearm fracture as seen in Figure A. During irrigation and debridement a 1 cm of cortex is removed leaving a segmental gap. Which of the following adjuvants is recommended to supplement your internal fixation?

     

     

     

     

    1. Bone grafting

    2. Tricalcium phosphate

    3. Calcium phosphate

    4. Calcium sulphate

    5. BMP-3

     

    Corrent answer: 1

     

    Bone grafting in the primary fixation of fractures is typically limited to those with segmental defects. While the increased surface area of a comminuted fracture may aid in the healing potential, the segmental defect is a limiting factor to fracture healing. Calcium sulphate, tricalcium phosphate, BMP-3, and calcium phosphate are not indicated in this clinical picture. BMP-2,4,6, and 7 all have osteoinductive activity but BMP-3 does not demonstrate osteoinductive activity.

     

    In their retrospective review of 198 fracture, Wright et. al found BG was not indicated for comminuted open fracture, and only recommend BG when there is a segmental bone defect.

     

    Wei et. al also performed a retrospective review of 64 fractures in 49 patients and found BG was not indicated for comminuted open fracture, and only recommend BG when there is a segmental defect.

     

    Finally, Schemitsch and Richards determined that optimal outcome post forearm fracture depends on restoration of the radial bow.

     

     

     

    OrthoCash 2020

  83. A 30-year-old female falls onto her outstretched arm and sustains the injury shown in Figures A and B. After intra-articular lidocaine injection, her elbow range of motion is 30°-95° extension/flexion, 45° supination, 65° pronation. There is no wrist tenderness and the radius pull test is symmetric to the contralateral forearm. What is the most appropriate treatment?

     

     

     

     

     

     

    1. Fragment excision via the extensor carpi ulnaris / anconeus approach

    2. Internal fixation with headless compression screws via the brachialis / pronator teres approach

    3. Internal fixation with a periarticular plate via the extensor carpi ulnaris /

      anconeus approach

    4. Radial head arthroplasty via the brachialis / pronator teres approach

    5. Sling and early elbow range of motion Corrent answer: 3

    This patient has a Mason type III radial head fracture composed of two articular fragments with a mechanical block to motion and should be treated with open reduction internal fixation (ORIF) using a lateral approach to the radial head.

     

    Treatment of radial head fractures depends on fracture anatomy, elbow/forearm motion and ipsilateral forearm injury. Minimally displaced fractures with no mechanical block to motion (Type I; see Illustration C) are most common and are treated with a sling and early motion. Displaced fractures with one fragment (Type II) and block to motion are often managed with ORIF. Fragments less than ~30% of the articular surface may be excised, provided there is no concern for ipsilateral forearm instability. Displaced multi-fragmentary fractures (Type III) are usually managed with radial head arthroplasty, but may be treated with ORIF in young patients with less than three fragments.

     

    Ring et al. performed a retrospective study of 56 patients who underwent radial head ORIF. Patients with more than three articular fragments had significantly poorer patient outcome scores, loss of forearm rotation and required more secondary procedures to ultimately resect the radial head. The authors concluded that ORIF is effective for fractures with less than three articular fragments and that fractures with three or more fragments are better managed with prosthetic replacement.

     

    Ruchelsman et al. reviewed radial head and neck fractures. Choice of treatment is impacted by fragment number, size (percentage of articular disc), comminution and stability, as well as radiocapitellar alignment and block to motion. When feasible, ORIF allows restoration of anatomic congruity and initiation of early motion. Radial head arthroplasty is preferred for radial head fractures with multiple fragments or if stable fixation cannot be ensured.

     

    Figures A and B are the AP and lateral radiographs of the left elbow showing a displaced complete articular radial head fracture with two articular fragments [Ruchelsman et al.]. Illustrations A and B are fluoroscopic images showing subsequent fixation of the fracture with a periarticular plate [Ruchelsman et al.]. Illustration C is the Mason classification of radial head fractures.

    Illustration D is an intraoperative photograph showing exposure of the radial head fracture via the Kocher approach (extensor carpi ulnaris / anconeus

    interval).

     

    Incorrect Answers:

    Answer 1: While the patient has no evidence of ipsilateral distal radioulnar joint or interosseous membrane injury (no wrist tenderness, normal radius pull test), fragment excision is reserved for partial articular fractures involving less than 30% of the articular surface. Complete radial head resection can lead to long-term elbow dysfunction and should be reserved for elderly, low-demand patients.

    Answer 2: Using the medial approach to the elbow for an isolated radial head procedure is not appropriate. ORIF can be performed with either headless compression screws or periarticular plates. Use of headless compression screws has been associated with improved postoperative ROM compared to plates.

    Answer 4: Using the medial approach to the elbow for an isolated radial head procedure is not appropriate. Radial head arthroplasty should be reserved for older patients and fractures with three or more articular fragments.

    Answer 5: Nonoperative treatment is not appropriate for this patient with a displaced fracture and mechanical block to motion.

     

     

     

     

     

     

     

     

     

     

     

     

     

    OrthoCash 2020

     

  84. What nerve is most at risk when applying the external fixator shown in Figure A using a minimally invasive fluoroscopic technique of pin insertion?

     

     

     

    1. Ilioinguinal nerve

    2. Obturator nerve

    3. First branch of the femoral nerve

    4. Lateral femoral cutaneous nerve

    5. Superior gluteal nerve Corrent answer: 4

    Pelvic external fixation with supraacetabular pins through the AIIS can be utilized to stabilize a pelvic fracture. While using this technique, care must be taken not to injure the lateral femoral cutaneous nerve (LFCN). Gardner et al describe the technique for placement of supraacetabular external fixation pins and state that pins in this location are more stable biomechanically compared to other locations in the iliac crest. Grothaus et al performed a cadaveric study to determine the anatomic detail and variation of the LFCN and the distances it traveled from various landmarks.The found the nerve to potentially be at risk as far as 7.3 cm medial to the anterior superior iliac spine along the inguinal ligament and as much as 11.3 cm distal on the sartorius muscle from the anterior superior iliac spine. Riina et al performed a cadaveric study to define the neurovascular structures at risk with the placement of anterior-posterior locking screws in the proximal femur. They found that risks to the neurovascular structures during anterior-posterior locking in the proximal femur are diminished if locking is performed above the level of the lesser trochanter.

     

     

     

    OrthoCash 2020

     

  85. A patient sustains a comminuted calcaneus fracture. Three months after the injury the patient complains of shoewear problems secondary to clawing of the lesser toes. What is the most likely explanation for this deformity?

    1. Sural nerve injury

    2. Tethering of the flexor hallucis longus by fracture fragments

    3. Medial plantar nerve neuropathy

    4. Weakness of the tibialis posterior

    5. Unrecognized foot compartment syndrome Corrent answer: 5

    Contracture of the intrinsic flexor muscles of the foot can be the result of unrecognized foot compartment syndrome. Foot compartment syndrome is a known complication of calcaneus fractures.

    Myerson reported 3/43 patients in his series below had chronic foot compartment syndrome. There are 9 compartments in the foot: (1) medial, (2) superficial, (3) lateral, (4) adductor, (5-8) four interossei, and (9) calcaneal.

    The plantar fascia limits the space available for hematoma and swelling, causing damage to the intrinsic flexors of the foot (particularly the lumbricals and interossei), resulting in clawtoes.

     

     

     

    OrthoCash 2020

     

  86. A 25-year-old male sustains a closed elbow dislocation after falling during a soccer game. Two months later, the patient continues to complain of pain and instability. Radiographs and physical exam are concerning for posteromedial instability. Which of the following fracture patterns is most consistent with this diagnosis?

    1. Coronoid tip

    2. Coronoid anteromedial facet

    3. Radial head

    4. Olecranon

    5. Capitellar impaction injury Corrent answer: 2

    Clinical episodes of posteromedial rotatory instability are most commonly associated with fractures of the anteromedial facet of the coronoid.

     

    Coronoid injuries are pathognomonic of an elbow instability event. In episodes of severe varus stress, the shear forces exerted on the coronoid by the distal humerus lead to injury. The anteromedial facet, which is important for varus stability, is subsequently fractured. When combined with lateral collateral ligament complex injury, posteromedial rotatory instability results.

     

    O’Driscoll et al. review treatment of coronoid fractures as part of a review on management of difficult elbow injuries. They indicate that anteromedial coronoid injuries are representative of varus posteromedial rotatory instability injuries. In the setting of coronoid injury, the radial head assumes a critical role in elbow stability.

     

    Tashjian et al. review the etiology, diagnosis, and management of complex elbow instability. In their discussion of coronoid fractures, they indicate that identification of anteromedial facet injuries is often difficult and that treatment should be instituted promptly to limit persistent instability and subsequent arthritis.

    Illustration A shows an AP radiograph of the affected elbow. An event of posteromedial instability is evidenced by the fracture through the anteromedial facet of the coronoid. Illustrations B and C show the AP and lateral radiographs of a patient who developed significant arthrosis after inadequate treatment of varus posteromedial rotatory instability.

     

    Incorrect answers

    Answers 1, 3, 5: These injuries may be seen as the sequlae of posterolateral rotatory instability.

    Answer 4: Large coronoid fractures are seen in olecranon fracture-dislocations around the elbow.

     

     

     

     

     

     

     

     

    OrthoCash 2020

     

  87. A 7-year old boy presents to the emergency room following a ATV accident with complaints of left pelvic pain. In the emergency room he is alert and oriented and is hemodynamically stable. On physical exam he is unable to bear weight on his left lower extremity. There is no tenderness to palpation at the posterior pelvis. A radiograph is performed and shown in Figure A and CT examination shows the posterior ring is stable and age-appropriate. What is the most appropriate treatment for this injury pattern?

     

     

     

     

    1. Nonoperative management with weight bearing as tolerated

    2. Percutaneous sacroiliac screw

    3. Pelvic external fixation

    4. Anterior pelvic ring plating

    5. Anterior and posterior pelvic ring plating Corrent answer: 1

    The clinical presentation is consistent for a mildly displaced parasymphyseal fracture in a pediatric patient with an open triradiate cartilage. Weight bearing as tolerated is the most appropriate treatment.

     

    In skeletally immature pelvic ring fractures, the majority of cases can be treated nonoperatively. Open reduction and internal fixation is required for acetabular fractures with >2 mm of fracture displacement and for any intraarticular or triradiate cartilage fracture displacement >2 mm. External fixation is necessary for pelvic ring displacement of >2 cm to prevent limb-length discrepancies.

     

    Holden et al. emphasize that children with open triradiate cartilage have

    different fracture patterns than do children whose triradiate cartilage has closed. They report because of the immaturity of the pelvis, the iliac wing is weaker than the elastic pelvic ligaments, resulting in bone failure before pelvic ring disruption has a chance to occur. For this reason fractures usually involve the pubic rami and iliac wings and rarely require surgical treatment.

     

    Spiguel et al. reviewed 2850 pediatric trauma admissions at their institution and reviewed cases with a pelvic ring fracture. They found that although pelvic fractures are an uncommon injury in pediatric trauma patients, the morbidity associated with these injuries is significant. They report while the majority of pelvic fractures in children are treated nonoperatively, more than one-half of these patients have concomitant injuries requiring operative management.

     

    Figure A shows an inferior rami fracture in a pediatric patient with an open triradiate cartilage.

     

    Incorrect Answers:

    Answers 1,3,4,5: These treatment options are not appropriate in a stable pelvic ring fracture in a child with open triradiate cartilage.

     

     

     

    OrthoCash 2020

     

  88. A 75-year-old-male presents after being struck by a vehicle while crossing the street. He complains of right leg pain, and physical exam reveals no evidence of an open fracture. Initial radiographs are shown in Figures A and B, and intramedullary nailing of the fracture is planned. What is the proper blocking screw technique to prevent apex anterior and valgus deformity of the fracture?

     

     

     

    1. Insertion of blocking screws lateral and posterior to the nail

    2. Insertion of blocking screws medial and posterior to the nail

    3. Insertion of blocking screws lateral and anterior to the nail

    4. Insertion of blocking screws medial and anterior to the nail

    5. Insertion of blocking screws medial, lateral, and posterior to the nail Corrent answer: 1

    Apex anterior and valgus deformity of the proximal tibia fracture can be prevented by inserting blocking screws lateral and posterior to the nail in the proximal segment.

     

    Blocking screws may be used to prevent deformity when performing intramedullary fixation of proximal tibia fractures. Insertion of the blocking screw lateral to the nail prevents valgus deformity, and insertion of the blocking screw posterior to the nail prevents apex anterior deformity. A more lateral starting point for nail insertion can also help to prevent valgus deformity. Stability screws can be placed prior to insertion of the nail to prevent deformity during nail passage, or after nail insertion to prevent postoperative deformity from developing.

     

    Ricci et al. describe the technique and results of using blocking screws and intramedullary nails to treat patients with fractures of the proximal third of the tibial shaft. Post-operatively, all patients in their series had less than 5 degrees

    of angular deformity in the planes in which blocking screws were used to control alignment. At 6 months follow-up, 10/11 patients maintained this alignment.

     

    Figures A and B demonstrate a segmental tibial shaft fracture. Illustration A shows an intraoperative fluoroscopic image demonstrating insertion of the blocking screws intraoperatively. Illustrations B and C show post-operative radiographs demonstrating placement of the blocking screws and adequate alignment of the proximal segment.

     

     

     

     

     

     

     

     

     

     

    OrthoCash 2020

     

  89. A 36-year-old man sustains blunt chest trauma, an open right femur fracture, and a closed left tibia fracture following a high-speed MVC. Upon presentation to the emergency room, blood pressure is 80/40, HR 135, and urine output is .4 cc/kg/hr. Fluids and blood products are administered, and the patient is transferred to the ICU for further care. As an alternative to lactate and base deficit measurements, which of the following would best indicate adequate resuscitation has been achieved?

    1. Systolic blood pressure > 120

    2. Heart rate between 60-100

    3. Urine output equals 0.4 cc/kg/hr

    4. Gastric intramucosal pH of 7.4

    5. Potassium between 3.5-4.5

     

    Corrent answer: 4

     

    Of the following variables, only a normal gastric mucosal pH (>7.3) is associated with restoration of tissue oxygenation.

     

    Shock is an abnormality of the circulatory system that results in inadequate organ perfusion and tissue oxygenation. This leads to anaerobic metabolism

    with the development of lactic acidosis and oxygen debt. Shock is either classified as compensated or uncompensated. Compensated shock exists when there is evidence of ongoing inadequate tissue perfusion despite the normalization of blood pressure, heart rate, and urine output. Uncompensated shock occurs when there is inadequate tissue perfusion and abnormal blood pressure, heart rate, and urine output. Thus, a patient may have normal vital signs but still be in a state of compensated shock that requires additional resuscitation.

     

    Porter et al. review the optimal end points of resuscitation in trauma patients. They conclude that using traditional end points such as blood pressure, urine output and heart rate, may leave up to 85% of patients in "compensated" shock. They urge the use of lactate, base deficit, and gastric intramucosal pH as appropriate end points of resuscitation.

     

    Roberts et al. discuss various aspects of damage control orthopaedics in the multiply injured trauma patient. Although they do not discuss the end points for resuscitation, they note that they presence of shock is a clinical parameter associated with adverse outcomes in the trauma patient.

     

    Incorrect answers:

    Answers 1-3: A patient may have normalized blood pressure, heart rate, and urine output but may still be in a state of compensated shock requiring further resuscitation.

    Answer 5: Potassium levels do not assess adequate resuscitation

     

     

     

    OrthoCash 2020

     

  90. Based on the following radiographs of tibial plateau fractures, which one is most likely to have a concomitant medial meniscus tear?

     

     

     

     

     

     

     

     

     

     

     

    1. A

    2. B

    3. C

    4. D

    5. E

    Corrent answer: 4

     

    Figure D shows a medial tibia plateau fracture (Shatzker IV). Gardner et al reviewed a 103 consecutive tibial plateau fractures to evaluate for associated soft tissue injury prevelance. The authors found that patients had a medial meniscal tear when they had a medial fracture (Shatzker IV) 86% of the time. To review the Shatzker classification of tibial plateau fracture: I) lateral split II) lateral split depression III) lateral compression (no split) IV) medial fracture V) bicondylar VI) metaphyseal dissociation. The correct choice makes intuitive sense since one would expect concomitant soft tissue injury with the bony fracture. Lateral meniscus pathology is the most common overall soft tissue injury with tibial plateau fractures (91%).

     

     

     

    OrthoCash 2020

     

  91. A 32-year-old man sustains the knee injury seen in Figure A after falling from a ladder. Which of the following options is the most biomechanically stable and appropriate definitive surgical treatment?

     

     

     

     

    1. Spanning knee external fixation

    2. Lateral plateau locking plate

    3. Posteromedial locking plate

    4. Lateral plateau percutaneous lag screws and posteromedial plate

    5. Lateral plateau and posteromedial plating Corrent answer: 5

    The above clinical scenario is consistent with a bicondylar tibial plateau fracture. Bicondylar tibial plateau fractures are best definitively treated with dual incision technique using separate lateral plateau and posteromedial plates.

     

    Yoo et al perfored a biomechanical study comparing locking and nonlocking single and dual plating constructs in maintaining posteromedial fragment reduction in a bicondylar tibial plateau fractures. They found that the posteromedial fracture fragment tolerated higher loads with a posteromedial conventional plate construct. The superiority of this dual plating construct may be caused by unreliable penetration of the posteromedial fragment by the lateral locking screws alone.

     

    Zeng et al also performed a biomechanical study comparing the biomechanical strength of four different fixation methods for the posteromedial tibial plateau split fracture. The four following constructs: anteroposterior lag-screws, an anteromedial limited contact dynamic compression plate (LC-DCP), a lateral locking plate, or a posterior T-shaped buttress plate were studied. They found that the posterior-based buttress plating technique was the most biomechanically stable fixation method allowing the least amount of fracture subsidence for posteromedial split tibial plateau fractures.

     

    Figure A shows an AP radiograph of a bicondylar tibial plateau. Illustration A shows a post-operative radiograph following the dual-plating technique for bicondylar tibial plateau fractures.

     

    Incorrect answers:

    Answer 1: External fixation is not the most appropriate definitive care for bicondylar tibial plateau fractures.

    Answers 2-4: Not as biomechanically stable as dual plating technique for bicondylar tibial plateau fractures.

     

     

     

     

     

    OrthoCash 2020

     

  92. A 35-year-old male sustains a closed tibial shaft fracture after falling from 12 feet. Which of the following measurements would be concerning for an evolving compartment syndrome?

    1. Intraoperative anterior compartment measurement of 29, with preoperative diastolic pressure 58

    2. Preoperative anterior compartment measurement of 25, with preoperative diastolic pressure of 60

    3. Intraoperative anterior compartment measurement of 25, with intraoperative diastolic pressure of 54

    4. Intraoperative anterior compartment measurement of 28, with intraoperative diastolic pressure of 72

    5. Preoperative anterior compartment measurement of 22, with mean arterial pressure of 70

    Corrent answer: 1

     

    A delta P (diastolic blood pressure minus compartment pressure measurement)of < 30 mmHg is an indication for fasciotomies with the caveat

    that the diastolic pressure is measured either pre- or postoperatively.

     

    Given the poor outcomes associated with missed compartment syndromes, it is important to obtain both clinical and objective data when determining if a patient needs fasciotomies. Determining if a patient needs fasciotomies in the operating room while a patient is under anesthesia is complicated by the fact that obtaining a clinical exam is impossible, and that the diastolic blood pressure may be falsely decreased compared to normal pre- or postoperative measurements. Currently, it is recommended that intraoperative compartment pressures be compared to preoperative diastolic blood pressures, with delta p

    < 30 indicating the need for fasciotomies.

     

    Kakar et al. review the preoperative, intraoperative, and postoperative diastolic blood pressure (DBP) in 242 patients with a tibia fracture treated operatively.

    They found the mean DBP was 18 points lower in the operating room compared to the preoperative measurement. In addition, they found the difference between preoperative and postoperative diastolic blood pressures to be within 2 points, indicating the decrease seen intraoperatively is likely a spurious value induced by anesthetic.

     

    McQueen and Court-Brown prospectively review 116 patients with tibia fractures that had continuous monitoring of their anterior compartment for 24 hours. They found that using an absolute pressure of 30 mmHg would have resulted in 50 patients (43%) treated with unnecessary fasciotomies. They conclude using a differential pressure of 30 mmHg is a more reliable indicator of compartment syndrome.

     

    Incorrect Answers:

    Answer 2: a delta p of 35 is not an indication for fasciotomies in the setting of a benign clinical exam.

    Answer 3: a delta p of 29 when calculated with the intraoperative diastolic blood pressure is not an indication for fasciotomies. This value should be obtained from preoperative diastolic blood pressure measurements.

    Answer 4: a delta p of 44 is not an indication for fasciotomies.

    Answer 5: mean arterial pressure is not used for calculation of delta p.

     

     

     

    OrthoCash 2020

     

  93. A 44-year-old male is involved in a motorcycle collision and presents with the radiographs shown in Figure A. A CT scan is obtained which shows intra-articular extension of the fracture, and lateral locked plating with intercondylar lag screw fixation is planned. Which of the following is important intra-operatively to ensure that the

    intercondylar screws are contained within the bone and are of appropriate length?

     

     

     

     

    1. AP fluoroscopic imaging with the leg in 30 degrees of internal rotation

    2. AP fluoroscopic imaging with the leg in 30 degrees of external rotation

    3. AP fluoroscopic imaging with the knee in full extension

    4. Lateral fluoroscopic imaging with the knee in 30 degrees of internal rotation

    5. Lateral fluoroscopic imaging with the knee in 15 degrees of flexion Corrent answer: 1

    Figure A illustrates a comminuted distal femur fracture. AP fluoroscopic imaging with the leg in 30 degrees of internal rotation is important to prevent intercondylar screw prominence.

     

    Hardware irritation is a common post-operative complication of distal femoral plate fixation. Two common sites of pain are laterally where the iliotibial band is in contact with the plate, and medially where intercondylar screws may penetrate the cortex if they are of inappropriate length. Iliotibial irritation most commonly presents with activities requiring knee flexion and extension. It is important to remember that the lateral metaphysis of the distal femur is angulated 10 degrees from the sagittal plane, and the medial metaphysis is angulated 25 degrees from the sagittal plane. Therefore, if a straight AP view is obtained, a distal screw can appear to be inside the bone even if it is too long. In order to assess the exact length of the screw, one must obtain an AP view with 30° internal rotation of the lower extremity.

     

    Gwathmey et al discuss distal femoral fractures in their review article. They state that the goal of surgical management is to promote early knee motion while restoring the articular surface, maintaining limb length and alignment,

    and preserving the soft-tissue envelope with a durable fixation that allows functional recovery during bone healing. They describe a variety of surgical exposures, techniques, and implants developed to treat these injuries, including intramedullary nailing, screw fixation, and periarticular locked plating, possibly augmented with bone fillers.

     

    Illustration A demonstrates the sagittal plane angulation of the medial and lateral cortex of the distal femur. Illustration B shows a knee in external rotation, with the intercondylar screw appearing to be of appropriate length. Illustration C shows the knee in internal rotation, which indicates that the screw is penetrating the medial cortex.

     

     

     

     

     

     

     

     

     

     

    OrthoCash 2020

     

  94. A 68-year-old male falls onto his outstretched hand and suffers the injury shown in Figures A and B. He undergoes operative treatment of his fracture, and immediate post-op radiographs are shown in Figure C. Two weeks later he presents with significantly increased pain and deformity. He denies any new trauma, and has followed all postoperative activity restrictions. Current radiographs are shown in Figure D and a clinical photograph of the affected wrist is shown in Figure E. Which of the following is the most likely cause for failure of fixation in this patient?

     

     

     

     

     

     

     

     

     

     

     

     

     

     

    1. Failure to support the lunate facet with fragment specific fixation

    2. Use of a non-locking plate

    3. Lack of volar tilt restoration

    4. Lack of radial styloid column plating

    5. Use of only three bicortical screws in the intact radial shaft proximally

    Corrent answer: 1

     

    The failure of this patient's fixation post-operatively is caused by failure to support the lunate-facet fragment noted on the injury radiographs.

     

    The stability of comminuted fractures of the distal part of the radius with volar fragmentation is determined not only by the reduction of the major fragments but also by the reduction of the small volar lunate fragment. The distal volar lunate fragment is the site of origin of the strong volar radiolunate ligaments which insert onto the lunate, and so displacement of this small piece volarly will allow the lunate and the rest of the carpus to subluxate volarly. The unique anatomy of this fragment may prevent standard fixation devices for distal radial fractures from supporting the entire volar surface effectively, as a standard volar plate cannot capture this small distal piece without risking injury to the flexor tendons. Fragment specific fixation of the volar lunate facet fragment with commercially available small plates, or with a tension-band construct or augmentation with K-wires may be required to reduce and stabilize this fragment.

     

    Harness et al. reported on a cohort of 7 patients with a volar shearing fracture of the distal radius who lost fixation of a volar lunate facet fragment with subsequent carpal displacement after open reduction and internal fixation. Five patients underwent revision surgery with adequate results. The authors concluded that with regards to lunate facet fracture fragments, it is preferable to recognize the complexity of the injury prior to the initial surgical intervention and to plan accordingly to prevent early post-operative failure.

     

    Taylor et al. compared the biomechanical stability of a fixed-angle volar plate versus a fragment specific fixation system in the treatment of an intraarticular, dorsally comminuted distal radius fracture model. They found that both fixed-angle volar plate and fragment-specific fixation systems performed comparably in a simulated early postoperative motion protocol. Fragment-specific fixation had improved stiffness characteristics only with respect to the smaller ulnar-sided fragment.

     

    Figures A and B show a shearing radiocarpal-fracture subluxation with small lunate-facet fracture. Figure C shows an immediate post-operative radiograph. Figure D shows subluxation of the radiocarpal joint caused by failure to support lunate-facet fragment. Figure E shows the clinical appearance of a volarly subluxated wrist.

     

    Incorrect Answers:

    Answer 2: The use of a non-locking plate in this situation did not directly lead to the failure of fixation, and applying a similar construct with locking fixation

    would not have prevented volar subluxation of the lunate facet fragment. Answer 3: Volar tilt was grossly restored post-operatively.

    Answer 4: Radial styloid plating would not have prevented volar subluxation of the lunate facet fragment.

    Answer 5: Three bicortical screws in the intact radial shaft proximally is adequate fixation.

     

     

     

    OrthoCash 2020

     

  95. Which of the following is most commonly associated with both simple and complex elbow dislocations?

    1. radial head fracture

    2. radial neck fracture

    3. loss of terminal extension

    4. repeat dislocation

    5. coronoid base fracture Corrent answer: 3

    Elbow dislocations are classified as either simple (no associated fracture) or complex (associated fracture). The goal of treatment is a stable joint that tolerates early motion. The initial range of motion is the stable arc found on postreduction examination. Studies have demonstrated a better outcome when simple elbow dislocations are treated non-surgically rather than with surgical repair. Simple elbow dislocations usually have an excellent outcome (return of functional range of motion with normal strength). A loss of terminal extension is the most common sequelae.

     

    Ross et al. reported on 20 patients with simple closed posterior elbow dislocations who were treated with immediate active ROM under close supervision. They found a final arc of motion of -4 to 139 degrees, with final motion reached a mean of 19 days after dislocation. Only 1 patient had recurrent instability.

     

     

     

    OrthoCash 2020

     

  96. A 36-year-old female sustains a knee injury after falling from a ladder onto her flexed knee; she cannot do a straight leg raise after a lidocaine injection into her knee. A radiograph is shown in Figure A. Which of the following treatment options has been shown to have the best outcomes with this injury?

     

     

     

    1. Long leg cast

    2. Hinged knee brace use with functional rehabilitation protocol

    3. Open treatment with internal fixation or excision with patellar tendon advancement

    4. Distal patellar resection and allograft reconstruction

    5. Placement of a cerclage wire from patella to proximal tibia Corrent answer: 3

    The clinical presentation is consistent with and inferior pole patella fracture. Open reduction and internal fixation or excision with patellar tendon advancement is the most appropriate treatment of this injury pattern.

     

    Whenever possible, salvage of the inferior pole through open reduction internal fixation is favored over simple excision and patellar tendon advancement, as this has been shown to be associated with improved outcomes. This is not always possible, however, and pole resection can be performed if the inferior comminution precludes fixation. Inferior outcomes of the partial distal patellectomy and patellar tendon advancement are (aside from the possible resulting patella baja) probably not directly related to the patellofemoral articulation. On the undersurface, the proximal 75% of the patella is covered with articular cartilage; however, the distal 25% is not, and does not articulate with the femoral trochlea.

     

    Kastelec et al. performed a retrospective review of ORIF v. pole resection followed over 4.6 years postoperatively. There was a significant increase (better) in patellofemoral score with ORIF; significant differences in knee pain,

    tolerated activity levels, and ROM were also noted. Patella baja was frequently noted with pole resection and correlated with worse functional outcomes.

     

    Matejcic et al. performed a retrospective review of ORIF v. pole resection followed over 5.3 years postoperatively. Results were excellent/good in 90.1% of the ORIF and only 73.1% of the pole resection patients. In addition, significant differences between the groups were noted with regard to knee pain, swelling, level activity, compression pain, range of motion, muscular atrophy, muscular strength, and final patellofemoral score (all better with ORIF).

     

    Figure A is a lateral knee radiograph showing an inferior pole patella fracture.

     

     

     

    OrthoCash 2020

     

  97. Which of the following factors increase the risk of nonunion in midshaft clavicle fractures when treated nonoperatively?

    1. Sling immobilization

    2. Displacement and comminution

    3. Age less than 40 years old

    4. Immediate motion exercises

    5. Male

     

    Corrent answer: 2

     

    Robinson et al have shown that lack of cortical apposition, comminution, female gender, and advancing age are the 4 factors that contribute to nonunion.

     

    The Canadian Orthopaedic Trauma Society in a randomized, prospective study showed that for midshaft fracture in adults with 100% displacement, ORIF results in improved DASH and Constant scores (p = 0.001 and p < 0.01, respectively), lower nonunion (2 vs. 7, p=0.042) & lower malunion (0 vs. 9, p=0.001). Surgery resulted in quicker radiographic union (16.4 weeks vs. 28.4 weeks, p=0.001). However, 15% had hardware and wound complications. At one year, the operative group was more likely to be satisfied with the shoulder in general (p=0.002) and the appearance of the shoulder in particular (p=0.001) in comparison to the nonoperative group.

     

    Prior studies have shown that greater than 2cm of shortening treated nonoperatively results in increased fatigueability and poor outcome, but not necessarily nonunion. The Lazarides article concluded that “Final clavicular shortening of more than 18 mm in male patients and of more than 14 mm in

    female patients was significantly associated with an unsatisfactory result.”

     

    Studies have shown no difference in outcome when treated with a Figure-of-8 harness compared to a simple sling.

     

     

     

    OrthoCash 2020

     

  98. A 34-year-old male presents after falling off a roof at his job. He has an obvious deformity of his left lower extremity, and injury radiographs are shown in Figures A and B. He has no other injuries. Which of the following definitive treatment algorithms will most likely lead to the best outcomes in this patient?

     

     

     

     

     

     

    1. Closed reduction and percutaneous screw fixation of the femoral neck, followed by reamed antegrade nailing of the femur fracture

    2. Reamed antegrade nailing of the femoral shaft fracture, followed by open reduction and percutaneous screw fixation of the femoral neck fracture

    3. Reamed retrograde nailing of the femoral shaft fracture, followed by closed

      reduction and percutaneous screw fixation of the femoral neck

    4. Open reduction and screw fixation of the femoral neck, followed by reamed retrograde nailing of the femoral shaft fracture

    5. Open reduction and screw fixation of the femoral neck, followed by plating of the femoral shaft fracture

    Corrent answer: 4

     

    The clinical presentation is consistent with a femoral shaft fracture with an ipsilateral femoral neck fracture. Of the options presented, the most appropriate treatment is open reduction and lag screw fixation of the femoral neck fracture followed by reamed retrograde nailing of the femoral shaft fracture.

     

    Ipsilateral femoral neck/shaft fractures are an uncommon injury estimated to occur in 2-6% of all femoral shaft fractures. It is generally agreed upon that due to the potentially devastating complications of the femoral neck fracture in young patients, the neck fracture should be treated first and the shaft fracture second. Anatomic reduction of the femoral neck fracture is paramount in obtaining successful healing, and therefore open reduction is recommended in the setting of fracture displacement. Provisional reduction of the femoral neck fracture, followed by antegrade nailing with subsequent addition of definitive fixation of the femoral neck is also acceptable.

     

    Peljovich et al. present a review article on ipsilateral femoral neck and shaft fractures. The treatment algorithm they propose consists of first treating the femoral neck fracture, and then addressing the femoral shaft fracture with retrograde nailing. They also highlight the risk and benefits of each treatment approach.

     

    Watson et al. reviewed 13 patients who had healing complications after undergoing surgical fixation of their ipsilateral femoral neck and shaft fractures. They found that lag screw fixation of the femoral neck fracture and reamed intramedullary nailing for shaft fracture stabilization were associated with the fewest complications. Therefore, this approach was recommended as the treatment of choice.

     

    Figures A and B demonstrate an ipsilateral comminuted femoral shaft fracture, and a displaced femoral neck fracture. Illustrations A-C show an example of a femoral shaft fracture with an ipsilateral femoral neck fracture treated with lag screws of the femoral neck fx and retrograde nailing.

     

    Incorrect Answers:

    Answer 1: Open reduction of the femoral neck fracture with anatomic

    alignment is preferred to closed reduction in this case.

    Answer 2,3: The femoral neck fracture should be addressed first.

    Answer 5: Nailing of the femoral shaft fracture would be preferred initially in this case to prevent disruption of the biology and fracture healing around the comminuted segments.

     

     

     

     

     

     

     

     

     

     

    OrthoCash 2020

     

  99. During surgical treatment of the most common variation of distal femoral "Hoffa" fractures, which of the following orientations for screw fixation should be used?

    1. Medial to lateral screw placement across lateral femoral condyle

    2. Anterior to posterior screw placement across medial femoral condyle

    3. Medial to lateral screw placement across medial femoral condle

    4. Anterior to posterior screw placement across lateral femoral condyle

    5. Anterior to posterior screw placement across intercondylar notch Corrent answer: 4

    The most common variation of a Hoffa fracture is a coronal fracture of the lateral femoral condyle. The most appropriate screw placement of the above answer choices in the treatment of the most common Hoffa fracture variant would be anterior to posterior screws across the lateral condyle for fixation.

     

    Hoffa fractures are coronally oriented fractures of the femoral condyles, with most occurring in the lateral condyle. They are commonly associated with high-energy fractures of the distal femur and can often be overlooked during the assessment and treatment of distal femur fractures. Hoffa fractures are best evaluated using CT scans.

    Nork et al. studied the association of supracondylar-intercondylar distal femoral fractures and coronal plane fractures. Of 202 supracondylar-intercondylar distal femoral fractures, they found coronal plane fractures were diagnosed in 38%. A coronal fracture of the lateral femoral condyle was involved more frequently than the medial condyle. Eighty-five percent of these coronal fractures involved a single lateral femoral condyle.

     

    Holmes et al. looked at five cases of coronal fractures of the femoral condyle. All cases received open reduction and internal fixation with lag screws through a formal parapatellar approach. They reported good results with all fractures healing within 12 weeks without complications with final range of motion at least 0 degrees to 115 degrees.

     

    Illustration A shows sagittal and axial CT scan cuts showing a Hoffa fracture of the lateral femoral condyle. Illustration B shows multiple anterior to posterior, and posterior to anterior oriented screws for ORIF of the Hoffa fracture.

     

     

     

     

     

     

     

     

    OrthoCash 2020

     

  100. A 26-year-old right hand dominant male is involved in a motor vehicle collision and sustains the left humerus injury demonstrated in Figure A. The brachial artery is disrupted and requires urgent attention in the operating room. The patient's preoperative nerve evaluation demonstrates that the patient is unable to initiate extensor carpi radialis longus, extensor carpi radialis brevis, extensor pollicis brevis, extensor digitorum, extensor indicis proprius, and extensor pollicis longus motor activity. What is the most likely etiology for this observed neurologic examination?

     

     

     

    1. Neurapraxia of the median nerve

    2. Axonotmesis of the radial nerve

    3. Neurotmesis of the ulnar nerve

    4. Neurotmesis of the radial nerve

    5. Axonotmesis of the ulnar nerve Corrent answer: 4

    The most likely cause of the radial nerve palsy in a high energy open humerus fracture is laceration or complete disruption of the radial nerve (Neurotmesis).

     

    Neurotmesis is complete disruption of nerve and is associated witn no spontaneous recovery without intervention. Axonotmesis constitutes axon disruption, but the surrounding neural connective tissue is intact and nerve regeneration can occur(Wallerian or antegrade degeneration). Neurapraxias occur often by compression and the axon maintains continuity but local demyelination and ischemia occur.

     

    Ring et al. present a Level 4 study of 24 patients that had radial nerve palsy associated with a humerus fracture. All 6 patients with a transected radial nerve had an open humerus fracture also. The results of primary nerve repair in this circumstance found that there was no recovery in any of the patients.

     

    Foster et al. authored a Level 4 review of 14 patients had a radial nerve palsy and an associated open humerus fracture. 64% of the 14 patients had a radial nerve that was either lacerated or interposed between the fracture fragments. They recommend exploration of the radial nerve in the setting of a radial nerve palsy and concomitant open humerus fracture in contrast to observation of a radial nerve palsy in closed humerus fractures.

    Figure A demonstrates an open left humerus fracture.

     

    Incorrect Answers:

    Answer 1,3,5: The radial nerve provides distal motor activity to the ECRL, ECRB, EPB, EIP, and EPL.

    Answer 2: Axonotmesis of the radial nerve is not the most common form of injury associated with closed or open humerus fractures.

     

     

     

    OrthoCash 2020

     

  101. Which of the following is true regarding the center of rotation of angulation (CORA) as it refers to tibial diaphyseal angular deformity?

    1. It is the point at which the proximal mechanical axis and distal mechanical axis meet

    2. It is the point at which the proximal anatomical axis and proximal mechanical axis meet

    3. It is always the point on the cortex at the most concave portion of the deformity

    4. It is the point at which the distal anatomical axis and distal mechanical axis meet

    5. It is always the point on the cortex at the most convex portion of the deformity

    Corrent answer: 1

     

    The center of rotation of angulation(CORA) in diaphyseal tibial deformity is defined as the intersection of the proximal mechanical(PMA) or anatomical axis(PAA), and the distal mechanical(DMA) or anatomical axis(DAA).

     

    Angular deformity of the femur or tibia involves angulation not only of the bone but also of its axes. When a bone is divided and angulated, the mechanical and anatomic axis of the bone are also divided into proximal and distal segments.

    The pairs of proximal and distal axis lines intersect to form an angle. The point at which the proximal and distal axis lines intersect is called the CORA.

     

    The axis line of the proximal bone segment is called the PMA or PAA, and axis line of the distal segment is called the DMA or DAA. In the tibia, because the mechanical and anatomical axes are almost the same, the PMA and PAA lines overlap, as do the DMA and DAA lines.

     

    Illustration A shows the CORA as it relates to the axes of an angulated tibia.

    Incorrect Answers:

    2-5:These do not describe the CORA or any other specific anatomical points as they related to long bone deformity.

     

     

     

     

     

     

    OrthoCash 2020

     

  102. At long term follow-up, a male who sustains multiple traumatic injuries compared with a premenopausal female, who sustained similar polytrauma, is most likely to have which of the following?

    1. Higher quality-of-life scores than females

    2. Increased rates of complex regional pain syndrome

    3. Require more psychiatric counseling and pharmacologic management than females

    4. Take more absentee days at work as a result of illness than females

    5. Decreased incidence of lower extremity amputation Corrent answer: 1

    Ten or more years after severe polytrauma, premenopausal women, compared to men, demonstrate a higher incidence of posttraumatic stress disorder (PTSD) and take more sick leave time from work.

    Depression following polytrauma should be screened for by treating orthopaedic surgeons. Depression commonly arises from a protracted injury and can compound the disability by increasing the perception of more physical illness.

     

    Probst et al. present a Level 4 review of over 600 polytrauma patients. They found that quality-of-life was significantly lower in women (Short form-12 psychologic F = 48.6 +/- 10.8 vs. M = 50.8 +/- 9.4; p = 0.02), but the same rate of women (75.3%) and men (75.4%; p = 0.995) felt well rehabilitated.

     

    Holbrook et al. report a Level 4 study of 1,048 polytrauma patients. They found that females had lower quality-of-life scores and were significantly more likely to develop early combined depression.

     

    Incorrect Answers:

    Answer 4: Females take more absentee days at work.

    Answers 2, 3, 5: There is no data supporting these answer options.

     

     

     

    OrthoCash 2020

     

  103. A 29-year-old healthy Caucasian female presents to the emergency department with her boyfriend with a left anterior shoulder dislocation and several facial abrasions after tripping in the shower approximately three hours prior to arrival. She is 5’7’’ and weights 120 lbs (BMI 18.8). The patient notes that she has not had a menstrual period in three months and that she is training for a long-distance triathlon. Physical examination reveals no other obvious injuries. Basic serum laboratory values are unremarkable. A urine pregnancy test is positive. In addition to treating her shoulder dislocation, the orthopaedic surgeon should

    1. Perform a skeletal survey to evaluate for other fractures or dislocations

    2. Question the patient in private about the specific details of this injury and her sense of safety in the home

    3. Refer the patient to a nutritionist given the patient’s BMI of 18.8

    4. Obtain additional laboratory studies and admit the patient to the hospital for treatment of anorexia nervosa

    5. Obtain advanced imaging of the shoulder to evaluate for underlying pathologic lesions

    Corrent answer: 2

    While this patient may have a perfectly reasonable explanation for her current injury, it is imperative for health care providers to inquire into the safety of the home environment in potential cases of intimate partner violence (IPV).

     

    Risk factors for female victims IPV include being in the 2nd or 3rd decade of life, current pregnancy, having 1 or more prior children, and low socioeconomic status. Reporting requirements for adult abuse are not standardized amongst most states and physicians must understand the importance of identification and documentation in cases of suspected IPV.

     

    Bhandari et al. state that musculoskeletal injuries are the second most common form of IPV (28%) after head and neck injuries (40%).

    Musculoskeletal injuries can be variable, including sprains, fractures, dislocations, and foot injuries.

     

    Della Rocca et al. conducted a survey and found orthopaedic surgeons have several misconceptions regarding victims of IPV and further, nearly half of their survey respondents reported identifying a victim of IPV while only 4% reported that they currently screen their injured female patients for IPV.

     

    Zillmer presents a review of the issues surrounding IPV including proper identification, documentation, appropriate questioning and how to involve community services.

     

    Illustration A shows some basic facts regarding IPV during pregnancy as reported by the CDC. Additional information can be found at the following link.

    http://www.cdc.gov/reproductivehealth/violence/IntimatePartnerViolence/index.htm

     

    Incorrect Answers:

    Answer 1. There is no indication for a skeletal survey and it would be inappropriate to subject this pregnant patient to additional radiation.

    Answer 3. There is no need to refer a patient to a nutritionist for a normal BMI Answer 4. There is no indication for admission or other laboratory studies in this otherwise healthy patient.

    Answer 5. There is no need for advanced imaging in the emergency department in the case of an isolated traumatic shoulder dislocation

     

     

     

     

     

    OrthoCash 2020

     

  104. A 19-year-old military recruit complains of 7 weeks of right heel pain. He notes the pain is worse with jumping and long distance running. He has completed a course of plantar fascia and Achilles tendon stretching with no significant improvement in his symptoms. He denies constitutional symptoms. On examination, his body mass index is 22, he has a normal foot posture and can perform a single leg heel rise without difficulty. There is no pain with palpation of the lateral border of the foot or with external rotation stress to the midfoot. There is tenderness with medial and lateral compression of the hindfoot and there is a negative syndesmosis squeeze test. There is a negative Tinel's sign at the tibial nerve. Axial and lateral radiographs are shown in Figures A and B. What is the most appropriate next step in management?

     

     

     

     

     

     

    1. Heel pad cortisone injection

    2. Platelet rich plasma injection and 6 weeks of physical therapy

    3. Restricted weight bearing and magnetic resonance imaging of the foot

    4. Release of the first branch of the lateral plantar nerve

    5. ASTYM or Graston physical therapy techniques to the achilles and plantar fascia

    Corrent answer: 3

     

    Restricted weight bearing and magnetic resonance imaging of the foot is indicated in this military recruit with a positive heel compression test and suspicion for calcaneal stress fracture.

     

    Calcaneal stress fractures occur as a result of repetitive loading and are more common in females with the female athlete triad (anorexia athletica) of disordered eating, amenorrhea, and osteoporosis. On exam tenderness is obtained with medial and lateral compression of the calcaneus.

    Sormaala et al. performed a Level 4 review over 8 years of the Finnish military. They found 34 calcaneal stress fractures with 19 occurring in the posterior part of the calcaneus. Only 15% of the patients had the stress fracture visualized on plain radiography. They conclude that MRI is warranted if plain radiography does not show abnormalities in a physically active patient with exercise-induced pain in the ankle or heel. A nuclear medicine bone scan is also appropriate for diagnosis but MRI may give greater detail.

     

    Gehrmann et al. present a Level 5 review of foot stress fractures. They report that with calcaneal stress fractures the fracture lines are oriented vertically or obliquely in the tuberosity of the calcaneus.

     

    Figures A and B show some sclerosis in the central calcaneus with no evidence of overt fracture and an enthesophyte at the Achilles tendon insertion.

    Illustration A and B are T2 sagittal and coronal images demonstrating a stress fracture in the superior calcaneal tuberosity with surrounding edema.

     

    Incorrect Answers:

    Answer 1: Heel pad cortisone injections are typically avoided due to risk of fat pad atrophy and are not indicated in calcaneal stress fractures.

    Answer 2: Platelet rich plasma is not indicated in calcaneal stress fractures. Answer 4: The patient presents with a negative Tinel's at the tibial nerve and presentation is not consistent with first branch of the lateral plantar nerve entrapment.

    Answer 5: ASTYM and Graston physical therapies are not indicated for calcaneal stress fractures.

     

     

     

     

     

     

     

     

    OrthoCash 2020

     

  105. A 35-year old male is involved in a fall from height and present with the isolated injury shown in Figures A and B. The body of the talus is extruded medially through a large linear open wound. Along with irrigation and debridement, what is the most appropriate definitive management of this injury?

     

     

     

     

     

     

    1. Reimplantation of the talar body followed by cast immobilization

    2. Reduction of talar body, fracture fixation with smooth Steinman pins, and spanning fixator placement

    3. Talar body allograft with internal fixation to native talar head

    4. Fragment removal, antibiotic spacer placement and external fixation

    5. Reduction of native talar body and ORIF of talar neck fracture

    Corrent answer: 5

     

    The patient is presenting with a displaced talar neck fracture with extrusion of the talar body. Reimplantation of the talar body and ORIF of the talar neck fracture is the most appropriate treatment.

     

    Talar extrusions are rare injuries and are caused by high energy mechanisms. Complete dislocation without a concomitant fracture is extremely rare. The talus is prone to dislocation as there are no muscular attachments. These injuries need to be treated with anatomic reduction and internal fixation to prevent avascular necrosis and post-traumatic arthritis.

     

    Smith et al. investigated the clinical results and functional outcome after reimplantation of the extruded talus. They concluded that salvage and reimplantation of the talus is a relatively safe procedure with only 2 of the 27 patients developing an infection. No association was found between outcome and associated talar fracture.

     

    Van Opstal et al. reported on two cases of talar extrusions following high energy injuries and reviewed the associated literature. Both cases were treated with wound I&D, reduction and external fixation. After 1 year they had pain free ROM with no signs of AVN or arthritis. Review of the literature showed that anterolateral dislocations are more common than anteromedial. Total talar dislocation is thought to be the endpoint of maximum pronation or supination injuries.

     

    Figures A and B show an extruded talar body in the setting of a Hawkins III talar neck fracture. Illustration A shows a clinical photo of an extruded talus.

     

    Incorrect Answers:

    Answer 1: Displaced talar neck fractures should not be treated in a cast, especially open fractures

    Answer 2: Definitive treatment of this injury is most effectively accomplished with formal ORIF. Steinman pin fixation is not adequate, even in the presence of an external fixator.

    Answer 3: There is no data supporting removal of fragment and use of allograft overuse of native talar body

    Answer 4: There is no data supporting removal of the fragment. The associated references support retaining the fragment.

     

     

     

     

     

    OrthoCash 2020

     

  106. An 18-year-old male sustains a right hip injury after being tackled on the football field. Figure A shows his radiograph upon presentation to the emergency room three hours later. On physical exam, he is noted to have a foot drop and decreased sensation globally throughout his entire lower leg. Closed reduction under conscious sedation is immediately performed, and the hip is able to be ranged through a stable arc of motion following reduction. A post-reduction radiograph is shown in Figure B. Shortly after the reduction, the patient continues to have a foot drop, but his sensation is slightly improved. Which of the following is the most appropriate next step in management?

     

     

     

     

     

     

    1. Exploration of his sciatic nerve

    2. EMG

    3. CT scan of his right hip

    4. Touch-down weight bearing of his right leg and observation of his sciatic nerve palsy

    5. Skeletal traction on the distal femur to relax tension on the sciatic nerve Corrent answer: 3

    Following successful reduction of a traumatic hip dislocation, a CT scan must be obtained to evaluate for any entrapped osteochondral fragments.

     

    Traumatic hip dislocations are typically the result of high-energy trauma and are frequently associated with posterior wall acetabular fractures. Initial management of a traumatic hip dislocation, with or without a fracture, is immediate closed reduction under conscious sedation. If closed reduction is unsuccessful, immediate open reduction in the operating room should be performed. Following successful reduction, a CT scan is necessary to rule-out

    intra-articular osteochondral fragments, even in the absence of fracture on radiographs. The incidence of sciatic nerve palsy following a traumatic hip dislocation is roughly 10%.

     

    Pascarella et. al. review 127 traumatic hip dislocations with and without associated fractures. 5 were anterior dislocations, 13 were central dislocations, and 109 were posterior dislocations. In 45 of the cases, an intra-articular fracture fragment was found after successful closed reduction. They stress the importance of post-reduction CT scans given the large incidence of intraarticular fragments.

     

    Bartlett et al. present a case study of a man who sustained cardiac arrest after attempted arthroscopic removal of a loose body in the hip following a traumatic hip fracture-dislocation. They believe that arthroscopic fluid extravasated through the fracture site under pump pressure and resulted in an intraabdominal compartment syndrome that presented as cardiopulmonary arrest. They do not support arthroscopic procedures of the hip following acute or healing acetabular fractures.

     

    Figure A shows an AP pelvic radiograph with a R hip dislocation. Figure B shows the post-reduction radiograph with a concentrically reduced hip joint and no evidence of fracture.

     

    Incorrect Answers:

    Answer 1: Acute exploration of the sciatic nerve is controversial, and shouldn't be entertained before a CT scan has been completed.

    Answer 2: EMG scan may be useful if the sciatic nerve fails to recover after a period of weeks to months.

    Answer 4: CT scan must be obtained, even in the absence of a fracture on radiographs. In the absence of entrapped fragments, and if a concentric reduction is obtained, the next step would be TDWB and observation of the sciatic palsy.

    Answer 5: Skeletal traction is not necessary if the hip is stable after concentric reduction and there are no associated fractures.

     

     

     

    OrthoCash 2020

     

  107. Which of the following injuries is anticipated to have a 20% chance of mortality and 50% chance of loss of independence at one year when sustained by an 85-year-old female?

    1. Intra-articular distal humerus fracture

    2. Distal radius fracture

    3. Femoral neck fracture

    4. Trimalleolar ankle fracture-dislocation

    5. Periprosthetic distal femur fracture Corrent answer: 3

    A femoral neck fracture has been shown to have an estimated mortality rate of 20% or more at one year after injury, and estimates regarding loss of independence are at 50%.

     

    Most literature on the topic points out the highest mortality rate is in the first 3-6 months, with mortality rates of 14-36% at one year; mortality rates have been found to return to normal, age-matched controls after that.

     

    Egol et al. provide a review of the factors involved in functional recovery of patients with femoral neck fractures. They report the successes of integrated care pathways and review the risk factors (institutionalization, comorbidities, etc.) that go into the outcomes of these patients. They recommend comanagement of these patients with a medical or geriatric service in order to improve patient outcomes.

     

    Incorrect Answers:

    Answer 1,2,4,5: These can be debilitating injuries, but no evidence exists to show these injuries are associated with these levels of morbidity and mortality at one year.

     

     

     

    OrthoCash 2020

     

  108. A 17-year-old male falls from a retaining wall onto his left arm. He sustains the injury shown in Figure A. The patient undergoes open reduction and internal fixation of the fracture. Upon discharge from the hospital the medication reconciliation includes an order for daily Vitamin C 500mg supplementation. This medication is given in an effort to decrease the incidence of which of the following?

     

     

     

    1. Upper extremity deep vein thrombosis (DVT)

    2. Acute carpal tunnel syndrome (ACTS)

    3. Complex regional pain syndrome (CRPS)

    4. Lower extremity deep vein thrombosis (DVT)

    5. Surgical site infection (SSI) Corrent answer: 3

    The patient has the clinical presentation of a distal radius fracture and Vitamin C administration has been associated with a lower incidence of complex regional pain syndrome (CRPS) in this group of patients.

     

    Vitamin C administration is associated with a lower risk of complex regional pain syndrome (CRPS), formerly known as reflex sympathetic dystrophy (RSD), after wrist fractures. Vitamin C is thought to reduce lipid peroxidation, scavenge free hydroxyl radicals, protect the capillary endothelium, and inhibit vascular permeability.

     

    Zollinger et al. performed a prospective randomized trial that included 317 adult patients who sustained 328 distal radius fractures and were treated conservatively. They had allocated treatment groups to 200mg, 500mg, or 1500mg vitamin C dosages for 50 days. RSD/CRPS occurrence was lowest in the 1500mg group, however the 500mg dosage for 50 days was recommended at the conclusion of the study. The prevalence of CRPS was 2.4% in the vitamin C group and 10.1% in the placebo group. They found that all of the affected patients were elderly women.

     

    Besse et al. performed a study of 392 patients that underwent foot surgery. They found that the group that received 1 gram daily of Vitamin C following foot surgery had a CRPS incidence of 1.7% compared to a control group that

     

    did not receive Vitamin C and had a CRPS incidence of 9.6%. Figure A is a clinical photo of a distal radius fracture.

     

    OrthoCash 2020

     

  109. A 34-year-old female requests a second opinion following open reduction internal fixation (ORIF) of her left ankle three weeks ago. Which of the following is most appropriate step based on Figures A and B?

     

     

     

     

    1. Progressive weightbearing in 3-4 weeks based on radiographs

    2. Deltoid ligament repair vs reconstruction

    3. Revision ORIF of fibula with lengthening

    4. Revision ORIF of fibula and syndesmosis

    5. Removal of syndesmotic screws in 3-6 months Corrent answer: 4

    The above clinical scenario shows acute postoperative fibular malrotation and tibiofibular syndesmotic malreduction. Restoration of the proper syndesmotic relationship involves regaining fibular length as well as reestablishing correct

    rotation and position of the fibula relative to the tibia. In addition, removal of interposed tissue (deltoid ligament) in the medial joint space may be necessary. However, deltoid reconstruction is not routinely required.

     

    Syndesmosis screw fixation in ankle fractures with syndesmotic disruption is indicated if there is residual or dynamic instability with stress testing of the tibiofibular joint after fixation of the fibula. Syndesmotic screws are commonly maintained in place for at least 12 weeks before removal, if necessary or desired. Controversy exists over use of 1 or 2 screws, screw size, and purchase of 3 or 4 cortices.

     

    Gardner et al. compared radiographic measurements vs CT scans to assess reduction of the tibiofibular syndesmosis in ankle fractures. They found CT was better able to detect syndesmotic malreduction. Although they did not seek to correlate this with functional outcomes, they recommended heightened vigilance for assessing accurate syndesmosis reduction.

     

    Zalavras et al. performed a review on ankle syndesmotic injuries. In their review, they highlight that syndesmotic injuries may occur in isolation or may be associated with ankle fractures. In the absence of fracture, physical examination findings suggestive of injury include ankle tenderness over the anterior aspect of the syndesmosis and a positive squeeze or external rotation test. They recommend stress testing for detecting syndesmotic instability with fixation of the syndesmosis when evidence of a diastasis is present.

     

    Figures A and B show an ankle fracture treated with ORIF and syndesmotic repair, with syndesmotic and fibular malreduction.

     

    Incorrect Answers:

    Answer 1&5: Would not address the current syndesmotic malreduction which should be addressed.

    Answer 2: Would not address the syndesmotic malreduction.

    Answer 3: Fibular length appears appropriate on AP and lateral radiographs.

     

     

     

    OrthoCash 2020

     

  110. A 45-year-old patient sustains the injury shown in figure A. What radiographic finding most highly suggests a lateral meniscal injury?

     

     

     

    1. Joint depression of 3mm

    2. Ipsilateral femoral shaft fracture

    3. Joint widening of 6mm

    4. Ipsilateral tibial shaft fracture

    5. Displaced tibial spine fracture Corrent answer: 3

    Figure A shows a Schatzker II tibial plateau fracture. Joint widening of 6mm is commonly asociated with a lateral meniscal injury.

     

    Tibial plateau fractures are commonly associated with soft tissue injuries and the operative surgeon needs to be aware of these commonly associated injuries.

     

    Gardner et al. (2005) examined 62 consecutive Schatzker type II fractures with radiographs and MRIs preoperatively. They found that joint depression greater than 6mm and widening of greater than 5mm was associated with a lateral meniscal injury over 80% of the time.

     

    Gardner et all. (2006) that looked at MRIs for 103 consecutive patients with all types of tibial plateau fractures, only one patient had no soft tissue injuries. 94 (91%) patients had a lateral meniscal injury, 79 (77%) patients had cruciate ligament injury and 70 (68%) patients had a posterior lateral corner injury.

    Figure A: shows a Schatzker type II fracture of a tibial plateau.

     

    Incorrect Answers:

    Answer 1: The above mentioned articles only found associations with lateral meniscal injuries when joint depression was at least 6mm.

    Answer 2: No known association with soft tissue injuries after tibial plateau fractures.

    Answer 4: No known association with soft tissue injuries after tibial plateau fractures.

    Answer 5: Suggestive of an ACL injury, not an injury to the lateral meniscus.

     

     

     

     

    OrthoCash 2020

     

  111. A 37-year-old male is struck by a car while walking at night. He is hemodynamically unstable at initial evaluation in the trauma bay. Advanced Trauma Life Support protocols are started, and an initial survey is completed. A chest radiograph and a pelvis AP radiograph (Figure A) are obtained. What is the most appropriate next step?

     

     

     

     

    1. The patient should be taken directly to the OR for percutaneous placement of a pelvic external fixator

    2. Dedicated inlet and outlet views of the pelvis to better classify the fracture

    3. Continued resuscitation and immediate CT of the chest, abdomen and plevis

    4. Emergent trip to interventional radiology for pelvic embolization

    5. Immediate application of pelvic binder, continued resuscitation and re-evaluation of hemodynamic status

    Corrent answer: 5

    The patient has an Anterior-Posterior Compression type 3 pelvic ring injury (APC3), and this injury places the patient at risk of life- threatening hemorrhage. The most appropriate next step in the trauma bay is to place the patient in a pelvic binder in order to minimize pelvic volume and impart stability to the injured hemipelvis to allow for clot formation.

     

    Pelvic fractures are high energy injuries with a high association of concomitant musculoskeletal trauma and damage to multiple organ systems. It is important that any patient with a high-energy pelvic ring injury undergo a complete

    work-up including a CT of the chest abdomen and pelvis to look for alternative sources of bleeding. Application of a pelvic binder should occur once a pelvic ring injury is identified as part of the ongoing resuscitation of the patient.

     

    Karadimas et al. retrospectively reviewed 34 patients at a single center who underwent pelvic arterial embolization as part of their resuscitation. APC injuries had the highest mean transfusion rate during the initial 24 hours, and the overall mortality for pelvic fractures requiring embolization was 23.5% in this series.

     

    Manson et al. conducted a retrospective case-controlled study, evaluating mortality factors on LC-1 fractures. They found that in LC-1 fractures, the sacral fracture pattern does not predict mortality; however, mortality rate was increased in patients with a brain injury, chest injury, or abdominal injury.

     

    Figure A demonstrates an APC3 pelvic ring injury with widening of both the symphysis and the right SI joint. Illustration A demonstrates the same injury as seen in Figure A after application of a pelvic binder with improved alignment of the pelvic ring. Illustration B shows appropriate application of a pelvic binder in a multiply injured patient.

     

    Incorrect Answers:

    Answer 1: While this patient may need to go emergently to the OR for multiple reasons, the work-up needs to be completed. However, the patient’s pelvis should be stabilized with a pelvic binder in the interim.

    Answer 2: These images should be obtained, but the pelvis should be closed with a pelvic binder first.

    Answer 3: While the pelvis may not be the only location of bleeding, the patient has a known source for bleeding, and it can be quickly stabilized with a pelvic binder. After the pelvic binder is placed, continued resuscitation and investigation of other possible locations of bleeding should occur.

    Answer 4: While this patient may benefit from embolization, the first step is to close down the pelvis. Closing down the pelvis may prevent the need for embolization.

     

     

     

     

     

     

     

     

    OrthoCash 2020

     

  112. A 33-year-old man sustains blunt trauma to his forearm and presents with the injury seen in Fig A and B. Definitive management of this injury involves the following:

     

     

     

    1. Perform closed reduction of the radius, then immobilize the forearm in a long arm cast in supination.

    2. Perform open reduction and internal fixation of the radius, then assess the proximal radioulnar joint for instability, and percutaneously fix the proximal radioulnar joint if instability persists.

    3. Perform open reduction and internal fixation of the radius, then assess the distal radioulnar joint for instability, and reconstruct the distal radioulnar joint with a looped palmaris longus autograft if instability persists.

    4. Perform closed reduction of the radius, then assess the distal radioulnar joint for instability, and perform internal fixation of the radius if instability persists.

    5. Perform open reduction and internal fixation of the radius, then assess the distal radioulnar joint for instability, and percutaneously fix the distal radioulnar joint if instability persists.

    Corrent answer: 5

     

    Galeazzi fracture-dislocations are fractures of necessity and must be managed surgically. The first step involves surgical fixation of the radial fracture. Next, the distal radioulnar joint (DRUJ) needs to be assessed for stability by looking for gross motion of the distal ulna in forearm supination. If DRUJ instability persists, this needs to be addressed with temporary percutaneous pin fixation with one or two 1.2- or 1.6mm K-wires placed transversely proximal to the sigmoid notch. This is followed by immobilization in above-elbow plaster casts

    in forearm supination for 6 weeks postop. Anatomic reduction and rigid fixation of the radius alone does not guarantee DRUJ stability.

     

    Rettig et al. found that the anatomical location of the radial shaft fracture could be used to predict DRUJ instability. Fractures within 7.5cm of the midarticular surface of the distal radius were more likely to require K wire stabilization (55%) compared with fractures more than 7.5cm away (6%).

     

    Korompilias et al. found fractures of the distal third were more likely to require DRUJ stabilization (54%) than fractures of the middle third (12%) and proximal third (11%).

     

    Giannoulis et al., in a review of Galeazzi fracture-dislocations, summarized the options as follows: (1) Stable DRUJ, cast in supination for 6 weeks; (2) Unstable DRUJ, TFCC repair and DRUJ pinning with a K wire in neutral rotation;

    (3) Unstable DRUJ with ulnar styloid fracture, ORIF of ulnar styloid with tension band wire or lag screw; (4) Irreducible DRUJ because of tendon interposition (ECU, EDC or EDM), open reduction and TFCC repair.

     

    Figures A and B are AP and lateral radiographs demonstrating a Galeazzi fracture-dislocation with marked disruption of the DRUJ. The radial head is visible in both radiographs and is not dislocated.

     

    Incorrect Answers:

    Answer 1: The radius fracture requires surgical fixation. While casting in supination may reapproximate DRUJ alignment, only pinning can prevent future subluxation.

    Answer 2: Dislocation of the radial head and proximal radioulnar joint instability is characteristic of Monteggia fracture-dislocations. In this injury complex, the proximal ulnar shaft is fractured and not the radius. Galeazzi fracture-dislocations must not be confused with a Monteggia fracture-dislocation.

    Answer 3: Tendon graft stabilization is an option for chronic DRUJ instability. Answer 4: The radial fracture must first be reduced and fixed with a plate.

    Assessment of DRUJ stability is only possible after rigid fixation of the radial fracture has been performed.

     

     

     

    OrthoCash 2020

     

  113. Which of the following is true when comparing Figure A to Figure B?

     

     

     

     

     

     

    1. Figure B is more likely to have an associated fracture

    2. Figure A is more likely to be blocked from closed reduction by the extensor digitorum brevis

    3. FIgure A is more likely to be open

    4. FIgure B is more likely to be blocked from closed reduction by the posterior tibial tendon

    5. Figure A more likely to be stable following closed reduction

    Corrent answer: 3

     

    While medial subtalar dislocations (Figure B) are more common, lateral subtalar dislocations (Figure A) are more likely to be open and have associated fractures.

     

    Subtalar dislocations are devastating injuries with high rates of subtalar arthritis. Medial dislocations are more common and are occasionally blocked from closed reduction by the extensor digitorum brevis. Lateral dislocations are more likely to be open, have an associated fracture, and are occasionally blocked from closed reduction by the posterior tibial tendon, flexor digitorum longus, or flexor hallucis longus. Despite these differences, the injuries are treated similarly with closed reduction and immobilization followed by range-of-motion exercises.

     

    DeLee et al. review 17 cases of subtalar dislocations. They found worse longterm results with lateral subtalar dislocations secondary to associated fractures and frequency of open injuries. They recommend cast immobilization for three weeks followed by early motion.

     

    Heppenstall et al. review 20 cases of subtalar dislocations. They maintained reduction in a short-leg cast for 6 weeks following the injury and had 14 excellent results. The most common complications were restriction of motion and radiographic evidence of degenerative changes.

     

    Goldner et al. review the long-term results of 15 patients who sustained a type III, open, subtalar dislocation. In contrast to closed injuries, they found that open subtalar dislocations are more likely to develop osteonecrosis of the talar body requiring pantalar arthrodesis.

     

    Figure A shows a lateral subtalar dislocation. Figure B shows a medial subtalar dislocation.

     

    Incorrect Answers:

    Answer 1: Lateral subtalar dislocations are more likely to have associated fractures.

    Answer 2: Lateral subtalar dislocations are most commonly blocked from closed reduction by the posterior tibial tendon.

    Answer 4: Medial subtalar dislocations are more commonly blocked from closed reduction by the extensor digitorum brevis.

    Answer 5: Medial subtalar dislocations are more likely to be stable, likely secondary to a decreased rate of associated fractures.

     

     

    OrthoCash 2020

     

  114. A 33-year-old motorcyclist is involved in a motor vehicle accident and sustains a Grade III open fracture of his tibia that is treated surgically. Over the next 35 years, he undergoes multiple debridements for a persistently draining wound. Over the last year, he has noticed "tissue growing out of the wound" and a malodorous smell. A photograph of the wound and a recent radiograph are seen in Figure A. A biopsy of the mass is shown in Figures B, and C. What is the most likely pathologic process?

     

     

     

     

     

     

     

     

     

    1. Infection

    2. Squamous cell carcinoma

    3. Basal cell carcinoma

    4. Melanoma

    5. Soft-tissue sarcoma

     

    Corrent answer: 2

     

    A Marjolin's ulcer is a malignant tumor (Figure A) that develops around chronic osteomyelitis. The increasing size and foul smell suggest malignant change.

    These tumors are most commonly squamous cell carcinoma (90%). Figures B and C confirm the diagnosis of squamous cell carcinoma.

     

    Marjolin's ulcers can arise in the setting of burns, venous and decubitus ulcers, vaccination, snake bites and hidradenitis suppurativa. They usually involve the lower extremities. While squamous-cell carcinoma is most common, a small percentage of cases are basal-cell carcinoma, melanoma and sarcoma.

     

    Copcu et al. examined 31 Marjolin's ulcers arising at burn sites. 58% were on the extremities, and had an average time lag of 19 years. Four of 31 cases had regional lymph node spread. They also drew attention to the fact that Marjolin's ulcers are more aggressive, with higher rates of metastasis, recurrence and fatality than other types of skin cancer.

     

    Pandey et al. reviewed the literature on Marjolin's ulcers in chronic bone infections and found that the treatment of choice was still amputation (90% of cases). The challenges faced by the surgeon considering a more conservative approach include: wide resection of cancer for cure, resection of infected bone and soft tissue, preservation of limb function, and reconstruction.

     

    Figure A shows the appearance of a fungating soft tissue tumor, together with a radiograph of chronic osteomyelitis. Figure B (low power) shows the

    transition between normal epithelium (left) and squamous cell carcinoma (right), which is infiltrating downward. Figures C is a high power image of SCC. Illustration A shows basal cell carcinoma (Answer 3) with a cleft (red arrow) separating basaloid cells (green arrow) from the stroma. Illustration B shows melanoma with asymmetric proliferation of nests of melanocytes (red arrow), descending into the dermis (Answer 4).

     

    Incorrect Answers:

    Answer 1: While there is underlying chronic osteomyelitis, the more concerning disease process is malignant transformation into squamous cell carcinoma (Marjolin's ulcer)

    Answer 3: There is no evidence of basal-cell carcinoma. Answer 4: There is no evidence of melanoma.

    Answer 5: There is no evidence of sarcoma.

     

     

     

     

     

     

     

     

     

    OrthoCash 2020

     

  115. A 36-year-old male sustains an open segmental tibia fracture associated with an overlying 8 cm soft tissue avulsion that requires

    skin grafting for soft tissue coverage. No vascular injury is identified. What is the most appropriate Gustilo-Anderson classification of this injury?

    1. Type I

    2. Type II

    3. Type IIIA

    4. Type IIIB

    5. Type IIIC

     

    Corrent answer: 3

     

    An 8cm open segmental tibia fracture requiring skin grafting (but not a skin flap) qualifies as a Gustilo Type IIIA.

     

    An open segmental tibia fracture meets criteria to be a type III injury, and subclassification as a IIIA is due to the lack of a need for free or regional tissue transfer for coverage. Classification as a type IIIB would require a free or regional soft tissue transfer (flap) for coverage. Also, remember that definitive classification is done intraoperatively, after full assessment of the fracture and wound are complete.

     

    Gustilo et al. performed a retrospective review of 673 patients and a prospective review of 352 patients with open fractures. Infection rate of the type I and II in the retrospective series was 12%, and this decreased with use of a standardized modern treatment protocol to 2.5% in the prospective cohort. Type III deep infection rate was 44% in the retrospective study and 9% in the prospective study.

     

    Incorrect Answers:

    Answer 1: Involves a wound <1 cm with minimal soft tissue stripping; simple fracture pattern

    Answer 2: Involves a wound 1-10cm with mild soft tissue stripping; simple fracture pattern or mild comminution

    Answer 4: Involves extensive soft tissue damage with high-energy fracture pattern; soft tissue requires free tissue transfer for coverage

    Answer 5: Involves a vascular injury requiring repair

     

     

     

    OrthoCash 2020

     

  116. A 25-year-old man sustains an open forearm fracture from an auger as depicted in Figures A and B. After debridement of nonviable bone, a 10cm bone defect is left. In planning future definitive treatment of the bone void, the use of an interposed strut allograft

    instead of transfer of a vascularized fibula graft would most likely result in which of the following complications?

     

     

     

     

     

     

     

    1. Higher incidence of infection

    2. Lower nonunion rate

    3. Decreased forearm arc of rotation

    4. Complex regional pain syndrome

    5. Synostosis

     

    Corrent answer: 1

     

    The patient has an open fracture with a large amount of bone loss. A higher incidence of infection would be expected with the interposed allograft strut graft treatment option.

     

    Segmental bone loss in a limb secondary to trauma, tumor resection, or other disease require reconstruction to produce a functional limb. Bone loss of up under 5 centimeters is amenable to bone grafting. Larger defects will require vascularized grafts transfers or allografts. Other surgical options include

    distraction osteogenesis and induced membrane technique. Vascularized fibular graft transfers have been shown to have good hypertrophy to support new biomechanical stresses and low rates of infection. Strut allografts have the advantage of unlimited size and no complications associated with donor site, however, complication rates including infection, fracture, and delayed union are higher than vascularized fibular grafts.

     

    Wood et al. retrospectively assessed the outcomes of using vascularized fibula grafts for upper extremity reconstruction. The investigators reconstructed 21 limbs (clavicles, ulnas, radiuses, and humeruses) with defects greater than 6cm and measured rate of union. They experienced a primary healing rate of 71.4%. This rate went up to 81% after 2 patients underwent a secondary procedure revising the construct and adding iliac bone graft to the nonunion site. They concluded that the use vascularized fibular bone grafts are successful for reconstruction of limbs with large bony defects.

     

    Falder et al. investigated the physiologic response to free vascularized fibula grafts. Thirty two graft outcomes were assessed for amount of hypertrophy of the fibular graft, bony union, and rate of limb salvage. They reported that 74% primary bony union occurred at a median of 4.75 months, median hypertrophy of the grafts was 71%, and 90% of patients regained the use of their affected limb within 12 months of the procedure. The authors concluded that vascularized free fibula flaps are an "excellent" option for limb salvage in patient with large bony defects and that these grafts respond well to biomechanical stresses.

     

    Calkins et al conducted a Level 4 study looking at segmental bone defects of the upper extremity with open injuries. The investigators performed thorough debridement and irrigation to the extremities, applied iliac corticocancellous bone grafts with fixation, and secondary wound closure for all 22 patients.

    They found a 60% rate of infection and 43% rate of union with use of corticocancellous strut grafts concluding that this technique translated well to the hand, but did not have satisfactory results in the arm and forearm for large defects.

     

    Incorrect Answers:

    Answer 2: There is a higher nonunion rate

    Answer 3: Strut allograft has not been shown to have inferior forearm motion Answer 4: Strut allograft has not been shown to be associated with complex regional pain syndrome

    Answer 5: Strut allograft has not been shown to have a higher rate of synostosis

     

     

    OrthoCash 2020

     

  117. A 46-year-old male falls 15 feet from a ladder while working. He presents with the radiographs shown in Figures A and B. The injury is closed, and soft tissues are intact upon arrival. Which of the following treatment regimens has been shown to decrease wound complications in the definitive management of these injuries?

     

     

     

     

     

    1. Immediate definitive fixation of the tibia, and nonoperative treatment of the fibula

    2. Immediate ankle-spanning external fixation device with consideration of immediate fixation of the fibula, followed by delayed reconstruction of the tibia

    3. Placement of a temporary splint, elevation, and definitive fixation 1 week from injury

    4. Immediate definitive fixation of the tibia and fibula

    5. Immediate placement of a spanning Ilizarov fixator with limited internal fixation of the distal tibia and fibula

    Corrent answer: 2

     

    Studies have shown that a staged treatment protocol consisting of immediate fixation of the fibula with placement of an ankle-spanning external fixation device followed by delayed reconstruction of the tibia results in minimal surgical wound complications.

     

    Historically, high rates of infection have been associated with open reduction and internal fixation of pilon fractures due to attempts at immediate fixation through swollen, compromised soft tissues. When a staged procedure is performed with initial restoration of fibula length and tibial external fixation, soft tissue stabilization is possible. Once soft tissue swelling has significantly diminished, anatomic reduction and internal fixation of the tibia can then be performed semi-electively with only minimal wound problems.

     

    Sirkin et al performed a study to determine whether open reduction and internal fixation of intra-articular pilon fractures using a staged treatment protocol (defined in the above paragraph) resulted in improved surgical wound complications. In their cohort of 29 patients with closed pilon injuries treated in this manner, all wounds healed, and none exhibited wound dehiscence or full-thickness tissue necrosis requiring secondary soft tissue coverage postoperatively.

     

    Patterson et al evaluated the use of a two-staged technique for the treatment of 21 consecutive patients with 22 C3 pilon fractures. All patients underwent immediate fibular fixation and placement of a medial spanning external fixator. After, on average, twenty-four days, patients underwent removal of the external fixator and formal open reduction and internal fixation of the pilon fractures. There were no infections or soft tissue complications.

     

    Figure A and Figure B demonstrate orthogonal views of a high-energy intraarticular distal tibia fracture.

     

    Incorrect Answers:

    1,3,5: These treatment regimens have not been shown in the trauma literature to be associated with decreased wound complications in the fixation of complex pilon fractures.

     

     

     

    OrthoCash 2020

     

  118. A 42-year-old man sustains the injury shown in Figure A after a fall from 6 feet. Physical exam after the injury reveals a flaccid ipsilateral limb. An MRI is performed that reveals nerve root avulsions from C5-T1. Which of the following is the most appropriate management of his fracture at this time?

     

     

     

     

    1. Closed management with a coaptation splint

    2. Closed management with a coaptation splint followed by transition to a functional brace after 7-10 days

    3. External fixation of humeral shaft fracture until brachial plexus injury resolves

    4. Open reduction, surgical fixation with plating

    5. Closed management with a sling until brachial plexus injury resolves Corrent answer: 4

    A coexisting brachial plexus injury is an absolute indication for open reduction and internal fixation of humeral shaft fractures.

     

    A majority of humeral shaft fractures may be treated non-operatively in a

    functional brace. Absolute indications of operative management include open fracture with severe soft tissue injury, vascular injury requiring repair, and a coexisting brachial plexus injury. Patients with a brachial plexus injury are more likely to go on to nonunion when treated non-operatively due to lack of muscular support controlling the fracture fragments.

     

    Sarmiento et al. review 620 patients with humeral shaft fracture treated nonoperatively in a coaptation splint followed by a functional brace. Only 16 patients developed a non-union, and any residual deformity was usually functionally and aesthetically acceptable.

     

    Rutgers et al. present a retrospective case series of 49 patients who had humeral shaft fractures that were treated non-operatively in a functional brace. 44 of 49 patients went on to successful union. Fractures of the proximal third of the humeral shaft were most likely to go on to nonunion.

    Figure A shows a midshaft humerus fracture. Incorrect Answers:

    Answer 1, 2, 5: operative management is indicated with a coexisting brachial

    plexus injury.

    Answer 3: a coexisting brachial plexus injury is not an indication for external fixation of humeral shaft fractures.

     

     

     

    OrthoCash 2020

     

  119. A 65-year-old diabetic male with forefoot gangrene is evaluated for possible amputation. When discussing the amputation levels with the patient, which of the following should be noted to require the greatest increase in energy expenditure for ambulation?

    1. Syme amputation

    2. Unilateral transtibial amputation

    3. Transfemoral amputation

    4. Bilateral transtibial amputations

    5. Through the knee amputation Corrent answer: 3

    A transfemoral level amputation requires the greatest increase in energy expenditure of the amputation levels given, and a Syme amputation the least.

     

    Biologic joints are energy couples. When performing amputation surgery, more

    proximal amputations, accompanied by the removal of more joints, decreases the ability of patients to walk and live independently. Therefore a Syme amputation would require only a small increase in energy needed for ambulation. Bilateral transtibial amputee patients have an energy expenditure that is 40% above baseline levels. According to Miller's Review text, unilateral transfemoral amputee's require a 65% increase in energy expenditure.

     

    Pinzur et al. retrospectively studied 97 adult patients with diabetes mellitus who underwent Syme ankle disarticulation because of a neuropathic foot with an infection or gangrene, or both, during an 11 year period. They found that overall 84.5% of the patients achieved wound healing. Their data support the value of Syme ankle disarticulation in diabetic patients with infection or gangrene.

     

    Laughlin et al. retrospectively reviewed the surgical results and functional outcome of 52 patients treated with Syme amputations for forefoot gangrene. Wound healing was correlated with the preoperative status of the posterior tibial artery and follow-up averaged 27 months. The authors found that that posterior tibial artery Doppler examination is predictive of healing in the Syme amputation performed on diabetics, and that furthermore, diabetics can attain a functional level of ambulation with a Syme amputation.

     

    Illustration A shows a clinical photo of a standard Syme amputation with a healthy heel pad which can allow for limited weight bearing in the absence of a prosthesis.

     

    Incorrect Answers:

    1,2,4,5: All of these amputation levels have lower energy expenditure increases than a transfemoral amputation.

     

     

     

     

     

     

    OrthoCash 2020

     

  120. A 76-year-old female with underlying osteoporosis presents with severe right leg pain after stepping off a curb. Current femur

    radiographs are shown in Figure A. Review of the patient's medical records reveal that she had been evaluated 3 months prior for right hip pain, and work-up at that time was negative. Radiographs of the patient's femur from that previous visit are shown in Figure B. What is the most likely cause of this patient's femur fracture?

     

     

     

     

     

    1. Fibrous cortical defect

    2. Metastatic lesion

    3. Acute trauma

    4. Bisphosphonate treatment

    5. Osteomyelitis

     

    Corrent answer: 4

     

    The patient's injury is most consistent with a bisphosphonate induced atypical femur fracture.

     

    Atypical subtrochanteric femoral fractures have been identified as a potential complication of long-term bisphosphonate therapy for the treatment of osteoporosis. Prodromal symptoms of thigh pain are common prior to fracture, and bony failure is usually associated with low energy mechanisms.

     

    Puhaindran et al. performed a retrospective review of the imaging studies and case notes for patients with skeletal malignant involvement who received a minimum of twenty-four doses of intravenous bisphosphonates. Patients were classified as having an atypical subtrochanteric femoral fracture if they had a transverse subtrochanteric fracture following low-energy trauma or an impending fracture, together with radiographic findings. In the study cohort of 327 patients, four patients developed an atypical subtrochanteric femoral fracture. All four patients were female, three had breast cancer, and one had myeloma.

     

    Feldman et al. reviewed the imaging presentations on routine radiographs, alternate imaging modalities, and associated pitfalls in nine atypical femur fractures in six patients. The author states that effects may differ with each bisphosphonate's route of administration and prolongation of activity, despite discontinuation. The review also discusses the theoretical mechanisms of bisphosphonates as a class rather than with a specific alendronate association and provides a broader basis for evaluating the recently observed clinical and radiographic complications.

     

    Figure A shows a transverse subtrochanteric femur fracture. Figure B shows diaphyseal cortical thickening and cortical beaking at the subtrochanteric area.

     

    Incorrect Answers:

    Answer 1: The radiographic findings are not consistent with a fibrous cortical defect.

    Answer 2: The radiographic findings are not consistent with the presence of a metastatic lesion.

    Answer 3: Subtrochanteric fractures are typically associated with high energy

    trauma, and this patients mechanism of injury is atypical.

    Answer 5: There is no radiographic or clinical evidence supporting the presence of chronic osteomyelits.

     

     

     

    OrthoCash 2020

     

  121. Risk factors for intimate partner violence after musculoskeletal injury includes all of the following EXCEPT?

    1. Shorter length of relationship

    2. Younger age

    3. Current pregnancy

    4. Alcohol dependency

    5. History of prior divorce Corrent answer: 5

    History of marriage ending in divorce has not been identified in the literature as a risk factor for intimate partner violence (emotional, physical, and sexual abuse).

     

    The approximate prevalence of intimate partner violence (emotional, physical, and sexual abuse) is 32% among females within the 1 year before they are seen by an orthopaedist for the treatment of musculoskeletal injuries.

     

    Bhandari et al. performed a level 2 cohort study of 282 females presenting to their institution for orthopaedic injuries. The prevalence of intimate partner violence (emotional, physical, and sexual abuse) within the twelve months prior to presentation was 32%.

     

    Bhandari et al. also performed a level 4 review of 263 females referred to the Minnesota Domestic Abuse Program. Identified risk factors for physical abuse included younger age, shorter length of relationship, emotional abuse, psychological abuse, sexual abuse, drug dependency, and alcohol dependency.

     

    Illustration A shows a table that outlines the roles and responsibilities of orthopaedic surgeons according to the Canadian Orthopaedic Association 2012 position statement.

     

    Incorrect Answers:

    Shorter length of relationship, younger age, current pregnancy, and alcohol dependency have all been indentified as risk factors for intimate partner violence.

     

     

     

     

     

    OrthoCash 2020

     

  122. A 30-year-old woman falls onto an outstretched arm while rollerblading. She presents to the emergency room with the elbow deformity shown in Figure A. On physical examination she is unable to range her elbow. She is distally neurovascularly intact. Her radiograph is shown in Figure B. What is the next step in management of this patient?

     

     

     

     

     

     

    1. Closed reduction, hinged external fixator

    2. Closed reduction, acute surgical repair of the lateral collateral ligament complex

    3. Open reduction and surgical repair of the lateral collateral ligament complex

    4. Closed reduction, splinting & early passive ROM

    5. Closed reduction, splinting & early active ROM Corrent answer: 5

    The clinical presentation is consistent with a simple elbow dislocation. Initial management should include closed reduction, splinting or sling placement for comfort and early active ROM exercises.

    Simple elbow dislocations should be reduced and treatment guided by the relative stability of the joint during the arc of motion. The elbow should be splinted where it is most stable. After 5-7 days the splint can be discontinued and active ROM started to apply compressive stability to the joint. A hinged brace with an appropriate extension block can facilitate motion through the stable arc.

     

    Armstrong discusses pathomechanics and basis for treatment in simple elbow dislocations. She emphasizes delineating the arc through which the elbow is stable and using this to guide initial management. Early surgical management may be warranted when the elbow is unstable with 60 degrees or more of flexion.

     

    Josefsson et al. studied 30 patients who had sustained simple elbow dislocations and prospectively randomized the patients into non-operative and operative study arms. Operative intervention included reattachment of avulsed ligaments/muscles from the distal humerus. They found no statistical difference in clinical outcomes between the groups regardless of the severity of the instability.

     

    Figure A demonstrates a clinical photo of a patient having sustained an elbow dislocation. Figure B is a radiograph demonstrating a posterolateral elbow dislocation, which is named based on the direction that the ulna/radius move relative to the humerus. Illustration A is a video that reviews the pathomechanics of elbow dislocations.

     

    Incorrect Answers

    Answer 1, 2: While use of a hinged external fixator may be appropriate when instability persists after repair/reconstruction of the LCL complex, it is not the first step in the initial management of a simple elbow dislocation.

    Answer 3: Open reduction may be warranted if closed reduction is unsuccessful, however an initial attempt at conservative measures should be made.

    Answer 4: Closed reduction needs to be followed by splinting to give the tissues a chance to recover from the initial injury; use of ACTIVE range of motion exercises early, will help to provide dynamic compressive stability to the joint.

     

     

     

    OrthoCash 2020

     

  123. A 21-year-old male undergoes intramedullary nailing of the closed tibial shaft fracture shown in Figure A. At his 6-week follow-up,

    he is noted to have peroneal nerve deficits that were not present preoperatively. Which of the following findings is most consistent with a diagnosis of transient peroneal nerve neurapraxia as the result of his intramedullary nailing?

     

     

     

     

    1. Decreased lateral hindfoot sensation

    2. Decreased Achilles reflex

    3. Decreased peroneus longus strength

    4. Decreased extensor hallucis longus strength

    5. Decreased plantar forefoot sensation Corrent answer: 4

    Decreased extensor hallucis longus strength is the physical exam finding most consistent with transient peroneal nerve neurapraxia.

     

    Transient peroneal nerve neuropraxia has been noted in up to 5% of patients undergoing closed nailing of tibial shaft fractures. This is of unknown etiology, although injury to the peroneal nerve branches can be from placement of the interlocking screws into the nail. In this syndrome, extensor hallucis longus weakness is noted and decreased sensation is seen in the 1st dorsal webspace (deep peroneal nerve distribution). These symptoms generally begin improving by 3 months and have variable rates of recovery.

    Robinson et al. reported on 'dropped hallux' syndrome, with weakness of extensor hallucis longus and numbness in the first web space, without clinical involvement of extensor digitorum longus or tibialis anterior. They found this in 5% of their patients after tibial nailing of closed tibia fractures, and all recovered either partially or completely by 4 months.

     

    Lawrence et al. performed an anatomic study that reported the deep peroneal nerve was located superficial to the anterior tibial artery between the tibialis anterior and extensor hallucis longus muscles in the distal one third of the leg. They also found that the deep peroneal nerve crossed deep to the extensor hallucis longus tendon to enter the interval between the extensor hallucis longus and extensor digitorum longus at an average distance of 12.5 mm proximal to the ankle.

    Figure A shows a tibial shaft fracture with mild displacement. Incorrect Answers:

    Answer 1: Lateral hindfoot sensation is generally via the sural nerve.

    Answer 2: Achilles reflex is via S1 roots.

    Answer 3: The peroneus longus is innervated by the superficial peroneal nerve. Answer 5: Plantar forefoot sensation is via the tibial nerve along with the medial and lateral plantar branches.

     

     

     

    OrthoCash 2020

     

  124. A 24-year-old male presents after being shot in the lower back. Radiographs and advanced imaging show that the bullet traversed the paraspinal musculature, entered the pelvis below the pelvic brim, did not cross the midline, and is currently lodged in the ilium. No intraarticular involvement was noted, and the bony injury is stable. No occult blood is noted on the rectal examination. The police report indicates that the bullet was fired from a low-velocity weapon. Which of the following is the most important treatment for this patient assuming he is hemodynamically stable?

    1. Immediate exploratory laparotomy and bullet removal

    2. Bullet removal followed by surgical stabilization of the ilium

    3. Immediate surgical debridement of the bullet tract and delayed closure of the soft tissue wound

    4. Empiric antibiotic therapy and observation

    5. Sigmoidoscopy

     

    Corrent answer: 4

    Based on the clinical presentation, this patient should be treated initially with empiric antibiotic therapy and observation.

     

    The available literature shows that transabdominal GSW with intra-articular contamination should be urgently débrided and irrigated; extra-articular transabdominal GSW with stable fracture patterns may be managed with observation and empiric antibiotics. Bullets lodged in intra-articular locations should be removed, but retained bullets in other anatomic locations do not necessarily warrant removal.

     

    Watters et al. retrospectively collected Information regarding surgical débridement of pelvis low velocity gunshot wounds, retained foreign bodies, and/or concomitant gastrointestinal (GI) injury was analyzed for relationships of such events to infection rate in 56 patients. The authors found that there was no increased incidence of infection in the absence of aggressive surgical débridement of pelvis gunshot wounds. The study also demonstrated that retained bullets and bullet fragments did not increase the risk of infection, even after penetrating the GI tract organs.

     

    Zura et al. completed a review article on the treatment of gunshot wounds to the hip and pelvis. The authors state that antibiotic treatment usually is indicated for patients with gunshot wounds to the pelvis, but is controversial in patients with low-energy wounds. Furthermore they state that fractures are treated based on their stability and wound care should be determined by the personality of the injury, and not solely based on the velocity of the missile.

     

    Miller et al. completed a recent review article on transabdominal gunshot wounds to the hip and pelvis. They state that extra-articular transabdominal GSW with stable fracture patterns may be managed with observation and empiric antibiotics.

     

    Incorrect Answers:

    Answer 1: Immediate exploratory laparotomy would be indicated in the setting of persistent hemodynamic instability or positive peritoneal signs.

    Answer 2: There is no evidence that bullet removal is necessary when it is in an extra-articular location. Furthermore the bony injury is stable, and does not warrant fixation.

    Answer 3: Immediate surgical debridement is not necessary without intraarticular involvement or severe soft tissue injury.

    Answer 5: Sigmoidoscopy would be necessary if occult blood was noted on rectal examination or if the bullet crossed the midline below the pelvic brim.

     

     

    OrthoCash 2020

     

  125. An 88-year-old female presents after a fall onto her left arm. She reports isolated left elbow pain, and radiographs are shown in Figure

    1. She lives in an assisted living facility, and reports no other major medical problems. The best functional outcome can be expected with which of the following definitive treatment options?

       

       

       

       

      1. Open reduction internal fixation of the distal humerus fracture

      2. Nonsurgical management with early passive range of motion exercises

      3. Initial nonsurgical management followed by interpositional arthroplasty when the fracture has healed

      4. Total elbow arthroplasty

      5. Excision of the capitellar fragments and fixation of the trochlear fragments Corrent answer: 4

        This elderly, low demand patient is presenting with a distal and comminuted distal humerus fracture in the setting of poor bone quality. These are extremely difficult to reconstruct and rehabilitate from, and therefore in this particular case total elbow arthroplasty is the best option.

         

        Total elbow arthroplasty has become the treatment of choice for complex, comminuted distal humerus fractures in patients older than age 70. It yields a faster recovery with more predictable functional outcomes, although limitations of lifting weight more than 10-15 pounds must be followed to avoid loosening.

         

        Kamineni et al. retrospectively reviewed 49 acute distal humerus fractures in 48 patients who were treated with total elbow arthroplasty as the primary option. Forty-three of these fractures were followed for 2 years. At the latest follow-up examination, the average flexion arc was 24 degrees to 131 degrees

        and the Mayo elbow performance score averaged 93 of a possible 100 points. This review supports a recommendation for total elbow arthroplasty for the treatment of an acute distal humerus fracture when strict inclusion criteria are observed.

         

        Frankle et al. compared open reduction and internal fixation (ORIF) with total elbow arthroplasty (TEA) for intraarticular distal humerus fractures in women older than 65 years of age. Follow-up was a minimum of two years. Using the Mayo Elbow Performance score, the outcomes of the 12 patients treated with ORIF were as follows: 4 excellent, 4 good, 1 fair, and 3 poor. Outcomes of the 12 patients treated with TEA were as follows: 11 excellent and 1 good. There were no fair or poor outcomes in the TEA group, and no patients treated with TEA required revision surgery.

         

        The AP and lateral elbow radiographs shown in Figures A demonstrate a severely comminuted and displaced distal humerus fracture.

         

        Incorrect Answers:

        Answer 1: Stable ORIF is nearly impossible to attain in the setting of a very distal fracture of the humerus with comminution and poor bone quality.

        Answer 2: Although nonsurgical management of these injuries showed some favor in the past, range of motion and pain control are ultimately better with TEA in the appropriate patient.

        Answer 3: Interpositional arthroplasty is indicated in younger patients who may not be able to comply with the strict weight lifting restrictions of a TEA. Answer 5: Fixation of the trochlear fragments would be extremely difficult in this case, and capitellar excision would likely lead to an unstable elbow joint in the setting of a compromised ulnohumeral articulation.

         

         

         

        OrthoCash 2020

         

  126. A 52-year-old carpenter falls off of a balcony while at work and sustains the injury shown in Figure A. The patient's BMI is 52 and he smokes 2 packs of cigarettes per day; a clinical photograph of the limb is shown in Figure B. What is the most appropriate next step in management?

     

     

     

    1. Short leg splint placement and transition to short leg cast at 2 weeks

    2. Closed reduction and spanning external fixation of the ankle

    3. Open reduction and internal fixation of the fibula and tibia

    4. Open reduction and internal fixation of the fibula with Blair arthrodesis of the ankle

    5. Open reduction and internal fixation of the tibia and articulating external fixation of the ankle

    Corrent answer: 2

     

    Closed reduction with spanning external fixation of the ankle is the most appropriate next step in management with planned definitive tibia ORIF when soft tissue swelling allows. Fixation of the fibula at the time of external fixation has been well-described but is a controversial subject.

     

    Pilon fracture treatment with a staged procedure is performed with initial restoration of length and tibial external fixation. Once soft tissue swelling has significantly diminished, anatomic reduction and internal fixation of the tibia can then be performed semi-electively with only minimal wound problems.

     

    Sirkin et al performed a Level 4 retrospective review of open reduction and internal fixation of intra-articular pilon fractures using a staged treatment protocol. The protocol consisted of ORIF of the fibula and application of an external fixator spanning the ankle joint within twenty-four hours. Patients then underwent formal open reconstruction of the articular surface by plating when soft tissue swelling had subsided at around the 2 week mark. This protocol resulted in improved surgical wound complications. In their cohort of 29 patients with closed pilon injuries treated in this manner, all wounds healed, and none exhibited wound dehiscence or full-thickness tissue necrosis requiring

    secondary soft tissue coverage postoperatively.

     

    Illustration A shows an external fixator used for initial stabilization of a pilon fracture.

     

    Incorrect Answers:

    Answer 1: Nonoperative management is not indicated in displaced pilon fractures.

    Answer 3 & 5: Immediate ORIF of the tibia plafond fracture has a higher incidence of wound complications.

    Answer 4: Blair fusion of the ankle is not indicated in pilon fractures

     

     

     

     

     

     

    OrthoCash 2020

     

  127. A 23-year-old male right hand dominant minor league hockey player sustains the injury shown in Figure A and B. The patient is apprised of the risks and benefits of both conservative and surgical treatments. He chooses to undergo surgical intervention and wishes to minimize the chance of requiring a second operation. Which of the following is the most appropriate surgical procedure for this patient?

     

     

     

     

     

     

    1. Distal clavicle resection

    2. Transacromial wire fixation with possible coracoclavicular ligament reconstruction

    3. Coracoclavicular screw fixation

    4. Hook plate fixation with coracoclavicular ligament reconstruction

    5. Small fragment plate fixation with possible coracoclavicular ligament reconstruction

    Corrent answer: 5

     

    Small fragment plate fixation with possible coracoclavicular ligament reconstruction is the most appropriate treatment for a displaced distal clavicle fracture in a patient that wishes to avoid a second procedure.

     

    The Neer classification describes lateral third distal clavicle fractures. Type II fractures (where the proximal fragment is detached from the CC ligaments and the distal fragment remains attached to the scapula via the AC joint capsule) and Type V (where a inferior clavicular fragment remains attached to the CC ligaments) are often displaced and are believed to have a higher rate of nonunion. Treatment of distal third as well as midshaft clavicle nonunions is best treated with open reduction and internal fixation of the nonunion.

     

    Banerjee et al performed a Level 5 review of the management of distal clavicle fractures. They recommend primary nonsurgical management of type I, type III, and nondisplaced type II distal clavicle fractures. For patients with displacement, the authors offer surgical treatment but counsel them that the current evidence suggests equivalent outcomes between surgical and nonsurgical treatment.

    Oh et al. performed a Level 4 systematic review of 425 cases from 21 studies. With surgical treatment, the nonunion rate was not significantly different among the modalities (p = 0.391). The complication rate was significantly higher in cases of the hook plate (40.7%) and the K-wire plus tension band wiring (20.0%) than those of the coracoclavicular stabilization (4.8%), the intramedullary (2.4%) and the interfragmentary fixation (6.3%).

     

    Jin et al present a Level 4 review of 17 patients that underwent CC screw fixation of their distal clavicle fracture. They routinely removed the CC screw at 8 weeks as a second procedure and noted that all but one patient had a good outcome based on the UCLA score.

     

    Figures A and B demonstrate displaced Type V distal clavicle fractures. Illustration A and B demonstrate an example of the use of a hook plate for distal clavicle fixation. Illustration C shows an example of a CC screw fixation and Illustration D demonstrates a distal clavicle plate fixation. Illustration E depicts the Neer classification for distal clavicle fractures. Illustration F displays a distal clavicle fracture ORIF with small fragment locking plate.

     

    Incorrect Answers:

    Answer 1: Isolated distal clavicle excision may lead to instability if the fragments are attached to the CC ligaments.

    Answer 2: Transacromial wire fixation has a high rate of secondary procedure for wire removal and concern for hardware breakage and migration.

    Answer 3: Coracoclavicular screw fixation routinely requires a secondary procedure for removal of the screw.

    Answer 4: Hook plate fixation has a high rate of secondary procedure for plate removal to prevent acromial osteolysis.

     

     

     

     

     

     

     

     

     

     

     

     

     

     

     

     

     

     

     

    OrthoCash 2020

  128. A 32-year-old male sustains the injury shown in Figures A through D as the result of a high-speed motorcycle collision. He initially undergoes spanning external fixation and returns to the office for soft tissue evaluation prior to his definitive surgery. During this visit, you discuss that the most appropriate fixation is which of the following?

     

     

     

     

     

     

     

     

     

     

     

     

    1. Lateral precontoured locked plating

    2. Posterior buttress plating

    3. Medial antiglide plating

    4. Anterolateral and posteromedial plating

    5. Posterolateral neutralization plating Corrent answer: 2

    Posterior tibial plateau fractures are typically sustained from an axial load with the knee in flexion, leading to a large posterior shearing force on the tibial plateau from the condyles. This most commonly leads to a posterior fracture fragment, with variable involvement of the posteromedial or posterolateral joint line. The shearing nature of this fracture leads to an unstable fracture pattern.

     

    Bhattacharyya et al reviewed 13 patients who underwent fixation of posterior tibial plateau fractures and noted a consistent fracture pattern with a primary, inferiorly displaced posteromedial shear fracture. They reported excellent clinical outcomes at a mean of 20 months, with the quality of articular reduction being one factor that correlated with functional outcome scores.

     

    Fakler et al. was the first English-language study to publish the surgical outcomes of posterior tibial plateau fractures using the Lobenhoffer approach. In their two case study, the authors suggest that the Lobenhoffer approach is less morbid to the soft-tissues in comparison to other commonly described posterior approaches.

     

    Figures A, B, C, and D show a tibial plateau fracture with posterior shear involvement. Illustrations A and B are postoperative images showing the same fracture pattern after posterior buttress plating.

     

    Incorrect Answers:

    Answer 1: This fracture fragment is not adequately captured with a lateral based plate; the lateral cortex is also intact, which limits the indications for this plate.

    Answer 3: The primary fracture fragment is posterior and not medial. Answer 4: No fracture is seen laterally; posteromedial plating can be utilized, however.

    Answer 5: No fracture fragment is seen posterolaterally.

     

     

     

     

     

     

     

     

    OrthoCash 2020

     

  129. A 40-year-old female sustains the injury seen in Figure A. What other associated soft-tissue knee injury is most commonly associated with this fracture?

     

     

     

    1. Anterior cruciate ligament midsubstance tear

    2. Horizontal cleavage lateral meniscus tear

    3. Peripheral lateral meniscus tear

    4. Lateral collateral ligament and popliteofibular ligament tear

    5. Lateral meniscus posterior root avulsion Corrent answer: 3

    Figure A shows a displaced Schatzker II (lateral split-depression) tibial plateau fracture. These injuries are commonly associated with peripheral meniscal tears.

     

    According to the referenced article by Abdel-Hamid et al, the overall incidence of associated soft tissue injuries of the knee in tibial plateau fractures is 71%. The incidence of each injury in their study was: meniscal tears (57% - usually peripheral tears), ACL (25% - more common in more severe fractures), PCL (5%), LCL (3%), MCL (3%), peroneal nerve (1%). No vascular injury was seen in their collection of 98 patients.

     

     

     

    OrthoCash 2020

  130. The posterior wall of the acetabulum is best visualized on which of the following radiographic views?

  1. Inlet pelvis

  2. Outlet pelvis

  3. Anteroposterior pelvis

  4. Obturator oblique pelvis

  5. Iliac oblique pelvis Corrent answer: 4

The posterior wall is best visualized on the obturator oblique pelvic view.

 

The obturator oblique and iliac oblique views make up the Judet views that are used to evaluate acetabular fractures, along with a standard AP pelvis radiograph. The obturator oblique pelvic view is best to view the anterior column and posterior wall in detail. The iliac oblique shows the profile of involved iliac wing, the posterior column, and the anterior wall.

 

Letournel reviewed his classification and treatment protocols, based on his 22 years of experience at that time. He noted that perfect anatomical reduction of the acetabulum led to the best outcomes.

 

Patel et al reviewed of the Letournel classification, and they found moderate to high inter- and intra-observer reliability with this classification system. The presence of articular displacement, marginal impaction, incongruity, intraarticular fragments and osteochondral injuries to the femoral head were found to have less reliability (intra- and interobserver).

 

Illustration A shows a right-sided obturator oblique radiograph, while Illustration B shows a right-sided iliac oblique radiograph. Illustration C shows a diagram of the obturator oblique radiograph, with the radiographic lines marked out.

 

Incorrect Answers:

Answers 1: Inlet pelvic imaging is best for assessing pelvic ring injuries (rotation and anterior-posterior or medial-lateral translation).

Answers 2: Outlet pelvic imaging is best for assessing pelvic ring injuries (proximal-distal translation, rotation).

Answers 3: AP pelvis is a good screening tool for pelvic and acetabular fractures.

Answers 5: Iliac oblique pelvis is best for assessing the posterior column and anterior wall of the acetabulum.